You are on page 1of 272

F7 Course notes

1! acowtancy.com
APPROACH TO EXAMINING THE SYLLABUS

The syllabus is assessed by three-hour paper based examination.

All questions are compulsory. It will contain both computational and discursive
elements. Some questions will adopt a scenario/case study approach.

Section A of the exam comprises 20 multiple choice questions of 2 marks each.

Section B of the exam comprises two 15 mark questions and one 30 mark question.

The 30 mark question will examine the preparation of financial statements for either a
single entity or a group. The section A questions and the other questions in section B
can cover any area of the syllabus.

An individual question may often involve elements that relate to different subject areas
of the syllabus. For example the preparation of an entity’s financial statements could
include matters relating to several accounting standards.

Questions may ask candidates to comment on the appropriateness or acceptability of


management’s opinion or chosen accounting treatment. An understanding of accounting
principles and concepts and how these are applied to practical examples will be tested.

Questions on topic areas that are also included in Paper F3 will be examined at an
appropriately greater depth in this paper.

Candidates will be expected to have an appreciation of the need for specified


accounting standards and why they have been issued. For detailed or complex
standards, candidates need to be aware of their principles and key elements.

(ACCA Syllabus Sept 2017 to Mar 2018)

2! acowtancy.com
CHAPTER 1:

A REGULATORY FRAMEWORK
FOR FINANCIAL REPORTING

1. ACCA SYLLABUS GUIDE OUTCOME 1:-


Explain why a regulatory framework is needed.

A regulatory framework is needed to ensure relevant and reliable information is given to


users. Financial statements are used by a wide range of users. Thus, these statements
need to be useful to these users.

A regulatory framework regulates the behaviour of companies towards their investors.


They increase users’ understanding of, and their confidence, in financial statements.

2. ACCA SYLLABUS GUIDE OUTCOME 2:-


Explain why accounting standards on their own are not a complete regulatory
framework.

Accounting standards on their own are not a complete regulatory framework. Legal and
market regulations are also required to regulate the preparation and presentation of
financial statements.

3. ACCA SYLLABUS GUIDE OUTCOME 3:-


Distinguish between a principles-based and a rules-based framework and
discuss whether they can be complementary.

Principles-based Framework

Principles which reflect the initial objectives of financial statements are set. All
accounting standards then follow these principles.

Therefore, a principles-based framework is based upon a conceptual framework. In the


next chapter, we will be discussing the IASB’s Conceptual Framework for Financial
Reporting, which is a conceptual framework.

3! acowtancy.com
Rules-based Framework

Rules are laid out as events arise, designed to cover all eventualities. Therefore,
accounting standards are a set of rules which companies must follow.

1.4 ACCA SYLLABUS GUIDE OUTCOME 4:-


Describe the IASB’s Standard setting process including revisions to and
interpretations of standards.

International Financial Reporting Standards (IFRSs) are developed through an


international consultation process, the "due process".

The due process comprises six stages1:-


1. Setting the agenda – the IASB identifies a subject (mainly by reference to the
needs of the investors)
2. Planning the project - after considering the nature of the issues and the level of
interest among constituents, the IASB may establish a working group at this
stage.
3. Developing and publishing the discussion paper - Typically, a discussion paper
includes:
a. a comprehensive overview of the issue;
b. possible approaches in addressing the issue
c. the preliminary views of its authors or the IASB;
d. an invitation to comment.
4. Developing and publishing the exposure draft for public comment, which is a
draft version of the intended standard.
5. Developing and publishing the standard following the consideration of
comments received on the exposure draft.
6. After the standard is issued, the staff and the IASB members hold regular
meetings with interested parties, including other standard-setting bodies, to help
understand unanticipated issues related to the practical implementation and
potential impact of its proposals.

1 www.iasb.org

4! acowtancy.com
1.5 ACCA SYLLABUS GUIDE OUTCOME 5:-
Explain the relationship of national standard setters to the IASB in respect of the
standard setting process.

In order to achieve its objectives, the IASB works in partnership with the major national
standard-setting bodies:-
1. They coordinate each other’s work plans.

2. They review each other’s standards and give priority where differences are
greatest.

3. National standard setters can issue IASB discussion papers and exposure draft
for comments in their own countries.

4. National standard setters may include more guidance in their exposure drafts on
relevant issues to them.

1.6 ACCA SYLLABUS GUIDE OUTCOME 6:-


The advantages and disadvantages of IFRS over a national regulatory
framework.2

1.6.1 What are the benefits of adopting IFRS to national standards?

IFRS have the following advantages for those companies that adopt them:

• IFRS are widely accepted as a set of high-quality and transparent global


standards that are intended to achieve consistency and comparability across the
world. They have been produced in cooperation with other internationally
renowned standard setters, with the aspiration of achieving consensus and
global convergence.

• Companies that use IFRS and have their financial statements audited in
accordance with International Standards on Auditing (ISA) will have an enhanced
status and reputation (e.g. improved credit ratings).

• The International Organisation of Securities Commissions (IOSCO) recognise


IFRS for listing purposes – thus companies that use IFRS need produce only one
set of financial statements for any securities listing for countries that are
members of IOSCO. This makes it easier and cheaper to raise finance in
international markets.

2ACCA F7 Exam June 2012 Qs 5b

5! acowtancy.com
• Companies that own foreign subsidiaries will find the process of consolidation
simplified if all their subsidiaries use IFRS.

• Companies that use IFRS will find their results are more easily compared with
those of other companies that use IFRS. This would help the company to better
assess and rank prospective investments in its foreign trading partners.

1.6.2 What are the challenges of adopting IFRS to national standards?

1. Changes to regulatory environment, i.e. laws and regulations

2. IFRS training to finance staff and regulators

3. Greater complexity in the financial reporting process

In addition to significant use of judgment, companies would need to increasingly


use fair value measures. For small and medium companies, the use of IFRSs
and the extensive disclosures they require, may significantly increase their costs
in the preparation of financial statements.

1.7 ACCA SYLLABUS GUIDE OUTCOME 7:-


Distinguish between the primary aims of not-for-profit and public sector entities
and those of profit oriented entities.

Not-for-profit organisations are distinguished from profit maximising organisations by


three characteristics.

1. Most not-for-profit organisations do not have external shareholders providing risk


capital for the business. Finance is mainly limited to donations and/or
government subsidies.
2. They do not distribute dividends, so any profit (or surplus) that is generated is
retained by the business as a further source of capital.
3. Their objective is to achieve value for money. Thus, it usually includes some
social, cultural, philanthropic, welfare or environmental dimension, which in their
absence, would not be readily provided efficiently through the workings of the
market system. In the case of a profit oriented sector, the main aim is to make a
profit and increase shareholder wealth.

6! acowtancy.com
1.8 ACCA SYLLABUS GUIDE OUTCOME 8:-
Discuss the extent to which International Financial Reporting Standards (IFRSs)
are relevant to specialised, not-for-profit and public sector entities.

Accounting standards are required to measure the financial position and performance of
organisations.
Not-for-profit and public sector entities do not have to produce financial statements for
the public but they will have to account for their income and costs. They rely on
measures that estimate the performance of the organisation in relation to:
1. effectiveness – the extent to which the organisation achieves its objectives
2. economy – the ability of the organisation to optimise the use of its productive
resources (keeping the cost of input resources as low as possible)
3. efficiency – the ‘output’ of the organisation per unit of resource consumed

Some standards will be relevant such as those relating to non-current assets,


inventories, leasing, etc. but others, e.g. earnings per share, are not so relevant3. Why?
A not-for-profit entity is not likely to have shareholders who need to assess its earnings
performance.

3 F7 Dec 14 MCQ 15

7! acowtancy.com
CHAPTER 2:

A CONCEPTUAL FRAMEWORK FOR


FINANCIAL REPORTING

2.1 ACCA SYLLABUS GUIDE OUTCOME 1:-


Describe what is meant by a conceptual framework of accounting.

In a broad sense, a conceptual framework can be seen as an attempt to define the


nature and purpose of accounting. A conceptual framework must consider the
theoretical and conceptual issues surrounding financial reporting and form a coherent
and consistent foundation that will underpin the development of accounting standards.

A conceptual framework can be seen as a statement of generally accepted accounting


principles (GAAP) that form a frame of reference for the evaluation of existing practices
and the development of new ones. As the purpose of financial reporting is to provide
useful information as a basis for economic decision making, a conceptual framework will
form a theoretical basis for determining how transactions should be measured (historical
value or current value) and reported – i.e. how they are presented or communicated to
users. 4

The IASB’s Conceptual Framework for Financial Reporting describes the basic
concepts by which financial statements are prepared. The main purpose of the
Framework is to:
• assist in the development of future IFRS and the review of existing standards by
setting out the underlying concepts
• promote harmonisation of accounting regulation and standards by reducing the
number of permitted alternative accounting treatments
• assist the preparers of financial statements in the application of IFRS, which
would include dealing with accounting transactions for which there is not (yet) an
accounting standard.

The Framework is also of value to auditors, and the users of financial statements, and
more generally help interested parties to understand the IASB’s approach to the
formulation of an accounting standard. 5

4“The need for and an understanding of a conceptual framework” by Steve Scott, October 2011
http://www.accaglobal.com/content/dam/acca/global/PDF-students/2012/sa_oct11_framework.pdf
5“The need for and an understanding of a conceptual framework” by Steve Scott, October 2011
http://www.accaglobal.com/content/dam/acca/global/PDF-students/2012/sa_oct11_framework.pdf

8! acowtancy.com
The Framework is not an accounting standard, and where there is perceived to be a
conflict between the Framework and the specific provisions of an accounting standard
then the accounting standard prevails.6

The content of the Framework can be summarised as follows:


• Identifying the objective of financial statements
• The reporting entity (to be issued)
• Identifying the parties that use financial statements
• The qualitative characteristics that make financial statements useful
• The remaining text of the old Framework dealing with elements of financial
statements: assets, liabilities equity income and expenses and when they should
be recognised and a discussion of measurement issues (for example, historic
cost, current cost) and the related concept of capital maintenance.

2.2 ACCA SYLLABUS GUIDE OUTCOME 2:-


Discuss whether a conceptual framework is necessary and what an alternative
system might be7 .

Some accountants have questioned whether a conceptual framework is necessary in


order to produce reliable financial statements. Past history of standard setting bodies
throughout the world tells us it is. In the absence of a conceptual framework, accounting
standards were often produced that had serious defects – that is:

• they were not consistent with each other particularly in the role of prudence
versus accruals/matching
• they were also internally inconsistent and often the effect of the transaction on
the statement of financial position was considered more important than its effect
on income the statement
• standards were produced on a ‘fire fighting’ approach, often reacting to a
corporate scandal or failure, rather than being proactive in determining best
policy
• some standard setting bodies were biased in their composition (i.e. not fairly
representative of all user groups) and this influenced the quality and direction of
standards
• the same theoretical issues were revisited many times in successive standards –
for example, does a transaction give rise to an asset (research and development
expenditure) or liability (environmental provisions)?

6“The IASB’s conceptual framework for financial reporting” by Tom Clendon, March 2011
http://www.accaglobal.com/content/dam/acca/global/PDF-students/acca/tech/sa_mar11_f7p2.pdf
7ACCA F7 Exam June 2012 Qs 5

9! acowtancy.com
It could be argued that the lack of a conceptual framework led to a proliferation of ‘rules-
based’ accounting systems whose main objective is that the treatment of all accounting
transactions should be dealt with by detailed specific rules or requirements. Such a
system is very prescriptive and inflexible, but has the attraction of financial statements
being more comparable and consistent.

By contrast, the availability of a conceptual framework could lead to ‘principles-based’


system whereby accounting standards are developed from an agreed conceptual basis
with specific objectives.8

What is the alternative to a conceptual framework?

The IASB’s Conceptual Framework for Financial Reporting is based on principles.


IFRSs do contain many rules and requirements but these are based on underlying
concepts.

In reality, most accounting systems have an element of both rules and principles. Their
designation whether they are rules-based or principles-based depends on the relative
importance of the principles compared to the volume and the manner in which the rules
are derived.

2.3 The IASB’s Conceptual Framework for Financial Reporting

2.3.1 Why are financial statements prepared?

The objective of financial reporting is to provide financial information about the reporting
entity that is useful to existing and potential investors, lenders and other creditors in
making decisions about providing resources to the entity.

Some observations about the objective: -

1. Users

This revised Framework lists as primary users of financial statements, existing or


potential investors, lenders and other creditors. Hence, it is aimed primarily at capital
providers. This does not mean that other users will not find financial reports useful; for
e.g. employees, suppliers, customers, governments and the general public. It is just

8“The need for and an understanding of a conceptual framework” by Steve Scott, October 2011
http://www.accaglobal.com/content/dam/acca/global/PDF-students/2012/sa_oct11_framework.pdf

! 10 acowtancy.com
that, in deciding on the principles for recognition, measurement, presentation and
disclosure, the information needs of capital providers are paramount.
2. Financial information
The revised Framework introduces a broader reference to financial information, i.e.
reporting of an entity’s economic resources, claims and changes therein.
Information about the nature and amounts of a reporting entity’s economic resources
and claims assists users to assess that entity’s financial strengths and weaknesses; to
assess liquidity and solvency, and its need and ability to obtain financing. A reporting
entity’s economic resources and claims are reported in the statement of financial
position.
Changes in a reporting entity’s economic resources and claims result from that entity’s
performance and from other events or transactions such as issuing debt or equity
instruments. Users need to be able to distinguish between both of these changes. This
is useful in assessing the entity’s past and future ability to generate net cash inflows.
Such information may also indicate the extent to which general economic events have
changed the entity’s ability to generate future cash inflows. These changes are
presented in the statement of comprehensive income – Statement of profit or loss and
other comprehensive income.
Information about a reporting entity’s cash flows during the reporting period also assists
users to assess the entity’s ability to generate future net cash inflows. This information
indicates how the entity obtains and spends cash, including information about its
borrowing and repayment of debt, cash dividends to shareholders, etc. The changes in
the entity’s cash flows are presented in the statement of cash flows.
Information about changes in an entity’s economic resources and claims resulting from
events and transactions other than financial performance, such as the issue of equity
instruments or distributions of cash or other assets to shareholders is necessary to
complete the picture of the total change in the entity’s economic resources and claims.
This is presented in the statement of changes in equity.

3. Decision usefulness and stewardship

Decision usefulness to capital providers is the overriding purpose of financial reporting,


as well as assessing the stewardship of resources already committed to the entity.

The ability of management to discharge their stewardship responsibilities effectively has


an effect on the entity’s ability to generate net cash inflows in the future, implying that
potential investors are also assessing management performance as they make their
investment decision.

11
! acowtancy.com
2.3.2 Underlying Assumption

The main underlying assumption is:

• Going Concern
The financial statements presume that an enterprise will continue in operation in
the foreseeable future or, if that presumption is not valid, disclosure and a
different basis of reporting are required.

What is ‘accrual basis’?

• Accrual Basis
The effects of transactions and other events are recognised when they occur,
rather than when cash or its equivalent is received or paid, and they are reported
in the financial statements of the periods to which they relate.

2.3.3 ACCA SYLLABUS GUIDE OUTCOME 3:-


Discuss what is meant by relevance and faithful representation and describe the
qualities that enhance these characteristics.
Discuss whether faithful representation constitutes more than compliance with
accounting standards.
Discuss what is meant by understandability and verifiability in relation to the
provision of financial information.
Discuss the importance of comparability and timeliness to users of financial
statements.

Qualitative Characteristics of Financial Information9

The revised Framework distinguishes between two types of qualitative characteristics


that are necessary to provide useful financial information: -

1. Fundamental qualitative characteristics (relevance and faithful representation)


and
2. enhancing qualitative characteristics (comparability (including consistency),
timeliness, verifiability and understandability).

2.3.3.1 Fundamental Qualitative Characteristics

For information to be useful, it must be both relevant and faithfully represented.

9 examined December 2013 Qs 4

! 12 acowtancy.com
1. Relevance

Influences economic decisions of user

Relevant financial information is capable of making a difference in the decisions made


by users.

Has predictive value and/or confirmatory value or both

Relevant information assists in the predictive ability of financial statements.

That is not to say the financial statements should be predictive in the sense of forecasts,
but that (past) information should be presented in a manner that assists users to assess
an entity’s ability to take advantage of opportunities and react to adverse situations.

For example, discontinued operations are presented separately in the statement of


profit or loss.

From this information, users will be better able to assess the parts of the entity that will
produce future profits (continuing operations).

Users can also judge the merits of the discontinuation:- why was that part of the
business sold? Was it to curtail the adverse effect of a loss making operation?

Materiality

Materiality is a threshold or cut-off point for information whose omission or misstatement


could influence the economic decisions of users taken on the basis of the financial
statements.

This depends on the size of the item or error judged in the particular circumstances of
its omission or misstatement.

Hence, materiality is not a matter to be considered by standard-setters but by preparers


and their auditors.

! 13 acowtancy.com
2. Faithful Representation

General purpose financial reports represent economic phenomena in words and


numbers.

To be useful, financial information must not only be relevant, it must also represent
faithfully the phenomena it purports to represent. Financial statements will generally
show a fair presentation when:

• They conform with accounting standards


• They conform with the any relevant legal requirements
• They have applied the qualitative characteristics from the Framework.

Financial information that faithfully represents economic phenomena has three


characteristics: -

❖ it is complete
❖ it is neutral
❖ it is free from error
.
The revised Framework acknowledges limitations in achieving a faithful representation,
e.g. due to inherent uncertainties, estimates and assumptions. Accordingly, financial
information might not always be entirely free from error. Faithful representation,
however, is achieved if no errors or omissions affect the description of economic
phenomena and the process applied to produce reported information has been selected
and applied without errors.

Disclosure

The entity should disclose:

• that management has concluded that the financial statements do give a fair
presentation
• that it has complied with IFRSs and Interpretations except where it has departed
in order to achieve a fair presentation
• the standard or interpretation from which the entity has departed, and an
explanation of the circumstances
• the financial impact of the departure. If the relevant regulatory framework
prohibits such departure, the circumstances should be explained

! 14 acowtancy.com
2.3.3.2 Enhancing Qualitative Characteristics10

Comparability, verifiability, timeliness and understandability are directed to enhance both


relevant and faithfully represented financial information. Those characteristics should be
maximised both individually and in combination.

1. Comparability

Users can identify similarities and differences

Comparability is fundamental to assessing the performance of an entity by using its


financial statements. Assessing the performance of an entity over time (trend analysis)
requires that the financial statements used have been prepared on a comparable
(consistent) basis. Generally this can be interpreted as using consistent accounting
policies (unless a change is required to show a fairer presentation). A similar principle is
relevant to comparing one entity with another; however it is more difficult to achieve
consistent accounting policies across entities11.

Consistent application of methods

Comparability is enhanced by the use and disclosure of consistent accounting policies.


Users can confirm that comparative information for calculating trends is comparable.
The disclosure of accounting policies at least informs users if different entities use
different policies.

Comparability should be distinguished from consistency (the consistent use of


accounting methods).

It is recognised that there are situations where it is necessary to adopt new accounting
policies (usually through new Standards) if they enhance relevance and reliability.
Consistency and comparability require the existence and disclosure of accounting
policies.

IAS 8 “Accounting Policies, changes in accounting estimates and errors” will be


discussed in more detail in Chapter 3.

10 Examinable Dec 2012 Qs 4a


11ACCA F7 Exam June 2008 Question 4

! 15 acowtancy.com
2. Verifiability

Financial information is verifiable when it enables knowledgeable and independent


observers to reach a consensus on whether a particular depiction of an event or
transaction is a faithful representation.
3. Timeliness

Timeliness means that information is available to decision-makers in time to be capable


of influencing their decisions.

4. Understandability

Understandability is enhanced when the information is:

1. classified
2. characterised
3. presented clearly and concisely

However, relevant information should not be excluded solely because it may be too
complex and cannot be made easy to understand. To exclude such information would
make financial reports incomplete and potentially misleading. Financial reports are
prepared for users who have a reasonable knowledge of business and economic
activities and who review and analyse the information with diligence.
The Cost Constraint on Useful Financial Reporting

Cost is a pervasive constraint to financial reporting. Reporting such information imposes


costs and those costs should be justified by the benefits of reporting that information.
The IASB assesses costs and benefits in relation to financial reporting generally, and
not solely in relation to individual reporting entities. The IASB will consider whether
different sizes of entities and other factors justify different reporting requirements in
certain situations.

2.3.4 ACCA SYLLABUS GUIDE OUTCOME 4:-


Define what is meant by ‘recognition’ in financial statements and discuss the
recognition criteria.

For an item to be recognised in the accounts it must pass three tests:


a) Meet the definition of an asset/liability or income/expense or equity
b) It is probable that any future economic benefit associated with the item will flow
to or from the entity
c) The item has a cost or value than can be reliably measured

! 16 acowtancy.com
2.3.5 ACCA SYLLABUS GUIDE OUTCOME 5:-
Apply the recognition criteria to:
a. assets and liabilities
b. income and expenses

Asset

An asset is a resource controlled by the enterprise as a result of past events and from
which future economic benefits (normally net cash inflows) are expected to flow to the
entity.

Assets can only be recognised in the statement of financial position when those
expected benefits are probable and can be measure reliably12.

While most assets will be both controlled and legally owned by the entity, it should be
noted that legal ownership is not a prerequisite for recognition, rather it is control that is
the key issue.

Liability

A liability is a present obligation of the enterprise arising from past events, the
settlement of which is expected to result in an outflow from the enterprise of resources
embodying economic benefits (normally cash)13.

Liabilities are also presented on the statement of financial position as being non-current
or current. Examples of liabilities include trade payables, tax creditors and loans.

It should be noted that in order to recognise a liability there does not have to be an
obligation that is due on demand but rather there has to be a present obligation. Thus
for example IAS 37, Provisions, Contingent Liabilities and Contingent Assets is
consistent with the Framework's approach when considering whether there is a liability
for the future costs to decommission oil rigs. As soon as a company has erected an oil
rig that it is required to dismantle at the end of the oil rig's life, it will have a present
obligation in respect of the decommissioning costs. This liability will be recognised in full
as a non-current liability and measured at present value to reflect the time value of
money. The past event that creates the present obligation is the original erection of the

12 ACCA F7 Exam Dec 07 Q5a; PP Q4bii


13 ACCA F7 Exam Dec 08 Q4

! 17 acowtancy.com
oil rig as once it is erected the company is responsible to incur the costs of
decommissioning.14

Equity

Equity is the residual interest in the assets of the enterprise after deducting all its
liabilities.

This definition acknowledges the supreme conceptual importance of identifying,


recognising and measuring assets and liabilities, as equity is conceptually regarded as
a function of assets and liabilities, i.e. a balancing figure.

Equity includes the original capital introduced by the owners, i.e. share capital and
share premium, retained earnings, unrealised asset gains in the form of revaluation
reserves and, in group accounts, the equity interest in the subsidiaries not enjoyed by
the parent company, i.e. the non-controlling interest (NCI) (will be discussed in Chapter
5). Equity can also include the equity element of convertible loan stock (will be
discussed in Chapter 15), equity settled share based payments, differences arising
when there are increases or decreases in the NCI, group foreign exchange differences
and contingently issuable shares. These would probably all be included in equity under
the umbrella term of Other Components of Equity.15

Income

Income is increases in assets (or decreases of liabilities) that result in increases in


equity, other than contributions from equity participants.

Most income is revenue generated from the normal activities of the business in selling
goods and services, and as such is recognised in the Income section of the Statement
of Profit or Loss and other Comprehensive Income. However certain types of income
are required by specific standards to be recognised directly to equity, i.e. reserves, for
example certain revaluation gains on assets. In these circumstances the income (gain)
is then also reported in the Other Comprehensive Income section of the Statement of
Profit or Loss and other Comprehensive Income.

The reference to ‘other than those relating to contributions from equity participants’
means that when the entity issues shares to equity shareholders, while this clearly

14 “The IASB’s Conceptual Framework for Financial Reporting”, T. Clendon, March 2011

http://www.accaglobal.com/content/dam/acca/global/PDF-students/acca/tech/sa_mar11_f7p2.pdf
15 “The IASB’s Conceptual Framework for Financial Reporting”, T. Clendon, March 2011

http://www.accaglobal.com/content/dam/acca/global/PDF-students/acca/tech/sa_mar11_f7p2.pdf

! 18 acowtancy.com
increases the asset of cash, it is a transaction with equity participants and so does not
represent income for the entity. 16

Expense

Expenses are decreases in assets or (incurrences of liabilities) that result in decreases


in equity, other than distributions to equity participants.

The reference to ‘other than those relating to distributions to equity participants’ refers to
the payment of dividends to equity shareholders. Such dividends are not an expense
and so are not recognised anywhere in the Statement of Profit or Loss. Rather they
represent an appropriation of profit that is as reported as a deduction from Retained
Earnings in the Statement of Changes in Equity.
Examples of expenses include depreciation, impairment of assets and purchases. As
with income most expenses are recognised in the Statement of Profit or Loss section of
the Statement of Profit or Loss and Comprehensive Income, but in certain
circumstances expenses (losses) are required by specific standards to be recognised
directly in equity and reported in the Other Comprehensive Income Section of the
Statement of Profit or Loss and other Comprehensive Income. An example of this is an
impairment loss, on a previously revalued asset, that does not exceed the balance of its
Revaluation Reserve17 (will be discussed in detail in Chapter 19).

2.4 ACCA SYLLABUS GUIDE OUTCOME 6:-


Discuss revenue recognition issues; indicate when income and expense
recognition should occur.

Looking into the definitions provided in the Framework, you can see that the Framework
takes a financial position approach. It focuses on assets and liabilities and that income
and expense definitions are derived from them rather than vice-versa.

Income results from changes that increase the entity’s wealth, and losses result from
changes that decrease its wealth. Thus, revenues arise from increases in assets or
decreases in liabilities, while expenses result from decreases in assets or increases in
liabilities.

Further Questions

16 “The IASB’s Conceptual Framework for Financial Reporting”, T. Clendon, March 2011

http://www.accaglobal.com/content/dam/acca/global/PDF-students/acca/tech/sa_mar11_f7p2.pdf
17 “The IASB’s Conceptual Framework for Financial Reporting”, T. Clendon, March 2011

http://www.accaglobal.com/content/dam/acca/global/PDF-students/acca/tech/sa_mar11_f7p2.pdf

! 19 acowtancy.com
Question 118
Financial statements represent transactions in words and numbers. To be useful,
financial information must represent faithfully these transactions in terms of how they
are reported.

Which of the following accounting treatments would be an example of faithful


representation?

A. Charging the rental payments for an item of plant to the statement of profit or
loss where the rental agreement meets the criteria for a finance lease
B. Including a convertible loan note in equity on the basis that the holders are
likely to choose the equity option on conversion
C. Derecognising factored trade receivables sold without recourse
D. Treating redeemable preference shares as part of equity in the statement of
financial position

Question 219

Which of the following is NOT a purpose of the IASB’s Conceptual Framework?

A. To assist the IASB in the preparation and review of IFRS


B. To assist auditors in forming an opinion on whether financial statements
comply with IFRS
C. To assist in determining the treatment of items not covered by an existing
IFRS
D. To be authoritative where a specific IFRS conflicts with the Conceptual
Framework

Question 320

Although the objectives and purposes of not-for-profit entities are different from those of
commercial entities, the accounting requirements of not-for-profit entities are moving
closer to those entities to which IFRSs apply.

Which of the following IFRS requirements would NOT be relevant to a not-for-profit


entity?

18 Specimen Exam Applicable from December 2014

19 Specimen Exam Applicable from December 2014


20 Specimen Exam Applicable from December 2014

! 20 acowtancy.com
A. Preparation of a statement of cash flows
B. Requirement to capitalise a finance lease
C. Disclosure of earnings per share
D. Disclosure of non-adjusting events after the reporting date

Question 4
The IASB’s Conceptual Framework for Financial Reporting is

A. A set of financial reporting standards


B. A set of items which make up an entity’s financial statements
C. A set of principles which underpin financial reporting
D. A set of regulations which govern financial reporting
Question 5

The IASB conceptual framework is being developed jointly with:

A. The UK Accounting Standards Board


B. National Accounting Standard Setters
C. The European Union
D. The US Financial Accounting Standards Board

Question 6

The objective of preparing financial statements is to provide information about the


reporting entity which is useful to:

A. Investors and employees


B. Investors and lenders
C. Employees and lenders
D. Investors and customers
Question 7

The two fundamental qualitative characteristics of financial information are:

A. Relevance and faithful representation


B. Relevance and comparability
C. Faithful representation and substance over form
D. Verifiability and understandability

! 21 acowtancy.com
Question 8

The enhancing qualitative characteristics of financial information include:

A. Relevance and faithful representation


B. Comparability and understandability
C. Relevance and timeliness
D. Understandability and faithful representation

Question 9

Which of the following is not a contributory factor towards faithful representation?

A. Completeness
B. Freedom from error
C. Neutrality
D. Predictive value

Question 10

Which of the following statements about the Framework are true?


(1) The Framework is an accounting standard.
(2) It assists in harmonising accounting practice.
(3) It assists national standard setters in developing national standards.
(4) It assists users of the accounts to interpret financial statements.

A. 1 and 2
B. 2, 3 and 4
C. All 4
D. 1 and 3
CHAPTER 3:

ACCOUNTING POLICIES AND CONCEPTS

IAS 8: ACCOUNTING POLICIES, CHANGES IN ACCOUNTING ESTIMATES AND


ERRORS

! 22 acowtancy.com
3.1 ACCA SYLLABUS GUIDE OUTCOME 1:-
Distinguish between changes in accounting policies and changes in accounting
estimates and describe how accounting standards apply the principle of
comparability where an entity changes its accounting policies.
Recognise and account for changes in accounting policies.

3.1.1 Accounting Policies

Definition

Accounting policies are:

the specific principles, bases, conventions, rules and practices applied by an entity in
preparing and presenting the financial statements

An entity should follow accounting standards when deciding its accounting policies. If
there is no guidance in the standards, management should use the policy which gives
the most relevant and reliable information as outlined in the Framework for the
Preparation and Presentation of Financial Statements.

Changes in accounting policies are

only made if:

• It is required by a Standard or Interpretation; or

• It would give more relevant and reliable information

Accounting treatment

• Adjust the opening balance of the item affected in equity (e.g. retained earnings)
– put this in the statement of changes in equity also (second line)

• Adjust the comparative amounts for the affected item as if the policy had always
been applied

Disclosures Required

! 23 acowtancy.com
• The nature of the change in accounting policy

• The reasons for the change

• The amount of the adjustment

3.1.2 Changes in Accounting Estimates

Definition
A change in an accounting estimate is an adjustment of the carrying amount of an asset
or liability, or related expense, resulting from reassessing the expected future benefits
and obligations associated with that asset or liability.

Examples

• Allowances for doubtful debts;

• Inventory obsolescence;

• Useful economic life of property, plant and equipment

Accounting Treatment

Simply change the current period. No change to comparatives

3.2 ACCA SYLLABUS GUIDE OUTCOME 2:-


Recognise and account for the correction of prior period errors.

3.2.1 Prior Period Errors

Definition

Prior period errors are omissions from, and misstatements in, an entity's financial
statements for one or more prior periods arising from a failure to use, or misuse of,

! 24 acowtancy.com
reliable information that was available and could reasonably be expected to have been
obtained and taken into account in preparing those statements.

Examples

• mathematical mistakes,
• mistakes in applying accounting policies,
• oversights or misinterpretations of facts, and
• fraud.

Accounting treatment

• adjust the opening balance of the item affected in equity (eg retained earnings) –
put this in the statement of changes in equity also (second line)
• restate the comparative amounts for the prior period(s) presented in which the
error occurred; or
• if the error occurred before the earliest prior period presented, restating the
opening balances of assets, liabilities and equity for the earliest prior period
presented.
Disclosures Required

• The nature of the prior period error


• for each prior period presented, to the extent practicable, the amount of the
correction
• the amount of the correction at the beginning of the earliest prior period
presented
• if retrospective restatement is impracticable, an explanation and description of
how the error has been corrected

Lecture Example 1

Emerald has had a policy of writing off development expenditure to the income
statement21 as it was incurred.

In preparing its financial statements for the year ended 30 September 2007, it has
become aware that, under IFRS rules, qualifying development expenditure should be

21 now referred to as statement of profit or loss and other comprehensive income

! 25 acowtancy.com
treated as an intangible asset. Below is the qualifying development expenditure for
Emerald:

$’000

Year ended 30 September 2004 300

Year ended 30 September 2005 240

Year ended 30 September 2006 800

Year ended 30 September 2007 400

All capitalised development expenditure is deemed to have a four year life. Assume
amortisation commences at the beginning of the accounting period following
capitalisation. Emerald had no development expenditure before that for the year ended
30 September 2004.

Required:-

Calculate the amounts which should appear in the Statement of Profit or Loss
and SFP (including comparative figures), and SOCIE for the year ended 30
September 2007.

ACCA F7 Exam December 2007 Question 5(b)

3.3 ACCA SYLLABUS GUIDE OUTCOME 3:-


Explain the following measures and compute amounts using:
o historic cost
o fair value
o current cost
o net realisable value
o present value of future cash flows

3.3.1 Historic Cost

! 26 acowtancy.com
The amount paid or fair value of the consideration given.

3.3.2 Fair Value

IFRS 1322 defines fair value as “the price that would be received to sell an asset or paid
to transfer a liability in an orderly transaction between market participants at the
measurement date”. IFRS 13 is explained in more detail at the end of this section.

3.3.3 Current Cost

The amount that would have to be paid if the same or an equivalent asset was acquired
currently.

3.3.4 Net realisable value

The amount that could currently be obtained by selling the asset, net of the estimated
selling and completion costs.

3.3.5 Present value of future cash flows

The present discounted value of the future net cash inflows that the item is expected to
generate.

Illustration 1

A company owns a machine which was purchased last year for $280,000. Depreciation
is provided at 25% straight line.

It is estimated that this machine could be sold second hand for $88,000 although the
company would have to spend about $500 in advertising costs to do so.

If replaced, the machine would cost $360,000, although this current model is 20% more
efficient.

The machine is expected to generate net cash inflows of $40,000 for the next 5 years
after which time it will be scrapped.

The company’s cost of borrowing is 6%.

The discount factors at 6% at the end of:

22 IFRS 13, Fair Value Measurement

! 27 acowtancy.com
Year 1 0.943
Year 2 0.890
Year 3 0.840
Year 4 0.792
Year 5 0.747

What is the value of the following asset using:-

a. Historical Cost
b. Fair Value
c. Current Cost
d. Net Realisable Value
e. Present Value of Future Cashflows

Solution

Historical Cost

Cost – Accumulated Depreciation

280,000 – 70,000 = $210,000

(Accumulated depn = 0.25 x 280,000)

Fair Value

$88,000

Current cost

360,000 x 100/120 = 300,000 – 75,000(depn) = $225,000

Net realisable Value

88,000 – 500 = $87,500

! 28 acowtancy.com
Present Value of Future Cashflows

40,000 x 4.212 = $168,480

(0.943 + 0.890 + 0.840 + 0.792 + 0.747 = 4.212)

Lecture Example 2

Update has been considering the effect of alternative methods of preparing their
financial statements. As an example, they picked an item of plant that they acquired
from Suppliers on 1 April 2000 at a cost of $250,000.

The following details have been obtained:


▪ the company policy is to depreciate plant at 20% per annum on the reducing
balance basis.
▪ suppliers’ price catalogue at 31 March 2003 shows an item of similar plant at a
cost of $320,000. On reading the specification, it appears that the new model can
produce 480 units per hour whereas the model owned by Update can only
produce 420 units per hour.

Required:-

Calculate for Update the depreciation charge for the plant for the year to 31 March
2003 (based on year end values) and its carrying value on that date using:

i. the historical cost basis; and


ii.a current cost basis.

(ACCA Exam Paper 2.5 June 2003 Qs 5 a(ii) revised)

Lecture Example 323

Which of the following criticisms does NOT apply to historical cost accounts during a
period of rising prices?

A. They contain mixed values; some items are at current values, some at out of
date values

23 Specimen Exam Applicable from December 2014

! 29 acowtancy.com
B. They are difficult to verify as transactions could have happened many years ago
C. They understate assets and overstate profit
D. They overstate gearing in the statement of financial position

Fair Value Measurement

IFRS 13 provides guidance on how to measure the fair value of financial and non-
financial assets and liabilities. Its aim is to reduce complexity and improve consistency
when measuring fair value.

When measuring fair value, an entity considers the characteristics of the asset or
liability, for example:
• the condition and location of an asset
• restrictions, if any, on the sale or use of an asset (that would transfer with the
asset).

This means that when revaluing its property, plant and equipment, an entity should
consider the highest and best use of the assets.

A fair value measurement assumes that the transaction to sell the asset or transfer the
liability takes place either in the: -

1. principal market – the market with greatest volume and level of activity for the
asset or liability
2. most advantageous market (in the absence of a principal market) – the market
that maximises the amount that would be received to sell the asset or minimises
the amount that would be paid to transfer the liability.

IFRS 13 describes three different valuation techniques that may be used to measure fair
value: -

1. Market approach – uses prices from market transactions involving identical or


similar assets or liabilities, e.g. quoted prices of listed equity, debt securities or
futures, or market interest rates
2. Income approach - this converts future cash flows to a single discounted
amount; e.g. discounted cash flow models and option pricing models

3. Cost approach – this reflects the amount required currently to replace the
service capacity of an asset, i.e. the current replacement cost.

When measuring fair value, an entity is required to maximise the use of relevant
observable inputs and minimise the use of unobservable inputs.

! 30 acowtancy.com
Level 1 Level 2 Level 3

Definition Quoted prices in Observable inputs Unobservable inputs


active markets for
identical assets or
liabilities
Example Quoted prices for an Current market rents Projected cash flows
equity security that for similar properties used to value a
trades on the Stock and market interest business or non-
Exchange rates or yields taken controlling interest in
to measure the fair an entity that is not
value of an publicly listed
investment property

How does all this work in practice?

E.g. an entity owns 10,000 ordinary shares in M & S. Since there is an active market for
these shares through the London stock exchange, the entity must use a market
approach (level 1 input).

The same principle applies to a listed debt security that is quoted in an active market.
However, the measurement of the fair value of an unlisted debt security may require the
use of an income approach, e.g. a discounted cash flow model using market interest
rate for similar debt securities (level 2 input) and market credit spreads adjusted for
entity-specific credit risk (level 2 or 3 inputs).

3.4 ACCA SYLLABUS GUIDE OUTCOME 4:-


Describe the advantages and disadvantages of the use of historical cost
accounting.

Historical cost has been defined as the amount paid or fair value of the consideration
given.
Advantages of Historical Cost Accounting

1. the cost is known and can be proved (e.g. against an invoice). It is therefore
objective

2. it enhances comparability

! 31 acowtancy.com
3. it leads to stable pricing – using current market values would lead to volatility in
asset values

Disadvantages of Historical Cost Accounting

1. non-current asset values are unrealistic

2. since non-current asset values are low, depreciation is low and does not fully
reflect the value of the asset consumed during the accounting year

3. lower costs, e.g. depreciation expense, would lead to higher profits. There is a
possibility that this may lead to higher taxation, wage demands and dividend
expectation (based on overstated earnings per share). The combination of these
effects is that a company may overspend or over distribute its profits and not
maintain its capital base.

4. comparisons over time are unrealistic

5. Understatement of asset values tends to overstate gearing, and leads to a low


asset per share value and can make the company vulnerable to a take over

6. Where assets, particularly land and buildings, are being used as security to raise
finance, it is current value that lenders are interested in, not historical values

These disadvantages usually arise in times of rising prices. In fact, in times of rising
prices, historical cost accounting tends to understate asset values and overstate profits.

! 32 acowtancy.com
3.5 ACCA SYLLABUS GUIDE OUTCOME 5:-
Discuss whether the use of current value accounting overcomes the problems of
historical cost accounting.

Current cost accounting attempts to provide more realistic book values by valuing
assets at current replacement cost, rather than the amount actually paid for them. This
contrasts with the usual historical cost approach. In fact, the current cost is usually
calculated by adjusting the historical cost for inflation.

The current operating profit is considered to be more relevant to many decisions such
as dividend distribution, employee wage claims and even as a basis for taxation.

The problems that current cost accounting (and other approaches to accounting for
inflation) attempt to solve are obviously linked to inflation. In practical terms, it can be
very difficult to determine the current value of assets. It is often subjective and complex.

3.6 ACCA SYLLABUS GUIDE OUTCOME 6:-


Describe the concept of financial and physical capital maintenance and how this
affects the determination of profits.

The concept of capital maintenance is concerned with how an entity defines the capital
that it seeks to maintain. It provides the linkage between the concepts of capital and the
concepts of profit. Profit is the residual amount that remains after expenses (including
capital maintenance adjustments, where appropriate) have been deducted from income.
If expenses exceed income the residual amount is a loss.

Financial capital maintenance

Under this concept a profit is earned only if the financial (or money) amount of the net
assets at the end of the period exceeds the financial (or money) amount of net assets at
the beginning of the period, after excluding any distributions to, and contributions from,
owners during the period. Financial capital maintenance can be measured in either
nominal monetary units or units of constant purchasing power.

Physical capital maintenance

Under this concept, a profit is earned only if the physical productive capacity (or
operating capability) of the entity (or the resources or funds needed to achieve that
capacity) at the end of the period exceeds the physical productive capacity at the
beginning of the period, after excluding any distributions to, and contributions from,
owners during the period.

! 33 acowtancy.com
Further Notes

Not-For-Profit and Public Sector Entities

Not-for-profit and public sector entities have different goals and objectives to profit-
making organisations. They operate in different environments and are responsible to
different stakeholders. They have to account for funds received and show how they
have been spent. They need to focus on generating cash flows but are not expected to
show a profit. E.g. include government departments and agencies, higher education
institutions, charitable bodies.

Phase G of the Conceptual Framework for Financial Reporting deals with the
Application to not-for-profit entities in the private and public sector. A set of International
Public Sector Accounting Standards (IPSASs) are being developed which closely match
the IASs and IFRSs. They are all based on the accrual concept although many not-for-
profit organisations still use cash accounting. However, some issues are debatable, e.g.
the definition of a liability in public entities: does the commitment to provide public
benefits meets the definition of a liability? Can the obligation be reliably measured when
we are aware that some projects cost much more that the amount actually budgeted?

Who are the users for not-for-profit entities?

The main users are the providers of funds, very often the taxpayers for government
departments. In the case of charities, it will be the different individuals/entities who are
financially supporting the charities.

How is performance measured?

The principle used is referred to a Value for Money (VFM) approach. It is made up of the
3 Es: -

1. effectiveness – the extent to which the organisation achieves its objectives


2. economy – the ability of the organisation to optimise the use of its productive
resources (keeping the cost of input resources as low as possible)
3. efficiency – the ‘output’ of the organisation per unit of resource consumed

Performance measurement is based on Key Performance Indicators (KPIs). E.g. % of


roads resurfaced, average spend on each student’s education, average waiting time for
an operation.

Best Value is the term used by the United Kingdom's (UK) national government for its
performance management initiative

! 34 acowtancy.com
1. Challenging why, how and by whom a service is provided
2. Comparing performance against other local authorities.
3. Consulting the local community, users, etc
4. Using fair competition to secure efficient and effective services.

June 2010 Qs 3 (part)

On a separate matter, you have been asked to advise on an application for a loan to
build an extension to a sports club which is a not-for-profit organisation. You have been
provided with the audited financial statements of the sports club for the last four years.

Required:

Identify and explain the ratios that you would calculate to assist in determining whether
you would advise that the loan should be granted. (5 marks)

Answer

1. To assess the gearing of the sports club: e.g. long-term loans compared to
capital employed (total assets less current liabilities)
2. To assess the repayment of loan + interest: e.g. excess of income over
expenditure is compared to interest payments – i.e. find the number of times this
excess covers interest; the higher the number of times, the lower the risk of
defaulting
3. Other factors to be considered: the value of assets to be used as security; any
‘one-off’ donations; future prospects of the sports club.

Please also refer to page 6 of the notes

Current Value Accounting (CVA) (pls refer first to notes pgs 25 to 31)

As described in the notes, the problems of historical cost accounting arise in periods of
inflation. In order to offset some of these disadvantages, entities are now permitted to
revalue non-current assets (e.g. land and buildings). Financial assets and liabilities are
carried at fair value (IFRS 13), i.e. “the price that would be received to sell an asset or
paid to transfer a liability in an orderly transaction between market participants at the
measurement date”.

! 35 acowtancy.com
Under current value accounting, the original cost of an asset, e.g. receivables and
payables, would be replaced with its discounted present value (the present value of its
future cash flows). For non-current assets such as motor vehicles and machinery, the
current cost can be its replacement cost; for inventories, it will be either the current
replacement cost or net realisable value.

Two techniques which address the disadvantages of historical cost accounting are: -
1. Current purchasing power (CPP)
2. Current cost accounting (CCA)
But before we look at these two techniques, how is profit measured? Profit can be
measured as the difference between how wealthy a company is at the beginning and
end of an accounting period.

Financial capital maintenance – profit is the increase in nominal money capital over the
period. This is used in CPP and historical cost.

Physical capital maintenance – profit is the increase in the physical productive capacity
over the period. This is used in CCA.

Current Purchasing Power (CPP)

CPP accounts for general inflation. The profit for the year in CPP terms is found by
converting sales, inventory, purchases and other expenses into year-end units of $CPP.
In addition, a profit on holding net monetary liabilities, e.g. payables, or a loss on
holding monetary assets, e.g. cash and receivables, is calculated.

E.g.

1.1.2011 – purchased 1000 units for $5 each

30.6.2011 – sold these 1000 units for $7 each

30.11.2011 – purchased 500 units for $6.50 each

The general price index is: -

1.1.2011 – 100
30.6.2011 – 120
30.11.2011 - 140
31.12.2011 – 150

Under historical cost accounting, gross profit is


Sales 7000

! 36 acowtancy.com
COS 5000
Profit 2000

Under CPP,

Sales would be (7000/120 x 150) = 8750


COS would be (5000/100 x 150) = 7500

There is also a loss of holding the cash for 6 months between July and December –
8750 – 7000 = 1750
There is a gain for owing creditors for one month
500 x 6.50 = 3250/140 x 150 = 3482 – 3250 = 232

Advantages of CPP

1. CPP method adopts the same unit of measurement by taking into account the price
changes.

2. Under CPP method, historical accounts continue to be maintained. CPP statements
are prepared on supplementary basis.
3. CPP method facilitates the calculation of gain or loss in purchasing power due to the
holding of monetary items (cash, receivables and payables).
4. CPP method uses common purchasing power as measuring unit. So, the
comparative study is easy.
5. CPP method provides reliable financial information for taking management decision
to formulate plans and policies.
6. CPP method ensures keeping intact the purchasing power of capital contributed by
shareholders. So, this method is of great importance from the point of view of the
shareholders.

Disadvantages of CPP

1. CPP method considers only the changes in general purchasing power. It does not
consider the changes in the value of individual items.
2. CPP method is based on statistical index number which cannot be used in an
individual firm.
3. It is very difficult to choose a suitable price index.
4. CPP method fails to remove all the defects of historical cost accounting system.

Current Cost Accounting

CCA is based on the physical concept of capital maintenance. It makes adjustments to


allow for specific price movements. The value of assets consumed or sold and the value
of assets in the SFP should be stated at their deprival value, i.e. their value to the

! 37 acowtancy.com
business. Deprival value can be either the replacement cost, net realisable value or
value in use (economic value).
Example: -

Item A
Cost $10
Selling Price $15
Replacement cost $12

The current cost profit, when the item is sold, is: -


Sales 15
RC 12
Profit 3

The difference of $2 ($12 - $10) is known as a holding gain. This holding gain is
excluded from profit.

Advantages of CCA

1. By excluding the holding gain from profit, CCA indicates whether the physical
capital maintenance has been maintained. Did the dividends paid to
shareholders reduce the operating capability of the entity?
2. It is more relevant to users and provides up-to-date information
3. The value is arrived at after considering the opportunity cost of holding the assets
and the expected benefits from their future use

Disadvantages of CCA

1. It is not objective – subjective judgement has to be used


2. It may be difficult to estimate replacement cost
3. Using the deprival value concept may not be meaningful. Some assets will be
valued at replacement cost, but others will be valued at NRV or value in use.

! 38 acowtancy.com
Further Questions

Question 124

Which of the following would be a change in accounting policy in accordance with IAS 8
Accounting Policies, Changes in Accounting Estimates and Errors?

A. Adjusting the financial statements of a subsidiary prior to consolidation as its


accounting policies differ from those of its parent
B. A change in reporting depreciation charges as cost of sales rather than as
administrative expenses
C. Depreciation charged on reducing balance method rather than straight line
D. Reducing the value of inventory from cost to net realisable value due to a valid
adjusting event after the reporting period

Question 2

If the current cost measurement basis is used, assets are measured at:

A. Replacement cost
B. The amount paid to acquire them
C. The amount which could be obtained by selling them
D. Present value

Question 3

In times of falling prices, the historical cost convention

A. Understates asset values and profits


B. Understates asset values and overstates profits
C. Overstates asset values and profits
D. Overstates asset values and understates profits

24 Specimen Exam Applicable from December 2014

! 39 acowtancy.com
Question 4

Which of the following are advantages of historical cost accounting?


(1) It maintains financial and physical capital
(2) The statement of financial position shows the value of the business.
(3) Reported amounts are objectively verifiable.
(4) The profit concept is well understood.

A. 3 and 4
B. 1 and 2
C. 1 and 3
D. 2 and 4

Question 5

Gironimo Plc makes two changes to accounting practice at the end of 20X7:
(1) It changes the way in which it depreciates motor vehicles from 20% straight line to
25% reducing balance.
(2) It starts to capitalise interest costs where allowed in accordance with the relevant
standard. Previously it had adopted a policy of writing off all interest costs to the
statement of comprehensive income.

What is the correct way to account for these two changes?

1 2
A. Do not adjust opening reserves Do not adjust opening reserves
B. Do not adjust opening reserves Adjust opening reserves
C. Adjust opening reserves Adjust opening reserves
D. Adjust opening reserves Do not adjust opening reserves

! 40 acowtancy.com
Question 6

ABC Co was set up on 1 January 2001. In the first 3 years of operation, it recognised
development expenditure as an intangible asset in its accounts. During 2003, the
managers of the company decided that all development expenditure should be written
off as incurred.

The following information is available: -

Development Exp Development Exp


Incurred and Capitalised Amortised to P&L
$’000 $’000

2001 500 -
2002 800 200
2003 1000 400

The 2003 accounts showed the following movements in retained earnings: -

Opening balance 1.1.2003 $9,000,000


Profits for the year $4,000,000
Closing balance 31.12.2003 $13,000,000

Profits for the year 2004 were $6,000,000 after charging the actual development
expenditure to the income statement.

Fill in the missing information in the following SOCIE extract for retained earnings:

Retained Earnings

2004 2003
’000 ’000
Balance b/fwd 13,000 9,000
Retrospective change
in accounting policy (b) (a)
Restated balance (d) (c)
Profit for the year (f) (e)
Balance c/fwd h g

! 41 acowtancy.com
CHAPTER 4:

IAS 16 –
Property, Plant and Equipment (PPE)

4.1 ACCA SYLLABUS GUIDE OUTCOME 1:-


Define and compute the initial measurement of a non-current (including a self-
constructed) asset.

IAS 16 defines property, plant and equipment (PPE) as tangible items that are:
• held for use in the production or supply of goods or services, for rental to others,
or for administrative purposes and
• expected to be used during more than one accounting period.

An item of PPE should be recognized as an asset if and only if it is probable that future
economic benefits associated with the asset will flow to the entity and the cost of the
item can be measured reliably.

IAS 16 requires that PPE should initially be measured at cost.

Cost includes all costs necessary to bring the asset to working condition for its
intended use.

Examples:

• Original purchase price


• Costs of site preparation
• Delivery and handling
• Installation
• Related professional fees for architects and engineers
• Estimated cost of dismantling and removing the asset and restoring the site

Where these costs are incurred over a period of time (such as employee benefits), the
period for which the costs can be included in the cost of PPE ends when the asset is
ready for use, even if the asset is not brought into use until a later date.

! 42 acowtancy.com
4.2 ACCA SYLLABUS GUIDE OUTCOME 2:-
Identify subsequent expenditure that may be capitalised (including borrowing
costs), distinguishing between capital and revenue items25.

4.2.1 IAS 23: Borrowing Costs26

IAS 23 requires that, where borrowing costs are directly incurred on a ‘qualifying asset’,
they must be capitalised as part of the cost of that asset.

A qualifying asset may be a tangible or an intangible asset that takes a substantial


period of time to get ready for its intended use or eventual sale. Property construction
would be a typical example, but it can also be applied to intangible assets during their
development period.

Borrowing costs include interest based on its effective rate (which incorporates the
amortisation of discounts, premiums and certain expenses) on overdrafts, loans and
(some) other financial instruments and finance charges on finance leased assets.

If the entity has borrowed funds specifically to finance the construction of an asset, then
the amount to be capitalised is the actual finance costs incurred. Where the borrowings
form part of the general borrowing of the entity, then a capitalisation rate that represents
the weighted average borrowing rate of the entity should be used.

Capitalisation should commence when expenditure is being incurred on the asset,


which is not necessarily from the date funds are borrowed. Capitalisation should cease
when the asset is ready for its intended use, even though the funds may still be
incurring borrowing costs. Also capitalisation should be suspended if there is a
suspension of active development of the asset.

Any borrowing costs that are not eligible for capitalisation must be expensed. Borrowing
costs cannot be capitalised for assets measured at fair value.

Illustration 1

On 1.1.2012, a company borrowed $1m at a rate of 10% p.a. to finance the building of a
factory, which is expected to take one year to build. All the $1m was drawn down on
1.1.2012 but only $500,000 of this $1m was used immediately on 1.1.2012. The

25 ACCA F7 Exam Dec 08 Q5


26 ACCA F7 Exam June 2010

! 43 acowtancy.com
remaining $500,000 was utilised on 1.7.2012. As a result, these $500,000 had been
invested temporarily at a rate of 8% p.a.

Calculate the borrowing costs which may be capitalised and the cost of the asset
as at 31.12.2012.


Borrowing Costs

1m x 10% = $100,000


Investment income

500,000 x 8% x 6/12 = $20,000


Net borrowing costs = 100 - 20 = $80,000


Cost of Asset:

Total expenditure $1m + borrowing costs $80,000 = $1080,000

4.2.2 Revenue Items

Revenue items refer to the normal short term day by day costs of operating the
business – buying materials/products, paying wages, and all the normal overheads like
rent, stationery, fuel bills.

These are charged to the Statement of Profit or Loss and Other Comprehensive Income
each month as they are used up.

4.2.3 Capital Items

Capital items refer mainly to assets bought to use long term in the business – desks,
computers, productive equipment and machinery, vehicles, shop-fitting costs, and so
on27.

Lecture Example 1

On 1 October 20X6, Omega began the construction of a new factory. Costs relating to
the factory, incurred in the year ended 30 September 20X7, are as follows:

$000
Purchase of the land 10,000
Costs of dismantling existing structures on the site 500

27 ACCA F7 Exam Dec 07 Q2; Jun 09 Q5

! 44 acowtancy.com
Purchase of materials to construct the factory 6,000
Employment costs (Note 1) 1,800
Production overheads directly related to the construction (Note 2) 1,200
Allocated general administrative overheads 600
Architects’ and consultants’ fees directly related to the construction 400
Costs of relocating staff who are to work at the new factory 300
Costs relating to the formal opening of the factory 200
Interest on loan to partly finance the construction
of the factory (Note 3) 1,200

Note 1
The factory was constructed in the eight months ended 31 May 20X7. It was brought
into use on 30 June 20X7. The employment costs are for the nine months to 30 June
20X7.

Note 2
The production overheads were incurred in the eight months ended 31 May 20X7. They
included an abnormal cost of $200,000, caused by the need to rectify damage resulting
from a gas leak.

Note 3
Omega received the loan of $12m on 1 October 20X6. The loan carries a rate of interest
of 10% per annum.

Note 4
The factory has an expected useful economic life of 20 years. At that time the factory
will be demolished and the site returned to its original condition. This is a legal
obligation that arose on signing the contract to purchase the land. The expected costs
of fulfilling this obligation are $2m. An appropriate annual discount rate is 8%.

Requirement
Compute the initial carrying value of the factory.28

28 Article
“Property, Plant and Equipment”, P Robins, Student Accountant, June/July 2007
http://www.accaglobal.com/content/dam/acca/global/pdf/sa_aug07_robins.pdf

! 45 acowtancy.com
Lecture Example 229

Which of the following items should be capitalised within the initial carrying amount of
an item of plant?

i) Cost of transporting the plant to the factory


ii) Cost of installing a new power supply required to operate the plant
iii) A deduction to reflect the estimated realisable value
iv) Cost of a three-year maintenance agreement
v) Cost of a three-week training course for staff to operate the plant

A. (i) and (ii) only


B. (i), (ii) and (iii)
C. (ii), (iii) and (iv)
D. (i), (iv) and (v)

4.3 ACCA SYLLABUS GUIDE OUTCOME 3:-


Discuss the requirements of relevant accounting standards in relation to the
revaluation of non-current assets30.

After initial recognition, the asset should be measured using either the cost model or the
revaluation model.

The cost model requires an asset, after initial recognition, to be carried at cost less
accumulated depreciation and impairment losses.

The revaluation model requires an asset, after initial recognition, to be measured at a


revalued amount, which is its fair value less subsequent depreciation and impairment
losses.

A revaluation surplus (gain) should be credited to a revaluation surplus (reserve)


whereas a revaluation deficit (loss) should be expensed immediately (assuming, in both
cases, no previous revaluation of the asset has taken place). A surplus on one asset
cannot be used to offset a deficit on a different asset (even in the same class of asset).

29 Specimen Exam Applicable from December 2014

30 Examined June 2014 Qs 4a

! 46 acowtancy.com
4.3.1 How often should you revalue an asset?

Regularly, so that the carrying amount of an asset does not differ materially from its fair
value at the end of the reporting period. Where the carrying amount of the asset differs
significantly from its fair value, a (new) revaluation should be carried out. Even if there
are no significant changes, assets should still be subject to a revaluation every three to
five years.

4.3.2 What needs to be revalued?

If an item is revalued, the entire class of assets to which that asset belongs should be
revalued.

4.3.3 Do we depreciate revalued assets?

Yes, revalued assets are depreciated in the same way as under the cost model. The
asset should be depreciated based on its revalued amount (less any residual value)
over its estimated remaining useful life, which should be reviewed annually irrespective
of whether it has been revalued.

An entity may choose to transfer annually an amount of the revaluation surplus relating
to a revalued asset to retained earnings corresponding to the ‘excess’ depreciation
caused by an upwards revaluation. Alternatively, it may transfer all of the relevant
surplus at the time of the asset’s disposal. 31

4.4 ACCA SYLLABUS GUIDE OUTCOME 4:-


Compute depreciation based on the cost and revaluation models and on assets
that have two or more significant parts (complex assets).

IAS 16 defines depreciation as ‘the systematic allocation of the depreciable amount of


an asset over its useful life’.

4.4.1 How much depreciation? (June 2009 Q5)

The depreciable amount (cost less prior depreciation, impairment, and residual value)
should be allocated on a systematic basis over the asset’s useful life.

31 examined June 2014 Qs 4a

! 47 acowtancy.com
4.4.2 How often should we review the residual value?

The residual value and the useful life of an asset should be reviewed at least at each
financial year-end. If either changes significantly, then that change should be accounted
for over the remaining estimated useful economic life.

4.4.3 Which depreciation method should be used?

A number of methods can be used to allocate depreciation to specific accounting


periods. Two of the more common methods, specifically mentioned in IAS 16, are the
straight line method, and the reducing (or diminishing) balance method.

The depreciation method used should reflect the pattern in which the asset’s economic
benefits are consumed by the enterprise.

4.4.4 How often should we review of this method is still right?

The method of depreciation should be reviewed at least annually.

4.4.5 Where does depreciation go?

Depreciation should be charged to the Statement of Profit or Loss, unless it is included


in the carrying amount of another asset. An example of this practice would be the
possible inclusion of depreciation in the costs incurred on a construction contract that
are carried forward and matched against future income from the contract, under the
provisions of IFRS 15.

4.4.6 When do we start and finish depreciating an asset?

Depreciation begins when the asset is available for use and continues until the asset is
derecognised, either on disposal or when no future economic benefits are expected
from the asset (in other words, it is effectively scrapped). A gain or loss on disposal is
recognised as the difference between the disposal proceeds and the carrying value of
the asset (using the cost or revaluation model) at the date of disposal. This net gain is
included in the statement of profit or loss – the sales proceeds should not be recognised
as revenue.

4.4.7 Significant parts depreciated separately

If the cost model is used, each part of an item of property, plant, and equipment with a
significant cost (in relation to the total cost) must be depreciated separately.

! 48 acowtancy.com
4.4.8 Parts which are regularly replaced

The replacement cost is added to the asset cost when the recognition criteria
(mentioned above) are met.

The carrying amount of the replaced parts is derecognised.

4.4.9 Major Inspections for faults (e.g. aircraft)

The inspection cost is added to the asset cost when recognition criteria (above) are
met.

If necessary, the estimated cost of a future similar inspection may be used as an


indication of what the cost of the existing inspection component was when the item was
acquired or constructed.

Lecture Example 3

Freehold property – at cost 1 October 2000 $63,000,000


Plant and equipment – at cost $42,200,000
Accumulated depreciation 1 October 2008
Building $8,000,000
Plant and equipment $19,700,000

The freehold property has a land element of $ 13 million. The building element is being
depreciated on a straight – line basis.

Plant and equipment is depreciated at 40% per annum using the reducing balance
method.

Required:

Calculate the depreciation expenses for the building and plant and equipment for the
year ended 30 September 2009.

(ACCA F7 Dec 2009 QS 2 Part)

! 49 acowtancy.com
4.5 ACCA SYLLABUS GUIDE OUTCOME 5:-
Account for revaluation and disposal gains and losses for non-current assets.

4.5.1 Normal Rule

When an asset is revalued, any increase in the carrying amount should be credited to
equity under the heading “revaluation surplus”.

Accounting treatment:

(1) Adjust cost account to revalued amount.


(2) Remove accumulated depreciation charged on the asset to date.
(3) Put the balance to the revaluation reserve.

The required double-entry is:

Dr Non-current asset cost


Dr Accumulated Depreciation
Cr Revaluation Reserve

Lecture Example 432

A property was purchased on 1 January 20X0 for $2m (estimated depreciable amount
$1m – useful economic life 50 years). Annual depreciation of $20,000 was charged from
20X0 to 20X4 inclusive and on 1 January 20X5 the carrying value of the property was
$1.9m. The property was revalued to $2.8m on 1 January 20X5 (estimated depreciable
amount $1.35m – the estimated useful economic life was unchanged).

Show the treatment of the revaluation surplus and compute the revised annual
depreciation charge.

4.5.2 Revaluation downwards

Any reduction in value arising from a revaluation should:-

First:- be debited to any revaluation surplus relating to the same asset


Then:- treat the remainder as an expense in the statement of profit or loss

32 Article,
“Property, Plant and Equipment and Tangible Assets”, by P. Robins, Student Accountant, August 2007
http://www.accaglobal.com/content/dam/acca/global/pdf/sa_aug07_robins.pdf

! 50 acowtancy.com
Lecture Example 533

The property referred to in Lecture Example 4 was revalued on 31 December 20X6. Its
fair value had fallen to $1.5m.

Compute the revaluation loss and state how it should be treated in the financial
statements.

4.5.3 What happens to the revaluation surplus on disposal of the asset


revalued?

Where assets are measured using the revaluation model, any remaining balance in the
revaluation surplus relating to the asset disposed of, is transferred directly to retained
earnings. No recycling of this balance into the statement of profit or loss is permitted.

4.5.4 Accounting for disposals of non-current assets

When a non-current asset is sold, there is likely to be a profit or loss on disposal. This
is the difference between the net sale price of the asset and its net book value at the
time of disposal.

If:

Sales proceeds > NBV → profit on disposal


Sales proceeds < NBV → loss on disposal

Accounting Treatment:-

(1) Remove the cost of the asset:


Dr Disposal account
Cr Non-current asset

(2) Remove the accumulated depreciation charged to date:


Dr Accumulated depreciation
Cr Disposal account

(3) Account for the sales proceeds:


Dr Cash
Cr Disposal account

(4) Balance off disposal account to find the profit or loss on disposal.

33 Article,
“Property, Plant and Equipment and Tangible Assets”, by P. Robins, Student Accountant, August 2007
http://www.accaglobal.com/content/dam/acca/global/pdf/sa_aug07_robins.pdf

! 51 acowtancy.com
A profit on disposal is shown in the statement of profit or loss as sundry income, a loss
as an expense in the statement of profit or loss.

4.6 Change in the useful life of an asset

The useful life of an item of property, plant and equipment should be reviewed at least
every financial year-end and, if expectations are significantly different from previous
estimates, the depreciation charge for current and future periods should be revised.

This is achieved by writing the net book value off over the asset's revised remaining
useful life.

NBV – residual value


Revised useful life

4.7 Change in method of depreciation

It is up to the business to decide which method of depreciation to apply to its non-


current assets. The chosen method of depreciation should be applied consistently from
year to year. This is an instance of the fundamental accounting assumption of
consistency.

The depreciation method has to be reviewed. If there are any changes in the expected
pattern of use of the asset, then the method used should be changed. In such cases,
the remaining net book value is depreciated under the new method, i.e. only current and
future periods are affected. The change is prospective.

Further Questions 134

Question 1

On 1 March 2008 Yucca acquired a machine from Plant under the following terms:
$
List price of machine 82,000

34 Article:“Accounting for Property, Plant and Equipment”, B.A. Retallack, Student Accountant
2010, http://www.accaglobal.com/content/dam/acca/global/PDF-students/2012/
sa_sept10_ias16.pdf

! 52 acowtancy.com
Import duty 1,500
Delivery fees 2,050
Electrical installation costs 9,500
Pre-production testing 4,900
Purchase of a five-year maintenance contract with Plant 7,000

In addition to the above information Yucca was granted a trade discount of 10% on the
initial list price of the asset and a settlement discount of 5% if payment for the machine
was received within one month of purchase. Yucca paid for the plant on 25 March 2008.

How should the above information be accounted for in the financial statements?

Question 2

Construction of Deb and Ham’s new store began on 1 April 2009. The following costs
were incurred on the construction:

$000
Freehold land 4,500
Architect fees 620
Site preparation 1,650
Materials 7,800
Direct labour costs 11,200
Legal fees 2,400
General overheads 940
The store was completed on 1 January 2010 and brought into use following its grand
opening on the 1 April 2010. Deb and Ham issued a $25m unsecured loan on 1 April
2009 to aid construction of the new store (which meets the definition of a qualifying
asset per IAS 23). The loan carried an interest rate of 8% per annum and is repayable
on 1 April 2012.

Required
Calculate the amount to be included as property, plant and equipment in respect of the
new store and state what impact the above information would have on the statement of
profit or loss (if any) for the year ended 31 March 2010.

Question 3

On 1 March 2010 Yucca purchased an upgrade package from Plant at a cost of $18,000
for the machine it originally purchased in 2008 (Question 1). The upgrade took a total
of two days where new components were added to the machine. Yucca agreed to
purchase the package as the new components would lead to a reduction in production

! 53 acowtancy.com
time per unit of 15%. This will enable Yucca to increase production without the need to
purchase a new machine.

Should the additional expenditure be capitalised or expensed?

Question 4

An item of plant was purchased on 1 April 2008 for $200,000 and is being depreciated
at 25% on a reducing balance basis.

Prepare the extracts of the financial statements for the year ended 31 March 2010.

Question 5

A machine was purchased on 1 April 2007 for $120,000. It was estimated that the asset
had a residual value of $20,000 and a useful economic life of 10 years at this date.

On 1 April 2009 (two years later) the residual value was reassessed as being only
$15,000 and the useful economic life remaining was considered to be only five years.

How should the asset be accounted for in the years ending 31 March 2008/2009/2010?

Question 6

A company purchased a property with an overall cost of $100m on 1 April 2009.

The property elements are made up as follows:

$000 Estimated life


Land and buildings
(Land element $20,000) 65,000 50 years
Fixtures and fittings 24,000 10 years
Lifts 11,000 20 years
100,000

Calculate the annual depreciation charge for the property for the year ended 31 March
2010.

! 54 acowtancy.com
Question 7

A company purchased a building on 1 April 2007 for $100,000. The asset had a useful
economic life at that date of 40 years. On 1 April 2009 the company revalued the
building to its current fair value of $120,000.

What is the double entry to record the revaluation?

Question 8

The carrying value of Zen’s property at the end of the year amounted to $108,000. On
this date the property was revalued and was deemed to have a fair value of $95,000.
The balance on the revaluation reserve relating to the original gain of the property was
$10,000.

What is the double entry to record the revaluation?

Question 9

A company revalued its property on 1 April 2009 to $20m ($8m for the land). The
property originally cost $10m ($2m for the land) 10 years ago. The original useful
economic life of 40 years is unchanged. The company’s policy is to make a transfer to
realised profits in respect of excess depreciation.

How will the property be accounted for in the year ended 31 March 2010?

Question 10

A company purchased a building on 1 April 2005 for $100,000 at which point it was
considered to have a useful economic life of 40 years. At the year end 31 March 2010
the company decided to revalue the building to its current value of $98,000.

How will the building be accounted for in the year ended 31 March 2010?

Question 11

At 1 April 2009 HD Ltd carried its office block in its financial statements at its original
cost of $2 million less depreciation of $400,000 (based on its original life of 50 years).
HD Ltd decided to revalue the office block on 1 October 2009 to its current value of
$2.2m. The useful economic life remaining was reassessed at the time of valuation and
is considered to be 40 years at this date. It is the company’s policy to charge
depreciation proportionally.

! 55 acowtancy.com
How will the office block be accounted for in the year ended 31 March 2010?
Question 12

An asset that originally cost $16,000 and had accumulated depreciation on it of $8,000
was disposed of during the year for $5,000 cash.

How should the disposal be accounted for in the financial statements?

Further Questions 2

Question 1

The following information is available for the year ended 31 October 2012:
$
Property
Cost as at 1 November 2011 102,000
Accumulated depreciation as at 1 November 2011 (20,400)
Carrying Value 81,600

On 1 November 2011, the company revalued the property to $150,000.

The company’s policy is to charge depreciation on a straight-line basis over 50 years.


On revaluation there was no change to the overall useful economic life. It has also
chosen not to make an annual transfer of the excess depreciation on revaluation
between the revaluation reserve and retained earnings.

What should be the balance on the revaluation reserve and the depreciation charge as
shown in the financial statements for the year ended 31 October 2012?

Depreciation charge Revaluation reserve


$ $
A. 3,750 68,400
B. 3,750 48,000
C. 3,000 68,400
D. 3,000 48,000

! 56 acowtancy.com
CHAPTER 5:

IAS 20 – GOVERNMENT GRANTS

5.1 ACCA SYLLABUS GUIDE OUTCOME 1:-


Apply the provisions of relevant accounting standards in relation to accounting for
government grants.

IAS 20 applies to all government grants and other forms of assistance by a government
aimed at providing an economic benefit to an entity or group of entities qualifying under
certain criteria. This government assistance can be of many types, including grants,
forgivable loans, and indirect or non-monetary form of assistance, such as technical
advice.

5.1.1 When can government grants be brought into the accounts?

Government grants should only be recognized when there is reasonable assurance


that:
• the enterprise will comply with any conditions attached to the grant and
• the grant will be received.

5.1.2 Where does it go in the accounts?

The grant is recognised as income over the period necessary to match them with the
related costs, for which they are intended to compensate, on a systematic basis.

5.1.3 Capital Grants

Non-monetary grants, such as a plot of land or a building in a remote area, (or money
towards the purchase of non-current assets) are usually accounted for at fair value.
They should be presented in the SFP in either of two ways:-

• as deferred income (a liability) and transfer a portion to revenue each year, or


• by deducting the grant in arriving at the asset’s carrying amount. Depreciate the
asset on the reduced cost.

! 57 acowtancy.com
When the grant is treated as deferred income, it is released to the statement of profit or
loss over the useful economic life of the asset.

5.1.4 Revenue Grants e.g. a grant for wages incurred

A grant relating to income may be reported separately as ‘other income’ (credit in the
statement of profit or loss) or deducted from the related expense.

5.1.5 Repayment of government grants

When a government grant becomes repayable (e.g. due to non-fulfillment of the terms
of the grant), it should be treated as a change in estimate under IAS 8 and accounted
for prospectively.

Where the original grant related to income, the repayment should be dealt with as an
expense.

Where the original grant related to an asset, the repayment should be treated as
increasing the carrying amount of the asset or reducing the deferred income balance.

The cumulative depreciation which would have been charged had the grant not been
received should be charged as an expense.

Lecture Example 1

The following is an extract of Errsea’s balances of property, plant and equipment and
related government grants at 1 April 2006.

Accumulated Carrying
Cost Depreciation Amount
$’000 $’000 $’000

Property, plant and equipment 240 180 60

Non-current liabilities
Government grants 30

Current liabilities
Government grants 10

Details including purchases and disposals of plant and related government grants
during the year are:

! 58 acowtancy.com
(i) Included in the above figures is an item of plant that was disposed of on 1 April
2006 for $12,000 which had cost $90,000 on 1 April 2003. The plant was being
depreciated on a straight-line basis over four years assuming a residual value of
$10,000. A government grant was received on its purchase and was being
recognised in the statement of profit or loss in equal amounts over four years. In
accordance with the terms of the grant, Errsea repaid $3,000 of the grant on the
disposal of the related plant.

(ii) An item of plant was acquired on 1 July 2006 with the following costs:

$
Base cost 192,000
Modifications specified by Errsea 12,000
Transport and installation 6,000

The plant qualified for a government grant of 25% of the base cost of the plant,
but it had not been received by 31 March 2007. The plant is to be depreciated on
a straight-line basis over three years with a nil estimated residual value.

(iii) All other plant is depreciated by 15% per annum on cost.

(iv) $11,000 of the $30,000 non-current liability for government grants at 1 April 2006
should be reclassified as a current liability as at 31 March 2007.

(v) Depreciation is calculated on a time apportioned basis.

Required:-

Prepare extracts of Errsea’s statement of profit or loss and statement of financial


position in respect of the property, plant and equipment and government grants
for the year ended 31 March 2007.

Note: Disclosure notes are not required.

(10 marks)

(ACCA Paper 2.5 June 2007 Qs 5a)

! 59 acowtancy.com
Further Questions

Please indicate which you think is closest to the correct answer.

Question 1

Government assistance includes:


A. Direct action to provide economic benefits to qualifying firms.
B. Imposing import tariffs.
C. Indirect help, such as improving local infrastructure.

Question 2

IAS 20 deals with:


A. Disclosure of government grants.
B. Government participation in the ownership of firms.
C. Tax benefits provided to a firm in relation to Government Grants.

Question 3

The impact of government assistance on financial statements is which of the


following:
A. Financial Statements must be able to reflect the receipt of government
assistance.
B. Financial statements must ignore all assistance.
C. Financial statements must show only 10% of total assistance.

Question 4

Which of the following may be categorized as purposes of government


assistance? Tick all that apply.

1. Reduce unemployment by subsidizing jobs and training.


2. Boosting capital by investing in specified assets.
3. Try to promote economic activity in specific regions

A. 1 and 2
B. 1,2 and 3
C. 2 and 3

! 60 acowtancy.com
Question 5

If a firm does not comply with the conditions of a government loan, then this may result
in the need:

1. To repay the loan.


2. To account for the loan on a cash basis only.
3. To record a contingent liability in the future.

A. 1,2 and 3
B. 1 and 2
C. 2 and 3

Question 6

A qualifying firm, it may receive grants related to the assets from the government, when
it:
A. Buys, builds, or acquires long-term assets.
B. Buys long-term assets.
C. Acquires long-term assets.
D. Builds long-term assets.

Question 7

Government grants are defined as:


A. Transactions with the government in the normal course of trade.
B. A transfer of resources to qualifying firms.
C. Provision of guarantees by the government.

Question 8

If the grants are intended to compensate certain costs, then they should be:
A. Recognised as income over the periods when the related costs are incurred.
B. Only entered in the books when those costs are incurred.
C. Ignored.

! 61 acowtancy.com
Question 9

If grants relate to depreciable assets:

A. They should not be recognised at all since the asset will have no value
eventually.
B. Recognised as income in the periods in which the depreciation is charged.
C. Credited immediately to other income.

Question 10

Government grants may be given in more than one form. They may be given as:

1. Forgivable loans.
2. Grants related to income.
3. Grants related to assets.

A. 2 and 3 only
B. 1,2 and 3
C. 1 only

Question 11

The recognition of government grants should only be made if:

1. The grants will never be repaid under any circumstances.


2. It is likely that the firm will comply with the qualifying conditions.
3. The grants will be received.

A. 1 and 2
B. 1,2 and 3
C. 2 and 3

Question 12

On the notification that a firm will receive a grant:


A. An account receivable will be set up, but the grant will be recorded on a cash
basis.
B. An account receivable will be set up, and the grant will be recorded on an
accruals basis.
C. Do nothing until the cash arrives.

! 62 acowtancy.com
CHAPTER 6:

IAS 40 – INVESTMENT PROPERTIES

6.1 ACCA SYLLABUS GUIDE OUTCOME 1:-


Discuss why the treatment of investment properties should differ from other
properties.

Investment property is property (land or a building or part of a building or both) held by


the owner or by the lessee under a finance lease to earn rentals or for capital
appreciation or both. An investment property may include property under an operating
lease, if used for the same purpose as other investment properties.

Generally, non-investment properties generate cash flows in combination with other


assets. A property that meets the definition of an investment property will generate cash
flows that are largely independent of the other assets held by an entity35.

6.2 ACCA SYLLABUS GUIDE OUTCOME 2:-


Apply the requirements of relevant accounting standards for investment property.

Recognition

Investment property should be recognised as an asset when

• it is probable that the future economic benefits that are associated with the
property will flow to the enterprise, and
• the cost of the property can be reliably measured.

Initial Measurement

Investment property is initially measured at cost, including transaction costs.

Measurement subsequent to initial recognition

IAS 40 permits enterprises to choose between:


a. a fair value model; and
b. a cost model.

35 June 2013 Qs 5

! 63 acowtancy.com
a. Fair value model

Under the fair value model:

• the asset is revalued to fair value at the end of each year


• the gain or loss is shown directly in the statement of profit or loss
• no depreciation is charged on the asset.

As discussed in Chapter 3, the measurement of the fair value of an investment property


is likely to require reference to current market rents for similar properties and market
interest rates36 . This may need to be adjusted to reflect the location and condition of the
property and other factors that market participants would take into account.

b. Cost model

Under the cost model the asset should be accounted for in line with the cost model laid
out in IAS 16.

• The property will be shown in the statement of financial position at cost less
accumulated depreciation.

One method must be adopted for all of an entity’s investment property.

Change is permitted only if this results in a more appropriate presentation. IAS 40 notes
that it is highly unlikely for a change from a fair value model to a cost model.

Gains or losses arising from changes in the fair value of investment property must be
included in net profit or loss for the period in which it arises

Transfers to or from investment property

Transfers to or from investment property should only be made when there is a change in
use.

When there is a transfer from investment property carried at fair value to owner-
occupied property or inventories, the property’s cost for subsequent accounting should
be its fair value at the date of change of use.

Where an owner-occupied property becomes an investment property, an entity should


apply IAS 16 up to the date of change of use. Any difference at that date between the
carrying amount of the property under IAS 16 and its fair value is a revaluation.

36 An income approach with level 2 inputs as per IFRS 13

! 64 acowtancy.com
Intra-company rentals

Property rented to a parent, subsidiary or fellow subsidiary is not investment property in


consolidated financial statements. The property is owner-occupied from the perspective
of the group. However, such property can qualify as investment property in the separate
financial statements of the lessor, if the definition of investment property is otherwise
met.

Partial own use

If the owner uses part of the property for its own use and part to earn rentals or for
capital appreciation, and the portions can be sold or leased out separately, they are
accounted for separately. If the portions cannot be sold or leased out separately, the
property is investment property only if the owner-occupied portion is insignificant.

Disposal of Investment Property

An investment property should be derecognised on disposal. The gain or loss on


disposal should be calculated as the difference between the net disposal proceeds and
the carrying amount of the asset and should be recognised as income or expense in the
statement of profit or loss.

Lecture Example 1

ABC Ltd, a manufacturing company, purchases a property for $1 million on 1 January


20X1 for its investment potential. The land element of the cost is believed to be
$400,000, and the buildings element is expected to have a useful life of 50 years. At 31
December 20X1, local property indices suggest that the fair value of the property has
risen to $1.1 million.

Show how the property would be presented in the financial statements as at 31


December 20X1 if ABC Ltd adopts:

(a) the cost model


(b) the fair value model.

! 65 acowtancy.com
Lecture example 237

Speculator owns a number of properties. An independent surveyor has assessed their


market values as

Property Cost Valuation Valuation


01-Jul-99 30-Jun-00 30-Jun-01
$ $ $
A 41,000 52,000 73,000
B 76,000 82,000 66,000

Total 117,000 134,000 139,000

Both properties had an estimated life of 50 year when they were acquired. They are
both let on short leases under commercial terms. The policy is to adopt the fair value
model in IAS 40 for investment properties.

Required:

How should properties A and B be recorded in the financial statements of Speculator?


37 Article“IAS 40 Investment Properties”, Student Accountant, Steve Scott, July 2001 (Amended)
http://www2.accaglobal.com/students/student_accountantx/archive/2001/18/57482?
session=fffffffeffffffff0a0121395395e0a2d32b3ca9cbf1f89ffbc61cf2cf55d5f8

! 66 acowtancy.com
CHAPTER 7:

IAS 38 – INTANGIBLE ASSETS

7.1 ACCA SYLLABUS GUIDE OUTCOME 1:-


Discuss the nature of internally generated and purchased intangibles.

An intangible asset is an identifiable non-monetary asset without physical substance. An


asset is a resource that is controlled by the entity as a result of past events (for
example, purchase or self-creation) and from which future economic benefits (inflows of
cash or other assets) are expected.

Therefore, the three critical attributes of an intangible asset are:


• identifiability (capable of being separated and sold or transferred)
• control (power to obtain benefits from the asset)
• future economic benefits (such as revenues or reduced future costs)

Examples of possible intangible assets include:


• computer software
• patents
• copyrights
• motion picture films
• customer lists
• franchises

Intangibles can be acquired:


• by separate purchase
• as part of a business combination
• by a government grant
• by exchange of assets
• by self-creation (internal generation)

! 67 acowtancy.com
7.2 ACCA SYLLABUS GUIDE OUTCOME 2:-
Distinguish between goodwill and other intangible assets.

Goodwill is calculated at the acquisition date as:

$
Fair value of consideration paid (shares issued plus cash paid plus
direct costs) X
Non-controlling interest
(valued either at fair value or as a proportion of net assets) X
X
Fair value of net assets of subsidiary acquired (X)
X

Goodwill may exist because of any combination of a number of possible factors:


• reputation for quality or service
• technical expertise
• possession of favourable contracts
• good management and staff.

Negative goodwill

If the difference above is negative, the resulting gain is recognised as a bargain


purchase in the statement of profit or loss.

What are the main characteristics of goodwill which distinguish it from other
intangible assets?

• It is a ‘balancing figure’. Goodwill itself is not valued but a comparison is made


between the fait value of the whole business and the fair value of the separable
net assets of the business. It cannot be valued on its own.
• Goodwill cannot be disposed of as a separate asset.
• The factors contributing to the value of goodwill cannot be valued, e.g. how can
one value the benefit of an experienced workforce?
• The value of goodwill is volatile – it can only be given a numerical value at the
time of acquisition of the whole business.

! 68 acowtancy.com
7.3 ACCA SYLLABUS GUIDE OUTCOME 3:-
Describe the criteria for the initial recognition and measurement of intangible
assets.

IAS 38 requires an enterprise to recognise an intangible only if:

• it is probable that the future economic benefits will flow to the enterprise; and
• the cost of the asset can be measured reliably.

If an intangible item does not meet these criteria, it should be written off as an expense.

How are intangible assets initially measured?

Intangible assets are initially measured at cost.

7.4 ACCA SYLLABUS GUIDE OUTCOME 4:-


Describe the subsequent accounting treatment, including the principle of
impairment tests in relation to goodwill.

How are intangible assets measured at each year end?

After initial recognition an entity must choose either the cost model or the revaluation
model for each class of intangible asset.

Cost model

After initial recognition, intangible assets should be carried at cost less any amortisation
and impairment losses.

Revaluation model

Intangible assets may be carried at a revalued amount (based on fair value) less any
subsequent amortisation and impairment losses.

The revaluation model is only used if fair value can be determined by reference to an
active market (however, such active markets are expected to be uncommon for
intangible assets). In fact, the fair values of customer contracts, brands or patents
cannot be measured using a market approach as per IFRS 13. Very often, an income
approach is used but this is not accepted under IAS 38.

Should intangible assets be amortised?

! 69 acowtancy.com
An intangible asset with a finite useful life must be amortised over that life, normally
using the straight line method with a zero residual value

What if intangible assets have an indefinite useful life?

If an intangible asset has an indefinite useful life: -

• it should not be amortised


• its useful life should be reviewed at each reporting period.

What about goodwill?

Goodwill is not amortised; it should be tested for impairment annually. An impairment


loss is the amount by which the carrying amount of an asset exceeds its recoverable
amount.

7.5 ACCA SYLLABUS GUIDE OUTCOME 5:-


Describe and apply the requirements of relevant accounting standards to
research and development expenditure.

Research is original and planned investigation undertaken with the prospect of gaining
new scientific or technical knowledge and understanding.

Development is the application of research findings or other knowledge to a plan or


design for the production of new or substantially improved materials, devices, products,
processes, systems, or services, before the start of commercial production or use.

Accounting Treatment of Research and Development

Research phase

IAS 38 states that all expenditure incurred at the research stage should be written off to
the statement of profit or loss as an expense when incurred, and will never be
capitalised as an intangible asset.

Development phase

Under IAS 38, an intangible asset must demonstrate all of the following criteria:
▪ Probable future economic benefits
▪ Intention to complete and use or sell the asset
▪ Resources (technical, financial and other resources) are adequate and available
to complete and use the asset
▪ Ability to use or sell the asset

! 70 acowtancy.com
▪ Technical feasibility of completing the intangible asset (so that it will be available
for use or sale)
▪ Expenditure can be measured reliably

If any of the recognition criteria are not met then the expenditure must be charged to the
statement of profit or loss as incurred.

Note that if all the recognition criteria have been met, capitalisation must take place:
Treatment of Capitalised Development Costs

Once development costs have been capitalised, the asset should be amortised in
accordance with the accruals concept over its finite life.

Amortisation must only begin when commercial production has commenced (hence
matching the income and expenditure to the period in which it relates). It is an expense
in the statement of profit or loss.

Each development project must be reviewed at the end of each accounting period to
ensure that the recognition criteria are still met. If the criteria are no longer met, then the
previously capitalised costs must be written off to the statement of profit or loss
immediately.

What if the research / development split is unclear?

The enterprise treats the expenditure for that project as if it were incurred in the
research phase only.

Lecture Example 1

How would you account for these 2 scenarios ?

Scenario 1

The following trial balance (extract) relates to Candel at 30 September 2008:


$’000
Capitalised development expenditure – at 1 October 2007 20,000
Development expenditure –
accumulated amortisation at 1 October 2007 6,000

Non-current assets – intangible:


In addition to the capitalised development expenditure (of $20 million), further research
and development costs were incurred on a new project which commenced on 1 October

! 71 acowtancy.com
2007. The research stage of the new project lasted until 31 December 2007 and
incurred $1.4 million of costs. From that date the project incurred development costs of
$800,000 per month. On 1 April 2008 the directors became confident that the project
would be successful and yield a profit well in excess of its costs. The project is still in
development at 30 September 2008.

Capitalised development expenditure is amortised at 20% per annum using the straight-
line method. All expensed research and development is charged to cost of sales.

ACCA F7 December 2008 Qs 2(ii)

Scenario 2

Trial Balance extract as at 30 June 2015 :

Research and development costs $7.8m

Moston commenced a research and deveopment project on 1 January 2015. It spent


$1m per month on research until 31 March 2015, at which date the project passed into
the development stage. From this date, it spent $1.6m per month until the year end (30
June 2015), at which date development was completed. However, i twas not until 1
May 2015 that the directors of Moston were confident that the new product would be a
commercial success.

ACCA F7 Sept/Dec 2015 Qs 1 part (ii)

! 72 acowtancy.com
Further Questions38

Question 1
Dempsey’s year end is 30 September 2014. Dempsey commenced the development
stage of a project to produce a new pharmaceutical drug on 1 January 2014.

Expenditure of $40,000 per month was incurred until the project was completed on 30
June 2014 when the drug went into immediate production. The directors became
confident of the project’s success on 1 March 2014. The drug has an estimated life span
of five years; time apportionment is used by Dempsey where applicable.

What amount will Dempsey charge to profit or loss for development costs, including any
amortisation, for the year ended 30 September 2014?

A. $12,000
B. $98,667
C. $48,000
D. $88,000

38 Specimen Exam Applicable from December 2014

! 73 acowtancy.com
CHAPTER 8:

IAS 36 – IMPAIRMENT OF ASSETS

8.1 ACCA SYLLABUS GUIDE OUTCOME 1:-


Define an impairment loss.

An impairment loss is the amount by which the carrying amount of an asset exceeds its
recoverable amount.

Carrying amount

The amount at which an asset is recognised in the SFP after deducting accumulated
depreciation and accumulated impairment losses.

Recoverable amount

The higher of an asset’s fair value less costs to sell and its value in use

Fair value

The amount obtainable from the sale of an asset in a bargained transaction between
knowledgeable, willing parties.

Value in use

The discounted present value of estimated future cash flows expected to arise from:
• the continuing use of an asset, and from
• its disposal at the end of its useful life.

8.2 ACCA SYLLABUS GUIDE OUTCOME 2:-


Identify the circumstances that may indicate impairments to assets.

At the end of each reporting period, review all assets to look for any indication that an
asset may be impaired.

If there is an indication that an asset may be impaired, then you must calculate the
asset’s recoverable amount.

! 74 acowtancy.com
Indications of Impairment

1. Losses / worse economic performance


2. Market value declines
3. Obsolescence or physical damage
4. Changes in technology, markets, economy, or laws
5. Increases in market interest rates
6. Loss of key employees
7. Restructuring / re-organisation

Always check for impairment of:

• an intangible asset with an indefinite useful life.


• an intangible asset not yet available for use.
• goodwill acquired in a business combination.

Determining Recoverable Amount

If fair value less costs to sell cannot be determined, then the recoverable amount is the
value in use.

For assets to be disposed of, the recoverable amount is fair value less costs to sell.

Fair Value Less Costs to Sell

If there is a binding sale agreement, use the price under that agreement less costs of
disposal.

If there is an active market for that type of asset, use market price less costs of
disposal.

The market price means current bid price if available, otherwise the price in the most
recent transaction.

If there is no active market, use the best estimate of the asset’s selling price less costs
of disposal.

Costs of Disposal

The direct added costs only (not existing costs or overhead)

! 75 acowtancy.com
Value in Use

The calculation of value in use should reflect the following elements:


• an estimate of the future cash flows the entity expects to derive from the asset in
an arm’s length transaction;

• expectations about possible variations in the amount or timing of those future


cash flows;

• the time value of money, represented by the current market risk-free rate of
interest;

• the price for bearing the uncertainty inherent in the asset.

Cash Flow Projections

• based on reasonable and supportable assumptions, the most recent budgets and
forecasts, and extrapolation for periods beyond budgeted projections.

• IAS 36 presumes that budgets and forecasts should not go beyond five years;
for periods after five years, extrapolate from the earlier budgets.

• should relate to the asset in its current condition – future restructurings to which
the entity is not committed and expenditures to improve or enhance the asset’s
performance should not be anticipated.

• should not include cash from financing activities, or income tax.

What happens if the recoverable amount is higher than the carrying value in the
SFP?

The unit and the goodwill allocated to that unit is not impaired.

! 76 acowtancy.com
What happens if the recoverable amount is lower than the carrying value in the
SFP?

The entity must recognise an impairment loss.

The impairment loss is allocated to the CGU in the following order:

1. Reduce the carrying amount of any goodwill allocated to the cash-generating


unit
2. Reduce the carrying amounts of the other assets of the unit pro rata

The carrying amount of an asset should not be reduced below the highest of:

• its fair value less costs to sell


• its value in use
.

Lecture Example 139

A company has an asset that has a carrying value of $800. The asset has not been
revalued. The asset is subject to an impairment review. If the asset was sold then it
would sell for $610 and there would be associated selling costs of $10.

The estimate of the present value of the future cash flows to be generated by the asset
if it were kept is $750.

Required:-

Determine the outcome of the impairment review.

8.3 ACCA SYLLABUS GUIDE OUTCOME 3:-


Describe what is meant by a cash generating unit (CGU)40.

The CGU is the smallest identifiable group of assets that generates cash inflows which
are (largely) independent of other asssets..

Take the example of a restaurant, the assets on their own do not generate cash, but all
together they do. We call the restaurant as a whole a CGU.

39 Article
“Impairment of Goodwill”, Student Accountant, T Clendon and S Baker, August 2009.
http://www.accaglobal.eu/pubs/students/publications/student_accountant/archive/sa_aug09_clendon_baker2.pdf
40ACCA F7 Exam June 2012 Qs 4

! 77 acowtancy.com
Where an asset forms part of a CGU, any impairment review must be made on the
group of assets as a whole. How should this impairment loss be allocated? This will be
described in the following section.

8.4 ACCA SYLLABUS GUIDE OUTCOME 4:-


State the basis on which impairment losses should be allocated, and allocate an
impairment loss to the assets of a cash generating unit.

An impairment loss should be recognised whenever recoverable amount is below


carrying amount.

The impairment loss is an expense in the statement of profit or loss (unless it relates to
a revalued asset where the value changes are recognised directly in equity).
It is important to adjust depreciation for future periods.

How should an impairment loss be allocated?

A cash-generating unit to which goodwill has been allocated shall be tested for
impairment at least annually by comparing the carrying amount of the unit,
including the goodwill, with the recoverable amount of the CGU.

If the carrying amount of the unit exceeds the recoverable amount of the unit, the entity
must recognise an impairment loss.

The impairment loss is allocated to reduce the carrying amount of the assets of the unit
in the following order:

1) reduce the carrying amount of any goodwill allocated to the cash-generating unit.
2) reduce the carrying amounts of the other assets of the unit pro rata.

Note:-

The carrying amount of an asset should not be reduced below the highest of:

• its fair value less costs to sell


• its value in use

Reversal of an Impairment Loss

The increased carrying amount due to reversal should not be more than what the
depreciated historical cost would have been if the impairment had not been recognised.

! 78 acowtancy.com
Reversal of an impairment loss is recognised as income in the statement of profit or
loss. If the reversal relates to a cash generating unit, the reversal is allocated to the
assets of the unit on a pro-rata basis according to their carrying value except goodwill.
Reversal of an impairment loss for goodwill is prohibited.

Lecture Example 2

The following carrying amounts were recorded in the books of a restaurant immediately
prior to the impairment: -

$m
Goodwill 100
Property, plant and equipment 100
Furniture and fixtures 100

The fair value less costs to sell of these assets is $260m whereas the value in use is
$270m.

Required:-

Show the impact of the impairment.

Lecture Example 3

On 1 January 20x0, Cloud Co suffers a failure of its technology. The following carrying
values, were recorded in the books immediately prior to the impairment:

$m
Goodwill 20
Technology 5
Brands 10
Land 50
Buildings 30
Other net assets 40

The recoverable value of the unit is estimated at $85 million. The technology is
worthless, following its complete failure. The other net assets include inventory,
receivables and payables. It is considered that the book value of other net assets is a
reasonable representation of its net realisable value.

Required:-

Show the impact of the impairment on 1 January.

! 79 acowtancy.com
Further Questions 41

Question 1

Riley acquired a non-current asset on 1 October 2009 at a cost of $100,000 which had
a useful economic life of ten years and a nil residual value. The asset had been
correctly depreciated up to 30 September 2014.

At that date the asset was damaged and an impairment review was performed. On 30
September 2014, the fair value of the asset less costs to sell was $30,000 and the
expected future cash flows were $8,500 per annum for the next five years.

The current cost of capital is 10% and a five year annuity of $1 per annum at 10% would
have a present value of $3·79.

What amount would be charged to profit or loss for the impairment of this asset for the
year ended 30 September 2014?

A. $17,785
B. $20,000
C. $30,000
D. $32,215

41 Specimen Exam Applicable from December 2014

! 80 acowtancy.com
CHAPTER 9:

GROUP ACCOUNTING –
THE MAIN PRINCIPLES

9.1 ACCA SYLLABUS GUIDE OUTCOME 1:-


Describe the concept of a group as a single economic unit.

The key principle underlying group accounts is the need to reflect the economic
substance of the relationship.

Owns more than 50% of equity shares


i.e.
P controls S
S

P is an individual legal entity, known as the parent. The parent is “an entity that controls
one or more entities”42.

S is an individual legal entity, known as the subsidiary.

P owns more than 50% of the ordinary shares of S. It has enough voting power to
appoint all the directors of S. P has the power to govern the financial and operating
policies of an entity so as to obtain benefits from its activities.

Although P and S remain distinct, in economic substance, they can be regarded as a


single unit, the group.

9.2 ACCA SYLLABUS GUIDE OUTCOME 2:-


Explain the objective of consolidated financial statements.

The objective of the consolidated financial statements is to show the position of the
group as if it were a single economic entity, therefore:

• Assets and liabilities of P and S are included in the consolidated statement of


financial position

42 IFRS 10 – published in May 2011

! 81 acowtancy.com
• Income and expenses of P and S are included in the consolidated statement of
profit or loss.
• All the other comprehensive income of P and S is included in the consolidated
statement of profit or loss and other comprehensive income showing other
comprehensive income.
• Intra-group balances are eliminated
• The parent’s investment in each subsidiary is offset against the parent’s portion
of equity of each subsidiary.

9.3 ACCA SYLLABUS GUIDE OUTCOME 3:-


Explain and apply the definition of a subsidiary within relevant accounting
standards.

IAS 27 Consolidated and Separate Financial Statement defines:

• Subsidiary – an entity that is controlled by another entity (known as the parent)

• Control – the power to govern the financial and operating policies of an entity so
as to obtain benefits from its activities.

9.4 ACCA SYLLABUS GUIDE OUTCOME 4:-


Identify and outline using accounting standards and other applicable regulation
the circumstances in which a group is required to prepare consolidated financial
statements.

IAS 27 outlines the circumstances in which a group is required to prepare consolidated


financial statements.

Consolidated financial statements should be prepared when the parent company has
control over the subsidiary. Control is usually based on ownership of more than 50% of
voting power.

However, IAS 27 lists the following situations where control exists, even when the
parent owns only 50% or less of the voting power of an enterprise.

(a) The parent has power over more than 50% of the voting rights by virtue of
agreement with other investments

(b) The parent has power to govern the financial and operating policies of the
enterprise by statute or under an agreement

! 82 acowtancy.com
(c) The parent has the power to appoint or remove a majority of members of
the board of directors (or equivalent governing body)
(d) The parent has power to cast a majority of votes at meetings of the board
of directors

As per IFRS 10, “an investor controls an investee if and only if the investor has all of
the following elements: -
• power over the investee i.e. the investor has existing rights that give it the ability
to direct the relevant activities (the activities that significantly affect the investee’s
returns)
• exposure, or rights to variable returns from its involvement with the investee
• the ability to use its power over the investee to affect the amount of the investor’s
returns.”

9.5 ACCA SYLLABUS GUIDE OUTCOME 5:-


Describe the circumstances when a group may claim exemption from the
preparation of consolidated financial statements.

A parent is required to present consolidated financial statements using uniform


accounting policies for like transactions and other events in similar circumstances.
A parent is not required to (but may) present consolidated financial statements if and
only if all of the following four conditions are met:
1. The parent is itself a wholly-owned subsidiary, or is a partially-owned subsidiary
of another entity and its other owners, including those not otherwise entitled to
vote, have been informed about, and do not object to, the parent not presenting
consolidated financial statements;

2. The parent's debt or equity instruments are not traded in a public market;

3. The parent did not file, nor is it in the process of filing, its financial statements
with a securities commission or other regulatory organisation for the purpose of
issuing any class of instruments in a public market; and

4. The ultimate or any intermediate parent of the parent produces consolidated


financial statements available for public use that comply with International
Financial Reporting Standards.

! 83 acowtancy.com
9.6 ACCA SYLLABUS GUIDE OUTCOME 6:-
Explain why directors may not wish to consolidate a subsidiary and outline using
accounting standards and other applicable regulation the circumstances where
this is permitted.

The directors of a parent company may not wish to consolidate some subsidiaries due
to: -
a. Poor performance of the subsidiary

b. Poor financial position of the subsidiary

c. Differing activities (nature) of the subsidiary from the rest of the group

These reasons are not permitted according to IFRSs.

As already mentioned, consolidated financial statements should include all subsidiaries


of the parent.

IFRS 3 requires exclusion from consolidation only if the parent has lost control over its
investment. An entity loses control when it loses the power to govern its financial and
operating policies. This could occur, for e.g., where a subsidiary becomes subject to the
control of the government, a regulator, a court of law, or as a result of a contractual
agreement.

If a parent loses control of a subsidiary, the parent: -

a. derecognizes the assets and liabilities of the former subsidiary from the
consolidated SFP
b. recognizes any investment retained in the former subsidiary at its fair value
c. recognizes the gain or loss associated with the loss of control attributable to the
former controlling interest.

If, on acquisition, a subsidiary meets the criteria to be classified as ‘held for sale’ in
accordance with IFRS 5 (i.e. there should be evidence that the subsidiary has been
acquired with the intention to dispose of it within 12 months, and that management is
actively seeking a buyer), then it must still be included in the consolidation but
accounted for in accordance with that standard. The parent’s interest will be presented
separately as a single figure on the face of the consolidated SFP, rather than being
consolidated like any other subsidiary. This will be described in more detail when we do
IFRS 3.

! 84 acowtancy.com
This might occur when a parent has acquired a group with one or more subsidiaries that
do not fit into its long-term strategic plans are therefore likely to be sold.

A subsidiary that has previously been excluded from consolidation and is not disposed
of within the 12 month period must be consolidated from the date of acquisition.

9.7 ACCA SYLLABUS GUIDE OUTCOME 7:-


Explain the need for using coterminous year ends and uniform accounting
policies when preparing consolidated financial statements.

Some companies in the group may have differing accounting dates. In practice, such
companies will often prepare financial statements up to the group accounting date for
consolidation purposes.

Where impracticable, the most recent financial statements of the subsidiary are used,
adjusted for the effects of significant transactions or events between the reporting dates
of the subsidiary and consolidated financial statements. The difference between the
date of the subsidiary’s financial statements and that of the consolidated financial
statements shall be no more than three months43.

IAS 27 allows the use of financial statements made up to a date not more than three
months earlier or later than the parent’s reporting date, with due adjustment for
significant transactions or other events between the dates.

43 IFRS 10

! 85 acowtancy.com
CHAPTER 10:

GROUP ACCOUNTING –
CONSOLIDATED STATEMENT OF FINANCIAL POSITION
SUBSIDIARY

10.1 ACCA SYLLABUS GUIDE OUTCOME 1:-


Prepare a consolidated statement of financial position for a simple group (parent
and one subsidiary) dealing with pre and post acquisition profits, non-controlling
interests and consolidated goodwill.
Describe and apply the required accounting treatment of consolidated goodwill.
Explain why it is necessary to eliminate intra-groups transactions.
Account for goodwill impairment.
Indicate the effect that the related party relationship between a parent and
subsidiary may have on the subsidiary’s entity statements and the consolidated
financial statements.
Explain why it is necessary to use fair values for the consideration for an
investment in a subsidiary together with the fair values of a subsidiary’s
identifiable assets and liabilities when preparing consolidated financial
statements.
Explain and account for other reserves (e.g. share premium and revaluation
reserves).
Account for the effects of fair value adjustments (including their effect on
consolidated goodwill) to:
• Depreciating and non-depreciating non-current assets
• Inventory
• Monetary liabilities
• Assets and liabilities not included in the subsidiary’s own statement of
financial position, including contingent assets and liabilities.

10.1.1 Preparing a consolidated SFP:

1. The investment in the subsidiary (S) shown in the parent’s (P) statement of financial
position is replaced by the net assets of S.

2. The cost of the investment in S is effectively cancelled with the ordinary share
capital and reserves of the subsidiary.

! 86 acowtancy.com
The consolidated statement of financial position shows:

• The net assets of the whole group (P + S)

• The share capital of the group which always equals the share capital of P only
and

• The retained profits, comprising profits made by the group (i.e. all of P’s historical
profits + profits made by S post-acquisition).

10.1.2 Goodwill

The value of a company will normally exceed the value of its net assets. The difference
is goodwill. This goodwill represents assets not shown in the statement of financial
position of the acquired company such as the reputation of the business and the loyalty
of staff.

Where less than 100% of the subsidiary is acquired, the value of the subsidiary
comprises two elements:

• The value of the part acquired by the parent;


• The value of the part not acquired by the parent, known as the non-controlling
interest.

There are two methods in which goodwill may now be calculated following the update to
IFRS 3.

(i) Proportion of net assets method (Partial Goodwill)


Only the parent’s share of goodwill is calculated.

(ii) Fair value of non-controlling interest method (Full Goodwill)


This results in 100% of the goodwill being shown in the group statement of financial
position – that belonging to the shareholders of the parent and that belonging to the
non-controlling interest.

Positive goodwill is:-

1. An intangible non-current asset in the SFP


2. Tested annually for impairment (amortisation of goodwill is not permitted)

! 87 acowtancy.com
Negative goodwill:-

1. Arises where the cost of the investment is less that the value of net assets
purchased.
2. Negative goodwill is credited directly to the statement of profit or loss.

Impairment of Goodwill

Goodwill should be checked for impairment annually. The examiner will tell you the
amount of impairment. How you account for it depends on the goodwill method used:-

1. Proportion of net assets method (Partial Goodwill)

Reduce Group Retained Earnings


Reduce Goodwill in group SFP

with the full amount of impairment given

2. Fair value of non-controlling interest method (Full Goodwill)

Reduce Goodwill in group SFP


Reduce NCI calculation by the NCI% x impairment amount
Reduce Group Retained Earnings by the % holding in S x impairment
amount

10.1.3 Pre- and Post-Acquisition Profits

Pre-acquisition profits are the reserves which exist in a subsidiary company at the
date when it is acquired.

These are included in the goodwill calculation.

Post-acquisition profits are profits made and included in the retained earnings of the
subsidiary company since acquisition.

They are included in group reserves.

Only the group share of the post-acquisition reserves of S is included in the group
statement of financial position, i.e. the reserves of S which arose after acquisition by P.

N.B. Where the acquisition occurs during the financial year, it is important to calculate
the value of profits at the date of acquisition using time-apportionment, unless stated
otherwise (see Pilot Paper Qs 1).

! 88 acowtancy.com
10.1.4 Non-controlling Interest (NCI)

A parent may not own all of the shares in the subsidiary, e.g. if P owns only 70% of the
ordinary shares of S, there is a non-controlling interest of 30%.

NCI is presented in the consolidated statement of financial position within equity,


separately from the equity of the owners of the parent.

A reporting entity must attribute the profit or loss and each component of other
comprehensive income to the owners of the parent and to the NCI.

Accounting treatment of a non-controlling interest

• In the consolidated statement of financial position, include all of the net assets of S
• Transfer back the net assets of S which belong to the non-controlling interest within
the capital and reserves section of the consolidated statement of financial position.
Where the full goodwill method is used to value the NCI (i.e. NCI is valued at its fair
value), a proportion of goodwill on acquisition is also transferred back to the NCI (as
per full goodwill method of goodwill, described above).

Lecture Example 1:

The following balances relate to H and S on 31 December 2009.

Holding Co. Subsidiary Co.


Investment in S 200
Other Net Assets 100 400

Share Capital 100 100


Retained Earnings 200 300

H acquired 80% of S on 1 January 2009 when S’s retained earnings were $80.

a) Prepare the consolidated SFP of the group using the proportion of net
assets method to value the non-controlling interest.

b) H’s policy is to value the non-controlling interest of S at the date of


acquisition at its fair value which the directors determined to be $50.

Calculate: -
i. Goodwill
ii. NCI
iii. Group Retained Earnings

! 89 acowtancy.com
10.1.5 Fair Value of Assets and Liabilities

The fair value of assets and liabilities is defined in IFRS 3 as ‘the amount for which an
asset could be exchanged or a liability settled between knowledgeable, willing parties in
an arm’s length transaction’.

IFRS 3 requires that the subsidiary’s assets and liabilities are recorded at their fair value
for the purposes of the calculation of goodwill and production of consolidated accounts.

Adjustments will therefore be required where the subsidiary’s accounts themselves do


not reflect fair value.

(1) Adjust both columns of the net assets calculation to bring the net assets to fair
value at acquisition and reporting date.

(2) At the reporting date, make the adjustment on the face of the SFP when adding
across assets and liabilities.

An adjustment should also be made for depreciation when calculating the net assets at
reporting date only. The amount of depreciation may have to be calculated based on
the remaining life (and depreciation policy) at the date of acquisition.

Lecture Example 2:

Draft SFPs of H and S on 31 March 2009 are as follows:

Holding Co. Subsidiary Co.


Cost of investment S 200
Other Net Assets 800 500

Share Capital 100 100


Retained Earnings 900 400

Two years ago H acquired 90% of S when S’s retained earnings were $100. At
acquisition, the fair value of S’s net assets exceeded their book value by $10. Any
difference in fair value is due to PPE which has a 10 year remaining useful economic
life at that date.

On the date of acquisition, the fair value of the non-controlling shareholding in S was
$26.

The group is to value the non-controlling interest at its fair value at date of acquistion.
There has been no impairment of goodwill since the acquisition.

! 90 acowtancy.com
Prepare the consolidated SFP of the group.

Lecture Example 3:

Below are the SFP of H and S as at 31 December 2004.

Holding Co. Subsidiary Co.


Investment in S 100
Other Net Assets 200 140

Share Capital 100 40


Retained Earnings 200 100

H acquired 80% of S two years ago when S’s retained earnings were $50. At that date,
S’s PPE had a fair value of $10 in excess of the carrying value and 10 years useful
economic life.

a) Prepare the consolidated SFP of the group using the proportion of net
assets method to value the non-controlling interest.

b) Calculate goodwill, NCI and group retained earnings if the group uses the
full goodwill method to value NCI. On the date of acquisition, the fair value
of non-controlling shareholding in S was $40.

10.1.6 Share for share exchanges

These are often required in the F7 exam. They form part, or all, of the cost of
investment which is used in the goodwill calculation.

If this exchange has yet to be accounted for, the double entry is always: -

Dr Cost of Investment
Cr Share capital (with the nominal value of P shares given out)
Cr Share premium (with the premium)

Lecture Example 4:

H acquired 80% of S shares via a 2 for 1 share exchange. At the date of acquisition, the
following balances were in the books of H and S:

! 91 acowtancy.com
Holding Co. Subsidiary Co.
Share Capital ($1) $400 (0.50c) $400
Share Premium $100 $50

The share price of H was $2 at the date of acquisition. This has not been accounted for.

The group uses the proportion of net assets method to value the non-controlling
interest. There has been no impairment of goodwill since the acquisition.

Show the accounting treatment required to account for the share exchange.

Lecture Example 5:

Holding Co. Subsidiary Co.


Share Capital ($1) $100 $100
Share Premium ($1) $100 $100

H acquired 80% of S shares via a 3 for 2 share exchange. The share price of H at
acquisition was $3. This has not been accounted for.

The group uses the proportion of net assets method to value the non-controlling
interest. There has been no impairment of goodwill since the acquisition.

Show the accounting treatment required to account for the share exchange.

10.1.7 Deferred and contingent consideration

Part of the purchase consideration may not be paid at the date of the acquisition, but is
deferred until a later date – deferred consideration.

Deferred consideration should be measured at fair value at the date of the


acquisition.

Where the deferred period is significant (usually one or more years), the amount of the
cash consideration will need to be discounted to a present value, at the rate for cost of
capital given in the question. In the period after the acquisition, the parent should
accrue a finance charge (at the rate of the cost of capital) in its statement of profit or
loss (which is consolidated) and add this to the carrying amount of the deferred
consideration (a liability) in its SFP (which is also consolidated):-

Debit consolidated retained earnings (with the finance charge)


Credit deferred consideration

! 92 acowtancy.com
Sometimes, the parent company may agree to pay a further amount, depending on the
post-acquisition performance of the subsidiary – contingent consideration. The payment
of contingent consideration may be in the form of equity, a liability (issuing a debt
instrument) or cash.

Any contingent consideration should always be included as long as it can be measured


reliably. It must be recognised at its fair value.

Where the contingent consideration involves the issue of shares, this should be
recognised as part of shareholders' funds under a separate caption representing shares
to be issued.

Lecture Example 6:44

Holdrite purchased 80% of the issued share capital of Staybrite on 1 April 2005. Details
of the purchase consideration given at the date of purchase are:
▪ a share exchange of three shares in Holdrite for every five shares in Staybrite
▪ the issue to the shareholders of Staybrite 8% loan notes, redeemable at par on
31 March 2008 on the basis of $100 loan note for every 125 shares held in
Staybrite
▪ a cash sum of $121 for every 100 shares in Staybrite, payable on 1 April 2007.
Holdrite’s cost of capital is 10% per annum.

The market price of Holdrite’s shares at 1 April 2005 was $4.50 per share. In order to
help fund the acquisition of new operating capacity for Staybrite, Holdrite also
subscribed for a 10% $4m loan note (2008) issued by Staybrite immediately after the
acquisition. A fair value exercise was carried out at the date of acquisition of Staybrite,
with the following results:

Carrying amount Fair value


$000 $000

Land 20,000 23,000


Plant 25,000 30,000
Inventory 5,000 6,000
The fair values have not been reflected in Staybrite’s financial statements.

In addition, a note to Staybrite’s financial statements gave details of a contingent liability


in respect of outstanding litigation. The directors of Holdrite considered that $5m would

44 Article
“Consolidations” by S. Scott – Student Accountant June/July 2006
http://www2.accaglobal.com/pubs/students/publications/student_accountant/archives/sa_jj06_scott_ACCA.pdf

! 93 acowtancy.com
be a reliable measurement of this contingent liability. The details of each company’s
share capital and reserves at 1 April 2005 are:

Holdrite Staybrite
$000 $000

Equity shares of $1 each 20,000 10,000


Share premium 5,000 4,000
Retained earnings 18,000 8,000

Required:-

Calculate the goodwill arising on the acquisition of Staybrite.

10.1.8 Intra Group Balances

If the companies within the same group trade with each other, then this will probably
lead to:

• A receivables account in one company’s SFP


• A payables account in the other company’s SFP.

These are amounts owing within the group rather than outside the group and therefore
they must not appear in the consolidated statement of financial position. They are
therefore cancelled against each other on consolidation.

10.1.9 Cash/goods in Transit

When there are goods or cash in transit, the rules are:

- If the goods or cash are in transit between P and S, make the adjusting entry to
the statement of financial position of the receiving company.

- Cash in transit adjusting entry is:

o Dr Cash in transit (increase)


o Cr Receivables account (decrease)
- Goods in transit adjusting entry is:

o Dr Inventory (increase)
o Cr Payables account (increase)

! 94 acowtancy.com
10.1.10 Unrealised Profit

Unrealised profit may arise within a group scenario on:

a. Inventory where companies trade with each other


b. Non-current assets where one company has transferred an asset to the other
company within the same group.

a. Adjustment for unrealised profit in inventory

(1) Determine the value of closing inventory which has been purchased from the
other company in the group.
(2) Use mark-up or margin to calculate how much of that value represents profit
earned by the selling company.
(3) Make the adjustments according to who the seller is.

If the seller is the parent company:

Dr Group retained earnings


Cr Group inventory (deduct the profit when adding P’s inventory + S’s
inventory on the face of the consolidated SFP).

If the seller is the subsidiary:

Dr Subsidiary retained earnings


Cr Group inventory (deduct the profit when adding P’s inventory + S’s
inventory on the face of the consolidated SFP).

Lecture Example 7:

H sells to S goods worth $600. H makes 20% profit margin. S sells $200 of these goods
at cost.

What is the unrealised profit?

Lecture Example 8:

S sells goods to H for $600. S makes a 20% mark up. H has goods at cost left in stock
worth $200.

What is the unrealised profit?

! 95 acowtancy.com
Lecture Example 9:

H sells goods to S for $100 per month making a 20% margin. H acquired 80% of S, 9
months ago. S has still in stock the last month’s goods transferred from H.

What is the unrealised profit?

Lecture Example 10:

H sold goods to S at a price of $12 million. These goods cost H $9 million. During the
year S sold $10 million (at the cost to S) of these goods for $15 million.

Calculate the unrealised profit.

Lecture Example 11:

At 1 January 2009, H acquired 80% of S shares when S’s retained earnings were $60.
The fair value of S’s net assets was $180. The difference is due to land. H sold goods to
S for $40 making a 20% margin. S has sold goods for $30 (at cost).

The SFPs of the two companies at 31 December 2009 are:

Holding Co. Subsidiary Co.


Investment in S $200
Other Net Assets $400 $300

Share Capital $100 $100


Retained Earnings $500 $200

The group uses the full goodwill method to value the non-controlling interest. On the
date of acquisition, the fair value of non-controlling interest of S was $36. There has
been no impairment of goodwill since the acquisition.

Prepare the consolidated SFP of the group.

Lecture Example 12:

The balances of H and S at 31 December 2009:

Holding Co. Subsidiary Co.


Investment in S $1000
Other Net Assets $8000 $4000

! 96 acowtancy.com
Share Capital $2000 $1000
Retained Earnings $7000 $3000

H acquired 80% of S when S’s retained earnings were $100. The fair value of S’s net
assets was equal to their carrying values at that time. S sold goods to H for $600
making 20% margin profit. H has sold 2/3 of these goods.

P’s policy is to value the non-controlling interest of S at the date of acquisition at its fair
value which the directors determined to be $250.

Prepare the consolidated SFP of the group.

b. Adjustment for unrealised profit in the transfer of non-current assets

Occasionally, a non-current asset is transferred within the group (say from a parent to a
subsidiary). The parent may have manufactured the asset as part of its normal
production (and therefore included the sale in revenue), or it may have transferred an
asset previously used as part of its own non-current assets. If the transfer is done at
cost, then, in the first case, the cost of the asset must be removed from both revenue
and cost of sales. In the second case, no elimination would be required.

If one company sells non-current assets to another company in the same group at a
profit, adjustments must be made for:

1. Profit on sale
2. Depreciation

The whole scenario has to be recreated as if the sales have never occurred.

Carrying value at reporting date X


Carrying value at reporting date if intra-group transfer
had not occurred X

Adjustment X

The double-entry of this adjustment is: -

Dr Retained Earnings of the seller


Cr Non-Current Assets (P’s NCA + S’s NCA – Adjustment for UP)

! 97 acowtancy.com
Lecture Example 13:

H sells PPE to S costing $1000 for a selling price of $1500, depreciation at 10% per
annum.

What is the unrealised profit?

Lecture Example 14:

S sold a machine with a NBV of $100 000 to H at a transfer price of $120 000 at the
year start. Group policy dictates that the machine is depreciated over its remaining life
of 5 years.

Calculate the unrealised profit on the sale of the machine.

10.1.11 Intangible Assets

The subsidiary may have intangible assets, e.g. development expenditure. These
assets can be recognised separately from goodwill if they are identifiable.

The subsidiary may also have internally-generated assets which have not yet been
recognised as intangible assets, e.g. brand. The parent will recognise this as an asset in
the consolidated financial statements.

Refer to lecture example 15 below.

10.1.12 Contingent liabilities

Contingent liabilities of the parent are recognised if their fair value can be measured
reliably.

After their initial recognition, the parent should measure contingent liabilities are the
higher of:

i. The amount that would be recognised as per IAS 37


ii. The amount initially recognised

! 98 acowtancy.com
10.1.13 Future Losses

The parent should not recognise any liabilities for future losses. It should not recognise
a liability for any plans to restructure a subsidiary.

10.1.14 Professional fees

Fees and other costs attributable to the combination should be expensed when
incurred.

Lecture Example 15:45

Highveldt, a public listed company, acquired 75% of Samson’s ordinary shares on 1


April 2005. The purchase consideration consisted of:
• a share exchange of one share in Highveldt for two shares in Samson. The
market price of Highveldt shares at the date of acquisition was $4 each
• an immediate $1.75 per share in cash
• a further amount of $81m payable on 1 April 2006. Highveldt’s cost of capital is
8% per annum.

Highveldt has only recorded the consideration of $1.75 per share.

The summarised SFPs of the two companies at 31 March 2006 are shown below:

Highveldt Samson
$m $m $m $m
Tangible non-current assets 570 380
Investments 150 nil
720 380
Current assets 130 90
Total assets 850 470

Share capital and reserves:


Ordinary shares of $1 each 270 80
Reserves:
Share premium 80 40
Revaluation reserve 40 nil
Retained earnings

45 Article
“Consolidations” by S. Scott, Student Accountant - June/July 2006 (adjusted for full goodwill method)
http://www2.accaglobal.com/pubs/students/publications/student_accountant/archives/sa_jj06_scott_ACCA.pdf

! 99 acowtancy.com
- 1 April 2005 160 120
- Year to 31 March 2006 190 350 101 221
740 341
Non-current liabilities
10% loan note nil 60
Current liabilities 110 69
Total equity and liabilities 850 470

The following information is relevant:

i. Highveldt has a policy of revaluing land and buildings to fair value. At the date of
acquisition, Samson’s land and buildings had a fair value of $20m in excess of
their carrying amounts, and at 31 March 2006 this had increased by a further
$4m (ignore any additional depreciation).
ii. Samson had established a line of products under the brand name of Titanware.
Acting on behalf of Highveldt, a firm of specialists had valued the brand name at
$40m with an estimated life of 10 years as at 1 April 2005. The brand is not
included in Samson’s statement of financial position.
iii. Immediately after acquisition, Highveldt sold Samson an item of plant for $15m
that it had manufactured at a cost of $10m. The plant had an estimated life of five
years (straight-line depreciation) and no residual value.
iv. On 1 October 2005 Samson issued $60m 10% (actual and effective rate) loan
notes. Highveldt subscribed for $20m of this issue. Samson has not paid any
interest on this loan, but it has recorded the amount due as a current liability.
Highveldt has also accrued for its interest receivable on this loan.
v. Post-acquisition, Samson sold goods at a price of $18m to Highveldt; $5m of
these goods were still in the inventory of Highveldt at 31 March 2006. Samson
applied a mark-up on cost of 25% to these goods.
vi. Due to poor trading performance, consolidated goodwill was impaired by $20m.
vii. Highveldt’s policy is to value the non-controlling interest of Samson at the date of
acquisition at its fair value. The directors determined this to be $75m.

Required:-

Prepare the consolidated statement of financial position of Highveldt at 31 March


2006.

10.2 ACCA SYLLABUS GUIDE OUTCOME 2:-


Goodwill and Non-Controlling Interests.

As discussed in Section 10.1.2, the parent measures any non-controlling interest either:
(i) at fair value as determined by the directors of the acquiring company (often
called the ‘full goodwill’ method); or

! 100 acowtancy.com
(ii) at the non-controlling interest’s proportionate share of the acquiree’s
(subsidiary’s) identifiable net assets.

Lecture Example 1646

Parent pays $100m for 80% of Subsidiary which has net assets with a fair value of
$75m. The directors of Parent have determined the fair value of the NCI at the date of
acquisition was $25m.

Required:-

Calculate goodwill using both the proportionate and the full goodwill method.

10.2.1 What happens when there is an impairment?

Lecture Example 1747

Parent owns 80% of Subsidiary (a CGU). Its identifiable net assets at 31 March 2010
are $500.

Scenario 1: Full Goodwill Method

$
Net assets included in the consolidated statement
of financial position 500
Consolidated goodwill (calculated under method (i)) 200
700

NCI 140

Scenario 2: Proportionate Method

$
Net assets included in the consolidated statement
of financial position 500
Consolidated goodwill (calculated under method (ii)) 160

46 Article
“IFRS 3, Business Combinations” by S. Scott, Student Accountant, July 2010
http://www.accaglobal.com/content/dam/acca/global/PDF-students/2012/sa_jul10_F7_IFRS3.pdf
47 Article
“IFRS 3, Business Combinations” by S. Scott, Student Accountant, July 2010
http://www.accaglobal.com/content/dam/acca/global/PDF-students/2012/sa_jul10_F7_IFRS3.pdf

! 101 acowtancy.com
660

NCI 100

An impairment review of Subsidiary was carried out at 31 March 2010.

Required:-

For scenarios 1 and 2, calculate the impairment losses and show how they would
be allocated if the impairment review concluded that the recoverable of
Subsidiary was:

(i) $450
(ii) $550

Further Questions

Question 148

Consolidated financial statements are presented on the basis that the companies within
the group are treated as if they are a single (economic) entity.

Which of the following are requirements of preparing group accounts?

(i) All subsidiaries must adopt the accounting policies of the parent
(ii) Subsidiaries with activities which are substantially different to the activities of
other members of the group should not be consolidated
(iii) All entity financial statements within a group should (normally) be prepared to
the same accounting year end prior to consolidation
(iv) Unrealised profits within the group must be eliminated from the consolidated
financial statements

A. All four
B. (i) and (ii) only
C. (i), (iii) and (iv)
D. (iii) and (iv)

48 Specimen Exam Applicable from December 2014

! 102 acowtancy.com
Question 2

Venus Co acquired 75% of Mercury Co’s 100,000 $1 ordinary share capital on 1


November 2011. The consideration consisted of $2 cash per share and 1 share in
Venus Co for every 1 share acquired in Mercury Co. Venus Co shares have a nominal
value of $1 and a fair value of $1.75. The fair value of the non-controlling interest was
$82,000 and the fair value of net assets acquired was $215,500.

What should be recorded as goodwill on acquisition of Venus Co in the consolidated


financial statements?
A. $16,500
B. $147,750
C. $91,500
D. $63,375

Question 3

Honey Co acquired 75% of Bee Co on 1 April 2013, paying $2 for each ordinary share
acquired. The fair value of the non-controlling interest at 1 April 2013 was $300. Bee
Co’s individual financial statements as at 30 September 2013 included:
$
Statement of financial position
Ordinary share capital ($1 each) 1,000
Retained earnings 710
1,710

Income statement
Profit after tax for the year 250

Profit accrued evenly throughout the year.

What is the goodwill on acquisition on 1 April 2013?

A. $715
B. $90
C. $517
D. $215

! 103 acowtancy.com
CHAPTER 11:

GROUP ACCOUNTING:
CONSOLIDATED STATEMENT OF FINANCIAL POSITION
ASSOCIATE

11.1 ACCA SYLLABUS GUIDE OUTCOME 1:-


Define an associate and explain the principles and reasoning for the use of
equity accounting.

IAS 28 defines an associate as:

An entity over which the investor has significant influence and that is neither a
subsidiary nor an interest in joint venture.

Significant influence is the power to participate in the financial and operating policy
decisions of the investee but is not control or joint control over those policies.

There are several indicators of significant influence, but the most important are usually
considered to be a holding of between 20% and 50% of the voting shares and board
representation.

Equity accounting brings an associate investment into the parent company’s financial
statements initially at cost.

The carrying amount of the investment is then adjusted in each period by the group
share of the profit of the associate less any impairment losses.49

49If a company can no longer exert significant influence over another company, it will be treated under IAS 39. It
should no longer be equity accounted from the date of loss of significant influence. Its carrying amount at that date
will be its initial recognition value under IAS 39 and thereafter it will be carried at fair value.

! 104 acowtancy.com
11.2 ACCA SYLLABUS GUIDE OUTCOME 2:-
Prepare a consolidated statement of financial position to include a single
subsidiary and an associate.

The associate is included as a non-current asset investment in the SFP calculated as:

$’000
Cost of investment X
P’s share of post acquisition profits of A X
Less: impairment losses of A (X)
X

The group share of the associate’s post acquisition profits /losses and the impairment of
goodwill must also be included in the group retained earnings calculation.

Group retained earnings

$
Parent (100%) X
S – group share of post-acquisition reserves X
A – group share of post-acquisition reserves X
Less: impairment losses to date (S + A) X
X

Fair Values

If the fair values of the associate’s net assets at acquisition are materially different from
their book value, the net assets should be adjusted in the same way as for a subsidiary.

Trading with the Associate

Generally, the associate is considered to be outside the group. Therefore if a group


company trades with the associate, the resulting payables and receivables will remain in
the consolidated statement of financial position.

Unrealised profit in inventory

(1) Determine the value of closing inventory which is the result of a sale to or from the
associate.

! 105 acowtancy.com
(2) Use mark-up/margin to calculate the profit earned by the selling company.

(3) Make the required adjustments. These will depend upon who the seller is:

Parent company selling to associate – the profit element is included in the parent
company’s accounts.

Dr Group retained earnings


Cr Investment in associate

Associate selling to parent company – the profit element is included in the associate
company’s accounts.

Dr Group retained earnings


Cr Group inventory

N.B. Don’t forget to take the unrealised profit and multiply by the percentage holding
which the parent has in the associate.

Lecture Example 1:

H sold goods to A worth $10 for $40. A sold 2/3 of these goods. H’s share in A is 40%.

What is the unrealised profit?

Lecture example 2:50

On 1 October 2006, Plateau acquired the following non-current investments:

Three million equity shares in Savannah by an exchange of one share in Plateau for
every two shares in Savannah, plus $1.25 per acquired Savannah share in cash. The
market price of each Plateau chare at the date of acquisition was $6, and the market
price of each Savannah share at the date of acquisition was $3.25.

Thirty per cent of the equity shares of Axle at a cost of $7.50 per share in cash.

50 December 2007 Question 1 revised by S. Scott in Article “IFRS 3, Business Combinations” July 2010
http://www.accaglobal.com/content/dam/acca/global/PDF-students/2012/sa_jul10_F7_IFRS3.pdf

! 106 acowtancy.com
Only the cash consideration of the above investments has been recorded by Plateau. In
addition, $500,000 of professional costs relating to the acquisition of Savannah are
included in the cost of the investment.

The summarised draft statements of financial position of the three companies at 30


September 2007 are:
Plateau Savannah Axle
$’000 $’000 $’000

Assets
Non-current assets:
Property, plant
and equipment 18,400 10,400 18,000
Investments in Savannah
and Axle 13,250 nil nil
Financial asset investments 6,500 nil nil
38,150 10,400 18,000
Current assets:
Inventory 6,900 6,200 3,600
Trade receivables 3,200 1,500 2,400
Total assets 48,250 18,100 24,000

Equity and liabilities


Equity shares of $1 each 10,000 4,000 4,000
Retained earnings
- at 30 September 2006 16,000 6,000 11,000
- for year ended
30 September 2007 9,250 2,900 5,000
35,250 12,900 20,000
Non-current liabilities
7% Loan notes 5,000 1,000 1,000

Current liabilities 8,000 4,200 3,000


Total equity and liabilities 48,250 18,100 24,000

The following information is relevant:

(i) At the date of acquisition, Savannah had five years remaining of an agreement to
supply goods to one of its major customers. Savannah believes it is highly likely
that the agreement will be renewed when it expires. The directors of Plateau
estimate that the value of this customer based contract has a fair value of $1m,
an indefinite life, and has not suffered any impairment.

! 107 acowtancy.com
(ii) On 1 October 2006, Plateau sold an item of plant to Savannah at its agreed fair
value of $2.5m. Its carrying amount prior to the sale was $2m. The estimated
remaining life of the plant at the date of sale was five years (straight-line
depreciation).

(iii) During the year ended 30 September 2007, Savannah sold goods to Plateau for
$2.7m. Savannah had marked up these goods by 50% on cost. Plateau had a
third of the goods still in its inventory at 30 September 2007. There were no intra-
group payables/receivables at 30 September 2007.

(iv) At the date of acquisition the non-controlling interest in Savannah is to be valued


at its fair value. For this purpose Savannah’s share price at that date can be
taken to be indicative of the fair value of the shareholding of the non-controlling
interest. Impairment tests on 30 September 2007 concluded that neither
consolidated goodwill nor the value of the investment in Axle had been impaired.

(v) The financial asset investments are included in Plateau’s statement of financial
position (above) at their fair value on 1 October 2006, but they have a fair value
of $9m at 30 September 2007.

(vi) No dividends were paid during the year by any of the companies.

Required:-

Prepare the consolidated statement of financial position for Plateau as at 30


September 2007.

(20 marks)

Lecture Example 3

On 1 October 2005 Pumice acquired the following non-current investments:

• 80% of the equity share capital of Silverton at a cost of $13.6 million

• 50%of Silverton’s 10% loan note at par

• 1.6 million equity shares in Amok at a cost of $6.25 each.

The summarised draft statement of financial positions of the three companies at 31


March 2006 are:

! 108 acowtancy.com
Pumice Silverton Amok
$’000 $’000 $’000
Non-current assets
Property. Plant and equipment 20,000 8,500 16,500
Investments 26,000 Nil 1,500

46,000 8,500 18,500


Current assets 15,000 8,000 11,000

Total assets 61,000 16,500 29,000

Equity and liabilities


Equity
Equity sharesof$1 each 10,000 3,000 4,000
Retained earnings 37,000 8,000 20,000
47,000 11,00024,000
Non-currents liabilities
8% loan note 4,000 Nil Nil
10% loan note Nil 2,000 Nil
Current liabilities 10,000 3,500 5,000

Total equity and liabilities 61,000 16,500 29,000

The following information is relevant:

(i) The fair value of Silverton’s assets were equal to their carrying amounts with the
exception of land and plant. Silverton’s land had a fair value of $400,000 in excess
of its carrying amount and plant had fair value of $1.6 million in excess of its
carrying amount. The plant had a remaining life of four years (straight-line
depreciation) at the date of acquisition.

(ii) In the post acquisition period Pumice sold goods to Silverton at a price of $6
million. These goods had cost Pumice $4 million. Half of these goods were still in
the inventory of Silverton at 31 March 2006. Silverton had a balance of $1.5 million
owing to Pumice at 31 March 2006 which agreed with Pumice’s records.

(iii) The net profit after tax for the year ended 31 March 2006 was $2 million for
Silverton and $8 million for Amok. Assume profits accrued evenly throughout the
year.

! 109 acowtancy.com
(iv) Pumice’s policy is to value the non-controlling interest at fair value at the date of
acquisition. The fair value of the non-controlling interest at acquisition was
determined to be $3 million.

An impairment test at 31 March 2006 concluded that the consolidated goodwill was
impaired by $500,000 and the investment in Amok was impaired by $200,000.

(v) No dividends were paid during the year by any of the companies.

Required:-

(a) Discuss how the investments purchased by Pumice on 1 October 2005


should be treated in its consolidated financial statements.
(b) Prepare the consolidated statement of financial position for Pumice as at 31
March 2006.

(ACCA F7 Exam Pilot Paper Qs 1 revised {FV of NCI method})

Further Questions

Question 151

An associate is an entity in which an investor has significant influence over the investee.

Which of the following indicate(s) the presence of significant influence?


(i) The investor owns 330,000 of the 1,500,000 equity voting shares of the
investee
(ii) The investor has representation on the board of directors of the investee
(iii) The investor is able to insist that all of the sales of the investee are made to a
subsidiary of the investor
(iv) The investor controls the votes of a majority of the board members

A. (i) and (ii) only


B. (i), (ii) and (iii)
C. (ii) and (iii) only
D. All four

51 Specimen Exam Applicable from December 2014

110
! acowtancy.com
Question 252

The Caddy group acquired 240,000 of August’s 800,000 equity shares for $6 per share
on 1 April 2014. August’s profit after tax for the year ended 30 September 2014 was
$400,000 and it paid an equity dividend on 20 September 2014 of $150,000.

On the assumption that August is an associate of Caddy, what would be the carrying
amount of the investment in August in the consolidated statement of financial position of
Caddy as at 30 September 2014?

A. $1,455,000
B. $1,500,000
C. $1,515,000
D. $1,395,000

Question 3

According to IAS 28, Investments in associates, which, if any, of the following


statements are correct?

(1) Equity accounting will always be used when an investing company holds between
20% - 50% of the equity shares in another company
(2) Dividends received from an investment in associate will be presented as investment
income in the consolidated accounts.

Statement 1 Statement 2
A. Correct Correct
B. Correct Incorrect
C. Incorrect Correct
D. Incorrect Incorrect

52 Specimen Exam Applicable from December 2014

111
! acowtancy.com
Question 4

Which of the following investments of Coffee Co should be equity accounted in the


consolidated financial statements?
(1) 40% of the non-voting preference share capital in Tea Co
(2) 18% of the ordinary share capital in Café Co with two of the five directors of Coffee
Co on the board of Café Co
(3) 50% of the ordinary share capital of Choc Co, with five of the seven directors of
Coffee Co on the board of Choc Co

A. 1 and 2
B. 2 only
C. 1 and 3 only
D. 2 and 3 only

112
! acowtancy.com
CHAPTER 12:

GROUP ACCOUNTING:
CONSOLIDATED STATEMENT OF PROFIT OR
LOSS - SUBSIDIARY

12.1 ACCA SYLLABUS GUIDE OUTCOME 1:-


Prepare a consolidated statement of profit or loss and consolidated statement of
profit or loss and other comprehensive income for a simple group dealing with an
acquisition in the period and non-controlling interest.

Basic principles

1. From sales revenue to profit after tax, include all of P’s income and expenses plus
all of S’s income and expenses (where a mid-year acquisition has occurred, these
must be time-apportioned).

2. Once the profit after tax is calculated, deduct share profits due to the non-controlling
interest.

Impairment of goodwill

The charge for the year, re impairment of goodwill, will be passed through the
consolidated statement of profit or loss, usually through operating expenses, unless
stated otherwise.

Non-controlling interest

This is calculated as: NCI% x subsidiary’s profit after tax (taken from S’s column of
consolidation schedule).

Dividends

A payment of a dividend by S to P must be cancelled. Any dividend income shown in


the consolidated statement of profit or loss must arise from investments other than
those in subsidiaries or associates.

113
! acowtancy.com
Fair Values

If a depreciating non-current asset of the subsidiary has been revalued, this will result in
an adjustment to the consolidated statement of profit or loss.

Unrealised Profits

The adjustment to unrealised profit should be shown as an increase to cost of sales. It


affects the books of the SELLER.

Sales and Purchases

Intra-group trading must be eliminated from the consolidated statement of profit or loss.

Consolidated sales revenue = P’s revenue + S’s revenue – intra-group sales

Consolidated cost of sales = P’s COS + S’s COS – intra-group sales

Interest on loan

If loans are outstanding between group companies, intra-group loan interest will be paid
and received. Both the loan and loan interest must be excluded from the consolidated
results.

Transfers of non-current assets

If one group company sells a non-current asset to another group company, the following
adjustments are needed in the statement of profit or loss:-

1. Any profit or loss arising on the transfer must be deducted


2. The depreciation charge must be adjusted so that it is based on the cost of the
asset to the group

Mid-year acquisitions

If a subsidiary is acquired part way through the year, then it is important to time
apportion the results of S in the year of acquisition. Unless indicated otherwise, assume
that revenue and expenses accrue evenly.

114
! acowtancy.com
Lecture Example 1

In the post-acquisition period, Holdrite sold goods to Staybrite for $72,000. Holdrite
achieved a mark-up on these goods of 20% on cost. At the year end, Staybrite, still had
$42,000 (at the transfer price) of these goods in its inventory.

On 1 April 2005, Holdrite sold an item of plant to Staybrite for $120,000. Holdrite had
manufactured this plant at a cost of $100,000 and treated it as a normal sale. Staybrite
is depreciating this plant on a straight-line basis over a five-year life with no estimated
residual value.

On 1 October 2005, Staybrite issued a $2m 8% (actual and effective rate) loan note,
redeemable in 2010. Holdrite had subscribed for $800,000 of this issue. All due interest
had been paid by 31 March 2006.

Required:-

Show how the above transactions should be accounted for in the consolidated
financial statements for the year ended 31 March 2006.

How to prepare extracts for Retained Earnings and NCI

Example:

P controls 80% of S


RE b/fwd: P $500, S $100


RE of S at date of acquisition: $80



Consolidated Profit for the year (from Consolidated Statement of Profit or Loss): $300

Attributable to owners of P: $250

Attributable to NCI: $50


Share Capital of S at date of acquisition: $1000


Retained Earnings

Balance b/fwd (500 + 80% of post-acq profits (100 - 80)) = 516

Profit for the year (attributable to owners of P) 250

less any dividends paid by P 0

Balance c/fwd 766


115
! acowtancy.com
NCI


Bal b/fwd –
take NA of S at beg of year x NCI share (SC 1000 + RE 100)x 20% 220

add Profit attributable to NCI 50

less any dividends which are paid outside the group (not to P) 0

Balance c/fwd 270

Further Questions

Question 153

On 1 January 2014, Viagem acquired 80% of the equity share capital of Greca.

Extracts of their statements of profit or loss for the year ended 30 September 2014 are:

Viagem Greca
$’000 $’000
Revenue 64,600 38,000
Cost of sales (51,200) (26,000)

Sales from Viagem to Greca throughout the year ended 30 September 2014 had
consistently been $800,000 per month. Viagem made a mark-up on cost of 25% on
these sales. Greca had $1·5 million of these goods in inventory as at 30 September
2014.

What would be the cost of sales in Viagem’s consolidated statement of profit or loss for
the year ended 30 September 2014?

A. $59·9 million
B. $61·4 million
C. $63·8 million
D. $67·9 million

Question 2

Tulip Co acquired 70% of the voting share capital of Daffodil Co on 1 March 2012.

53 Specimen Exam Applicable from December 2014

116
! acowtancy.com
The following extracts are from the individual profit or loss statements of the two
companies for the year ended 31 August 2012:
Tulip Co Daffodil Co
$ $
Revenue 61,000 23,000
Cost of sales (42,700) (13,800)
Gross Profit 18,300 9,200

What should be the consolidated gross profit for the year ended 31 August 2012?

A. $21,520
B. $22,900
C. $27,500
D. $24,740

Question 3

Panther Co acquired 80% of the equity shares in Seal Co on 31 August 2012.

The profit or loss statements of Panther Co and Seal Co for the year ended 31
December 2012 showed:

Panther Co Seal Co
$ $
Revenue 100,000 62,000
Cost of sales 25,000 16,000

During October 2012, sales of $6,000 were made by Panther Co to Seal Co. None of
these items remained in inventory at the year-end.

What is the consolidated revenue for Panther Group for the year ended 31 December
2012?

A. $156,000
B. $118,667
C. $144,800
D. $114,667

117
! acowtancy.com
CHAPTER 13:

GROUP ACCOUNTING:
CONSOLIDATED STATEMENT OF PROFIT OR
LOSS - ASSOCIATE

13.1 ACCA SYLLABUS GUIDE OUTCOME 1:-


Prepare consolidated financial statements to include a single subsidiary and an
associate.

The consolidated statement of profit or loss includes:

• 100% of the income and expenses of the parent and subsidiary company on a
line by line basis.

• One line ‘share of profit of associates’ which includes the group share of
any associate’s profit after tax.

Equity accounting

The equity method of accounting requires that the consolidated statement of profit or
loss:

• Does not include dividends from the associate


• Instead includes group share of the associate’s profit after tax less any
impairment of the associate in the year.

Trading with the associate

Generally the associate is considered to be outside the group.

Therefore any sales or purchases between group companies and the associate are not
normally eliminated and will remain part of the consolidated figures in the statement of
profit or loss.

118
! acowtancy.com
Unrealised Profit (UP)

If the associate sells goods to the parent at a profit and some of these goods are still in
inventory at year end, it is important to calculate the share of profits of associate
company as follows:-

Share of Associate Profit after Tax =


(% holding in A x profit of A for the year) - (% holding in A x UP) – any
impairment of the investment in A

Parent company selling to associate –

Dr P Cost of sales
Cr Investment in associate

Associate selling to parent company – the profit element is included in the associate
company’s accounts.

Dr Share of Associate Profit after Tax


Cr P Inventory

Dividends

Dividends from associates are excluded from the consolidated statement of profit or
loss. Only the group share of the associate’s profit is included.

Lecture Example 1:

Several years ago H acquired 80% of the ordinary share capital of S and 30% of A.
Their results for the year ended 31 December 2005 were as follows:

Statement of Profit or Loss

H S A
(80%) (30%)
Turnover 100 100 100
COS (40) (40) (40)
Expenses (40) (40) (40)
Profit after Tax 20 20 20

119
! acowtancy.com
Prepare the consolidated statement of profit or loss for the year ended 31
December 2005.

Lecture Example 2:

Several years ago H acquired 80% of the ordinary share capital of S and 30% of A.
Their results for the year ended 31 December 2005 were as follows:

Statement of Profit or Loss

H S A

Turnover 1000 800 400


COS (600) (200) (100)

Expenses (100) (100) (40)


Tax (100) (100) (60)
Profit after Tax 200 400 200

H acquired 80% of S. At that date, 3 years ago, S’s PPE had a fair value of $100 in
excess of the carrying value and a 5 year useful economic life.

Prepare the consolidated statement of profit or loss for the year ended 31
December 2005.

Lecture Example 3:

The following are the summarised accounts of H, S and A for the year ended 31
December 2009.

Statement of Profit or Loss

H S A
$ $ $
Sales Revenue 573 600 314 000 150 000
Operating costs (300 000) (200 000) (90 000)

Operating profit 273 600 114 000 60 000


Interest payable (20 000) (14 000) (8 000)
Dividend income from S 14 400
Dividend income from A 4 000
Dividend income from other sources 10 000

! 120 acowtancy.com
Profit before tax 282 000 100 000 52 000
Tax (72 000) (30 000) (16 000)

Profit for the financial year 210 000 70 000 36 000

Statements of financial position as at 31 December 2009

H S A
$ $ $
Investment in S Ltd (60%) 60 000
Investment in A Ltd (25%) 50 000
Other assets 300 000 120 000 100 000
410 000 120 000 100 000

$ $ $
Ordinary shares 20 000 30 000 10 000
Retained earnings 330 000 66 000 70 000
Current liabilities 60 000 24 000 20 000
410 000 120 000 100 000

1. The shares in S and A were acquired on 1 January 2009, when the balances of
the retained earnings accounts were:
S $20 000
A $50 000

2. Goodwill in the subsidiary has suffered an impairment of 20% of its value, and
the investment in associate has suffered an impairment of $7 000.
Subsidiary goodwill impairment is recognised in operating costs and impairment
of the associate is charged against associate profits. H has accounted for the
dividends from subsidiary and associate.

3. The H group values the non-controlling interest at its proportionate share of the
fair value of the subsidiary’s identifiable net assets.

Prepare the consolidated statement of profit or loss for the year ended 31
December 2009 and the consolidated statement of financial position as at that
date.

! 121 acowtancy.com
CHAPTER 14:

INVENTORIES AND BIOLOGICAL ASSETS


IAS 2:- INVENTORIES

14.1 ACCA SYLLABUS GUIDE OUTCOME 1:-


Describe and apply the principles of inventory valuation.

Inventories should be measured at the lower of cost and net realisable value

14.1.1 Cost

The cost of inventories will consist of all the following costs: -

1. Purchase
2. Costs of conversion
3. Other costs incurred in bringing the inventories to their present location and
condition, e.g. carriage inwards

Do NOT include:

1) Abnormal amounts
2) Storage costs
3) Administration overheads
4) Selling costs

Example

Item A has the following costs:

Direct Labour 100


Raw Materials 200
Depreciation on production machines 10
Factory Manager wage 10
Other production Overheads 8
Admin Overheads 5
What is the cost?

! 122 acowtancy.com
Direct Labour 100
Materials 200
Depreciation (PRODUCTION) 10
Factory Manager Wage (production) 10
Production overheads 8
328

Do not include admin costs

14.1.2 Net Realisable Value

The net realisable value of an item is essentially its net selling proceeds after all costs
have been deducted.

It is calculated as:

$
Estimated selling price X
Less: estimated costs of completion (X)
Less: estimated selling and distribution costs (X)
X

Lecture Example 1

In the post balance sheet period, prior to authorising for issue the financial statements
of Tentacle for the year ended 31 March 2007, the following material information has
arisen.

Sales of some items of product W32 were made at a price of $5.40 each in April and
May 2007. Sales staff receive a commission of 15% of the sales price on this product.
At 31 March 2007 Tentacle had 12,000 units of product W32 in inventory included at
cost of $6 each.

Required:-

State and quantify how the item above should be treated when finalising the
financial statements of Tentacle for the year ended 31 March 2007.

(ACCA Paper 2.5 June 2007 Qs 5b (ii))

! 123 acowtancy.com
14.2 ACCA SYLLABUS GUIDE OUTCOME 2:-
Apply the requirements of relevant accounting standards for biological assets

IAS 41: Agriculture

IAS 41 Agriculture sets out the accounting for agricultural activity – it introduces a fair
value model to agriculture accounting.

IAS 41 applies to: -

1. Biological assets:

• Living animals or plants, e.g. trees in a plantation or orchard, cultivated plants,


sheep, cattle that are in the process of growing, degenerating, regenerating and/
or procreating and which are expected to eventually result in agricultural
produce;

2. Agricultural produce:

• the harvested product obtained from a biological asset

Examples of biological assets and agricultural produce are:

Biological Asset Agricultural Produce


Vine Grapes
Chicken Eggs
Dairy cattle Milk
Sheep Wool
Pigs Meat

The standard distinguishes between two broad categories of biological assets: -


Consumable biological assets: those that are to be harvested as agricultural produce or
sold as biological assets. E.g. livestock intended for the production of meat, livestock
held for sale, fish in farms, crops such as maize and wheat.
Bearer biological assets: those other than consumable biological assets. E.g. livestock
from which milk is produced or livestock held for breeding, vines, fruit trees.

! 124 acowtancy.com
The standard does not apply to agricultural land or intangible assets related to
agricultural activity. After harvest, IAS 2 is applied.
14.2.1 Initial Recognition

An entity should recognise a biological asset or agriculture produce only when: -

1. CONTROL - The entity controls the asset as a result of past events,


2. VALUE - It is probable that future economic benefits will flow to the entity, and
3. MEASUREMENT - The fair value or cost of the asset can be measured reliably.

14.2.2 Measurement:

14.2.2.1 Biological assets

Biological assets should be measured on initial recognition and at subsequent reporting


dates at fair value less estimated point-of-sale costs, unless fair value cannot be
reliably measured. They should be classified as a separate class of assets in the SFP.

14.2.2.2 Agricultural produce

Agricultural produce should be measured at fair value less estimated point-of-sale


costs at the point of harvest. The point of harvest represents the transition between
accounting for agricultural produce assets under IAS 41 and IAS 2: - Fair value less
estimated point-of-sale costs at the point of harvest forms ‘cost’ for the purposes of IAS
2.

Agricultural produce should be classified as inventory in the statement of financial


position and disclosed separately either in the SFP or in the notes.

14.2.2.3 Point of sale costs

Point-of-sale costs include commissions to brokers and dealers, levies by regulatory


agencies and commodity exchanges, and transfer taxes and duties. Point-of-sale costs
exclude transport and other costs necessary to get assets to a market.

14.2.2.4 Fair Value

IAS 41 presumes that fair value can be reliably measured for most biological assets and
agricultural produce. However, that presumption can be rebutted when: -

1. There is lack of market-determined prices;


2. Values are not otherwise available

! 125 acowtancy.com
3. Alternative estimates of fair value are clearly unreliable

In such a case, the asset is measured at cost less accumulated depreciation and
impairment losses. This method is only allowed on initial recognition.

The entity must still measure all of its other biological assets at fair value less costs to
sell. If circumstances change and fair value becomes reliably measurable, a switch to
fair value less costs to sell is required.

14.2.3 Gains and Losses:

The gain or loss arising on initial recognition of biological assets at fair value less costs
to sell is reported as a gain or loss in the statement of profit or loss.

The change in fair value less costs to sell of a biological asset between two reporting
dates is also reported as a gain or loss in the statement of profit or loss.

A gain or loss arising on initial recognition of agricultural produce at fair value less costs
to sell should be included in net profit or loss for the period in which it arises.

14.2.4 Other issues:

The change in fair value of biological assets is part physical change (growth, etc.) and
part price change.

The aggregate gain or loss is analysed as follows:


1. The price change: -

the value of the biological asset at prices prevailing as at the current reporting date 

less the value of the biological asset at prices prevailing as at the previous reporting
date


2. The physical change: -



the value of the biological asset in its state as at the current reporting date

less the value of the biological asset in its state as at the previous reporting date:

Separate disclosure of the two components is encouraged but not required.

14.2.5 Government Grants:

! 126 acowtancy.com
Government grants received in respect of biological assets measured at fair value less
costs to sell are reported as income when the grant becomes receivable. If such a grant
is conditional (including where the grant requires an entity not to engage in certain
agricultural activity), the entity recognises it as income only when the conditions have
been met.

Lecture Example 254

As at 31 December 20X1, a plantation consists of 100 Pinus Radiata trees that were
planted 10 years earlier. Pinus Radiata takes 30 years to mature, and will ultimately be
processed into building material for houses or furniture. The enterprise’s weighted
average cost of capital is 6% p.a.

Only mature trees have established fair values by reference to a quoted price in an
active market. The fair value (inclusive of current transport costs to get 100 logs to
market) for a mature tree of the same grade as in the plantation is:

As at 31 December 20X1: 171

As at 31 December 20X2: 165

Required:

Calculate the aggregate gain or loss arising during the current period, according to IAS
41.

54 Article
“IAS 41, Agriculture”, Simon Riley, March 2002
http://www.accaglobal.com/ie/en/student/dipifr/dipifr-resources/technical-articles/ias-41.html

! 127 acowtancy.com
CHAPTER 15:

FINANCIAL INSTRUMENTS

15.1 ACCA SYLLABUS GUIDE OUTCOME 1:-


Explain the need for an accounting standard on financial instruments

In recent years, there has been a significant growth worldwide in the number and
complexity of financial instruments in international financial markets. There were
numerous concerns about the accounting treatment of financial instruments which led to
demands for an accounting standard.

There are four reporting standards that deal with financial instruments: -
• IAS 32 – Financial instruments: Presentation
• IAS 39 – Financial instruments: Recognition and Measurement
• IFRS 7 – Financial instruments: Disclosures
• IFRS 9 – Financial instruments

On 12 November 2009, the IASB issued IFRS 9 Financial Instruments as the first step
in its project to replace IAS 39. IFRS 9 introduces new requirements for classifying and
measuring financial assets that must be applied starting 1 January 2013, with early
adoption permitted.

On 28 October 2010, the IASB reissued IFRS 9, incorporating new requirements on


accounting for financial liabilities, and carrying over from IAS 39 the requirements for
derecognition of financial assets and financial liabilities. The IASB intends to expand
IFRS 9 to add new requirements for impairment of financial assets measured at
amortised cost, and hedge accounting.

15.2 ACCA SYLLABUS GUIDE OUTCOME 2:-


Define financial instruments in terms of financial assets and financial liabilities.

Financial Instrument - any contract that gives rise to a financial asset in one entity and
a financial liability or equity instrument of another.

Financial Asset – any asset that is

• Cash

! 128 acowtancy.com
• A contractual right to receive cash or another financial asset from another
enterprise
• A contractual right to exchange financial instruments with another enterprise
under conditions that are potentially favourable
• An equity instrument of another enterprise e.g. investments in shares of another
entity

Examples: - Cash, trade receivables, investment in equity shares

Financial Liability – a liability that is a contractual obligation:


• To deliver cash or another financial asset to another enterprise; or
• To exchange financial instruments on potentially unfavourable terms

Examples: - Trade payables, debenture loans, redeemable preference shares

And the trickier stuff…..IAS 39 also applies to derivatives such as call and put options,
forwards, futures, and swaps. IAS 39 does not apply to an entity’s own shares.

15.3 ACCA SYLLABUS GUIDE OUTCOME 3:-


Indicate how financial instruments should be initially measured

15.3.1 Initial Recognition of Financial Instruments

Financial assets and liabilities should be recognised on entering into the contract NOT
when the contract is settled. The asset or liability is measured at fair value – the actual
transaction price on the reporting date.

15.3.2 Derecognition of Financial Instruments

A financial asset should be derecognised (removed from the SFP) when: -

1. The contractual rights to the cash flows of the financial asset have expired or
2. The financial asset has been sold and all the risks and rewards of ownership
have been transferred from the seller to the buyer

A financial liability should be derecognised when: -

1. The obligation specified in the contract is discharged, cancelled or expires

! 129 acowtancy.com
15.4 ACCA SYLLABUS GUIDE OUTCOME 4:-
Indicate for the following categories of financial instruments how they should be
measured and how any gains and losses from subsequent measurement should
be treated in the financial statements: -

i) Amortised cost
ii) Fair value through other comprehensive income (including where an
irrevocable election has been made for equity instruments that are not
held for trading)
iii) Fair value through profit or loss

15.4.1 Financial Liabilities – Subsequent Measurement

After initial recognition, an entity should measure financial liabilities as follows: -

1. Liabilities that are held for trading and derivatives – re-measured to fair value at
each reporting date. Any gains and losses are included in the statement of profit
or loss.

2. All other financial liabilities – measured at amortised cost using the effective
interest rate method.

Illustration 1

A company issues 5.9% loan notes at their nominal value of $1,000. The loan notes are
repayable at a premium of $250 after 5 years. The effective interest rate is 10%.

What amount will be recorded as a financial liability when the loan notes are
issued?

What amounts will be shown in the Statement of Profit or Loss and SFP for years
1 – 5?

Year Op Balance Finance Cost Payment Cl Balance


10% 5.9%
1 1,000 100 (59) 1,041

! 130 acowtancy.com
2 1,041 104 (59) 1,086
3 1,086 109 (59) 1,136
4 1,136 113 (59) 1,190
5 1,190 119 (1,250 + 59) 0

When the loan notes are issued:

Dr Bank $1,000
Cr Loan notes $1,000

Statement of Profit or Loss

1 2 3 4 5
Finance
(100) (104) (109) (113) (119)
Costs

SFP

1 2 3 4 5
Non-current
1,041 1,086 1,136
liabilities
Current
1,190 0
liabilities

Deep Discount

A deep discount bond is issued at a significant discount to its par value. Typically, it has
a coupon rate much lower than market rates of interest, e.g. a 2% bond when the
market interest is 6% p.a.

This deep discount must simply be taken off from the opening value.

! 131 acowtancy.com
Illustration 2:

5% $1,000 loan with a deep discount of $200 creating an effective interest rate of 9%.

Year Op Balance Finance Cost Payment Cl Balance


9% 5%
1 800 72 (50) 822
2 822 74 (50) 846

15.5 ACCA SYLLABUS GUIDE OUTCOME 5:-


Explain the distinction between debt and equity capital.

Loans go to debt (liabilities); ordinary shares go to equity. Why?

It is back to the conceptual framework again and also to the important concept of
substance over form. The definition of liability includes the need for a present obligation.
As interest MUST be paid but dividends may not, only loans have this obligation and so
go to liabilities.

If an entity issues preference shares that pay a fixed rate of dividend and are redeemed
at a future date (redeemable preference shares), then there is a contractual obligation
to deliver cash and, therefore, should be recognised as a liability.

In contrast, normal preference shares do not have a fixed maturity (irredeemable


preference shares), and the issuer does not have a contractual obligation to make any
payment. Therefore, they are equity.

What about new complex items such as convertible loans? These are known as
compound instruments and are partly debt and partly equity.

! 132 acowtancy.com
15.6 ACCA SYLLABUS GUIDE OUTCOME 6:-
Apply the requirements of relevant accounting standards to the issue and finance
costs of convertible debt.

15.6.1 Compound financial instruments (Convertible loans)

A compound instrument is a financial instrument that has characteristics of both equity


and liabilities, for e.g. 2% Convertible Loan $1,000.

This basically means the company has offered the bank the option to convert the loan
at the end into shares instead of simply taking $1,000.

The important thing to notice is that that the bank has the option to do this. Should the
share price not prove favourable then it will simply take the $1,000 as normal.

If the bank wants cash rather than shares, it may still accept the option. The terms of
conversion are normally quite generous and the bank will accept the conversion and
then sell the shares on the market for a profit.

Better Interest rate

In exchange for these favourable terms of conversion, the bank will offer the company a
favourable interest rate compared to normal loans.
Higher Fair Value of loan

This lower interest rate has effectively increased the fair value of the loan to the
company. IAS 32 suggests this fair value needs to be shown immediately on inception.

Lower loan figure in SFP

Important: If the fair value of a liability has increased, the amount payable shown will be
lower. After all, fair value increases are good news and we all prefer lower liabilities!

! 133 acowtancy.com
Illustration 3

2% $1,000 loan repayable after 4 years


Normal (non-convertible) loans carry an interest rate of 5%.
The present value of $1 receivable at the end of each year, based on discount rates of
2% and 5% are:

2% 5%
End of year 1 0·980 0·952
2 0·961 0·907
3 0·942 0·864
4 0·924 0·823

Take what the company pays and discount them using the figures above as follows:

Capital $1,000 discounted @ 5% (4 years SINGLE discount figure):


1,000 x 0.823 = 823

Interest $20 discounted @ 5% (4 years CUMULATIVE):


20 x 3.546 = 71
894

This $894 represents the fair value of the loan and this is the figure we use in the SFP
initially.

The remaining $106 (1,000 - 894) goes to equity.

Op Balance Finance Cost Payment Cl Balance


894 45 -20 919

Lecture Example 1

8% Convertible loan note (2010)

On 1 April 2006 Wellmay issued an 8% convertible loan note with a nominal value of
$600,000 at par. It is redeemable on 31 March 2010 at par or it may be converted into
equity shares of Wellmay on the basis of 100 new shares for each $200 of loan note. An
equivalent loan note without the conversion option would have carried an interest rate of
10%. Interest of $48,000 has been paid on the loan and charged as a finance cost.

! 134 acowtancy.com
The present value of $1 receivable at the end of each year, based on discount rates of
8% and 10% are:
8% 10%
End of year 1 0·93 0·91
2 0·86 0·83
3 0·79 0·75
4 0·73 0·68

Show the accounting treatment in the statement of profit or loss and statement of
financial position as at 31 March 2007.
(June 2007 Qs 2 Part v)

15.6.2 Interest, dividends, transaction costs

The accounting treatment of interest, dividends, losses and gains relating to a financial
instrument follows the treatment of the instrument itself.

For example, dividends paid in respect of preference shares classified as a liability will
be charged as a finance expense in the statement of profit or loss. Dividends paid on
shares classified as equity will be reported in the statement of changes in equity
(SOCIE). They are deducted from retained earnings.

Category Treatment
FVTPL Expense
FVTOCI Add to Initial Opening Balance
Amortised Cost - Asset Add to Initial Opening Balance
Amortised Cost - Liability Deduct from Initial Opening Balance

.
15.7 ACCA SYLLABUS GUIDE OUTCOME 7:-
Indicate how any gains and losses from subsequent measurement of financial
assets should be treated in the financial statements

As stated in Section 15.3.1, financial assets are initially measured at fair value – this is
likely to be the purchase consideration paid and will normally exclude transaction costs.

! 135 acowtancy.com
15.7.1 Subsequent measurement of financial assets

Subsequent measurement of financial instruments depends upon whether the financial


asset is an investment in a debt instrument or an equity instrument.

Debt instruments

Debt instruments would normally be measured at fair value through profit or loss
(FVTPL) but could be measured at amortised cost, provided these two tests are
passed: -

a) Business model test


The asset is held within a business model whose objective is to hold the assets
to collect the contractual cashflows

b) Contractual cash flow characteristics test


This test determines whether the contractual terms of the financial asset give rise
to cash flows on specified dates that are solely of principal and interest based
upon the principal amount outstanding.

If an asset is not held for trading AND the cash flows are principal and interest only BUT
the business model is also to sell these loans, then these debt instruments are
measured at fair value through other comprehensive income. 55 Even if a financial
asset passes both tests, it is still possible to designate a debt instrument as FVTPL if
doing so eliminates or significantly reduces an accounting mismatch.

Equity instruments

Equity instruments are measured at either: -


• Fair value through profit or loss or
• Fair value through other comprehensive income (FVTOCI) if these conditions are
met: -
o The equity instrument cannot be held for trading. Hence, the intention is to
hold it to collect dividend income
o There must be an irrevocable choice for this designation upon initial
recognition. In this situation, initial recognition will also include directly
attributable transaction costs. Dividend income is still recognised in profit
or loss.

55 In 2014 the IASB published the complete version of IFRS 9, Financial Instruments, which replaces most of the guidance in IAS 39. It includes
amended guidance for the classification and measurement of financial assets by introducing a FVTOCI category for certain debt instruments.

! 136 acowtancy.com
Lecture Example 2

A company invests $5,000 in 10% loan notes. The loan notes are repayable at a
premium after 3 years. The effective rate of interest is 12%. The company intends to
hold the loan notes till maturity and the contractual cash flows consist solely of
repayments of interest and principal. Hence, it has chosen to record the financial asset
at amortised cost.

What amounts will be shown in the Statement of Profit or Loss and SFP for years
1-3?

Lecture Example 3

A company invested in 20,000 shares of a listed company in November 2010 at a cost


of $5.20 per share. At 31 December 2010, the shares have a market value of $5.90.

Situation 1 – the company is planning on selling these shares in March 2011.

Situation 2 – the company is not planning on selling these shares in the short term.

For both situations, prepare extracts from the statement of profit or loss for the
year ended 31 December 2010 and a SFP as at that date.

15.8 Factoring of Receivables

Factoring of receivables is where a company transfers its receivables balances to


another organisation (a factor) for management and collection and receives an
advance on the value of those receivables in return.

The key point here is:


Have the risks of the debts been passed on to the factor?

If the entity transfers ownership of receivables to the factor, derecognise


receivables from SFP. The entity is not holding these receivables to collect the
contractual cash flows but to sell them.

The only entry is:

Dr Cash
Cr Trade receivables

! 137 acowtancy.com
If the entity retains ownership of receivables, it means that the significant
risks and rewards of the receivables are not transferred. Hence, keep
receivables in the entity’s SFP and record the proceeds received from the
factor as a loan:

Dr Cash
Cr Loan

When the factor collects cash from the customers:

Dr Loan
Cr Trade Receivables

If the factor returns any debts to the seller:

Dr Loan
Cr Cash

Dr Bad debts
Cr Trade Receivables

Lecture Example 4

In September 2006 Angelino sold (factored) some of its trade receivables to


Omar, a finance house. On selected account balances Omar paid Angelino 80%
of their book value. The agreement was that Omar would administer the
collection of the receivables and remit a residual amount to Angelino
depending upon how quickly individual customers paid. Any balance
uncollected by Omar after six months will be refunded to Omar by Angelino.

Required:.

Describe how the above transaction should be treated in the financial statements
of Angelino for the year ended 30 September 2006.
(Paper 2.5 December 2006 Qs 3b)

15.9 Impairment of Financial Assets

Impairment loss: -

The difference between

! 138 acowtancy.com
• acquisition cost and
• current fair value (for equity instruments) or recoverable amount (for debt
instruments)

15.9.1 Financial assets held at amortised cost and FVTOCI

i. Provide for expected losses

On initial recognition of the asset, the entity must create a credit loss allowance.
This must be equal to 12 months’ expected credit losses, i.e. multiply the
probability that a default will occur in the next 12 months by the expected losses
arising from this default.

ii. If, subsequent to initial recognition, credit risk increases significantly, provide an
allowance for lifetime expected credit losses. This will replace the 12 month
expected losses mentioned in (i) above.

15.9.2 Financial assets carried at FVTPL

i. Impairment loss recognised in profit or loss for the year.

! 139 acowtancy.com
Financial Liabilities and Equity Instruments

Issuing Financial
Instruments

Equity instruments
Financial liabilities

Contain an Evidence of an
obligation ownership interest
in the residual
net assets

If classified as If classified as FVTPL; Initial measurement


amortised cost; initial measurement is is at fair value less
initial measurement at fair value, and then issue costs and,
is at fair value less so is subsequent subsequently, no
issue costs and then measurement with change as equity
subsequent gains and losses being instruments are not
measurement is at recognised in the re-measured
amortised cost statement of profit or loss

! 140 acowtancy.com
Financial Assets

Financial Assets

Debt instruments Equity instruments

Contain an Evidence of an
obligation ownership interest in
to be repaid the residual net asset

If classified as If classified as If classified as


amortised cost: FVTPL: initial FVTOCI: initial
initial measurement measurement is at measurement is at
is at fair value plus fair value, and then fair value plus
transaction costs and so is subsequent transaction cost and
then subsequent measurement with subsequent
measurement is at gains and losses measurement is at
amortised cost being recognised in fair value with gains
income and losses being
recognised directly
in reserves and OCI

The business model The default An irrevocable


and cash flow test accounting treatment election at inception
have to be met – also used where has to be made. On
the asset is acquired disposal no recycling
for trading purposes of previously
or to avoid an recognised gains or
accounting mismatch losses to income

! 141 acowtancy.com
Both diagrams extracted from Articles: What is a Financial Instrument? Part 1 & 2 by T. Clendon, Student
Accountant, 2012, http://www.accaglobal.com/content/dam/acca/global/PDF-students/2012s/sa_jul12_7p2_fin-
instruments.pdf & http://www.accaglobal.com/content/dam/acca/global/PDFstudents/2012s/
sa_aug12_f7p2_instruments.pdf

Further Questions56

Question 1

Dravid issues 10,000 $1 ordinary shares for cash consideration of $2.50 each. Issue
costs are $1,000.

Required
Explain and illustrate how the issue of shares is accounted for in the financial
statements of Dravid.

Question 2

Laxman raises finance by issuing zero coupon bonds at par on the first day of the
current accounting period with a nominal value of $10,000. The bonds will be redeemed
after two years at a premium of $1,449. The effective rate of interest is 7%.

Required
Explain and illustrate how the loan is accounted for in the financial statements of
Laxman.

Question 3

Broad raises finance by issuing $20,000 6% four-year loan notes on the first day of the
current accounting period. The loan notes are issued at a discount of 10%, and will be
redeemed after four years at a premium of $1,015. The effective rate of interest is 12%.
The issue costs were $1,000.

Required
Explain and illustrate how the loan is accounted for in the financial statements of Broad.

56 Articles:
What is a Financial Instrument? Part 1 & 2 by T. Clendon, Student Accountant, 2012, http://
www.accaglobal.com/content/dam/acca/global/PDF-students/2012s/sa_jul12_7p2_fin-instruments.pdf &
http://www.accaglobal.com/content/dam/acca/global/PDF-students/2012s/sa_aug12_f7p2_instruments.pdf

! 142 acowtancy.com
Question 4

Graham Gooch issues a 3% $200,000 two-year convertible bond at par. The effective
rate of interest of the instrument is 8%. The terms of the convertible bond is that the
holder of the bond, on the redemption date, has the option to convert the bond to equity
shares at the rate of 10 shares with a nominal value of $1 per $100 debt rather than
being repaid in cash. Transaction costs can be ignored. Graham Gooch will account for
the financial liability arising using amortised cost.

Required
Explain the accounting for the issue of the convertible bond.

! 143 acowtancy.com
CHAPTER 16:

IFRS 16 –
LEASING
IFRS 16 was issued in January 2016 and supersedes IAS 17, ‘Leases’.

IAS 17 required lessees to classify their leases as either finance leases or operating
leases and account for those two types of leases differently.

IFRS 16 introduces a single lessee accounting model for the lessee and requires a
lessee to recognise assets and liabilities for all leases with a term of more than 12
months, unless the underlying asset is of low value (we will be discussing this later on.).
This means that all leases (excluding the two exceptions of less than 12 months and
low value) will be showing on the SFP as an asset and liability.

What is a lease?

A contract, or part of a contract, that conveys the right to use an asset (the underlying
asset) for a period of time in exchange for consideration.

The contract is a lease if it meets 3 criteria: -

1. The asset must be identifiable either explicitly specified in the contract (e.g. the
serial number of the asset is included) or implicitly specified (there is only one
asset which is capable of being used to meet the terms of the contract).

2. A customer is not considered to have the right to use the asset if the supplier
has the right to substitute the asset during the period of use.

3. The customer has the right to control the use of an identified asset. The
customer must have both: -
a. The right to get substantially all of the benefits from using the asset
b. The right to direct the use of the asset: how and for what the asset is
used.

! 144 acowtancy.com
Examples from IFRS 16

• A customer enters into a contract with an airport operator (supplier) to use a


space in the airport to sell its goods (coffee, tea, sandwiches etc.) for a three-
year period.

1. Identified asset?
No, as no area is specified in the contract. The customer may be allocated a number of
areas.

2. Substantive substitution rights?


Yes and the supplier will benefit by using its retail space in the most efficient manner.

3. Customer has the right to control the use of the identified asset?
No, it is the supplier who allocates the space to the customer and obtains all of the
economic benefits from use of the concession space.

This is not a lease but is a service. The customer is purchasing space which can be
changed by the supplier.

• A customer enters into a contract with a supplier to use retail space Z (which is part
of a large shopping complex) for three years.

1. Identified asset?
Yes, specific retail space identified.

2. Substantive substitution rights?


No, although the supplier has the practical ability to substitute another retail outlet.
However, it will be required to pay for relocation costs and the circumstances, at
inception date, are not considered likely to arise.

3. Customer has the right to control the use of the identified asset?
Yes, the customer has exclusive use and the right to obtain all of the economic benefits
from the use of the retail space.

This is a lease.

Who is the lessor?

An entity that provides the right to use an underlying asset for a period of time in
exchange for consideration.

! 145 acowtancy.com
Who is the lessee?

An entity that obtains the right to use an underlying asset for a period of time in
exchange for consideration.

What is the lease term?

The lease term is the non-cancellable period for which a lessee has the right to use an
underlying asset, together with both:

(a) periods covered by an option to extend the lease if the lessee is reasonably certain
to exercise that option; and
(b) periods covered by an option to terminate the lease if the lessee is reasonably
certain not to exercise that option.

16.1 ACCA SYLLABUS GUIDE OUTCOME 1:-


Account for right-of-use assets and lease liabilities in the records of the lessee

Main Principle: -

At the beginning of the lease, the lessee should recognise a lease liability and a right-of-
use asset.

What is a right-of-use asset?

It is an asset that represents a lessee’s right to use an underlying asset for the lease
term.

Lease Liability

The lease liability is initially measured at the present value (PV) of the lease
payments that have not been paid. It should include: -

1. Fixed payments
2. Variable payments that depend on an index or rate
3. Residual value guarantees (i.e. the lessor is guaranteed that the underlying asset
will not be worth less than a specified amount at the end of the lease term)
4. Options to purchase the asset that are reasonably certain to be exercised
5. Termination penalties, if the lease term reflects the expectation that these will be
incurred.

! 146 acowtancy.com
To find the PV of the lease payments, we will need a discount rate, which should be the
rate implicit in the lease. If this cannot be determined, the entity should use the
incremental borrowing rate (the rate at which it could borrow funds to purchase a similar
asset).

Right-of-use Asset

The right-of-use asset is initially measured at cost.

This includes: -

1. The amount of the lease liability (the PV of lease payments)


2. Lease payments made at or before the lease started
3. Any initial direct costs
4. The estimated costs of removing or dismantling the underlying asset as per the
conditions of the lease.
the amount of the lease liability plus any initial direct costs incurred by the lessee. Adjustments may also be required for lease incentives, payments at or prior to commencement and restoration obligations or similar
the amount of the lease liability plus any initial direct costs incurred by the lessee. Adjustments may also be required for lease incentives, payments at or prior to commencement and restoration obligations or similar

Subsequent Measurement

Lease Liability

After initial recognition, the lease liability is increased by the interest charge and
reduced by the cash payments.

It is important to establish whether the payments are being made at the start of the year
(i.e. in advance) or at the end of the year (i.e. in arrears).

Illustration 1: Payment in Arrears

• Lease term 3 years


• Rental of $5,000 paid on 31 December each year
• PV of lease payments is $12,000
• Interest rate implicit in the lease is 12.04%

Initially,

Dr Right-of-use asset $12,000


Cr Lease liability $12,000

! 147 acowtancy.com
Year Opening Finance cost (Payment) Closing
balance ($) ($) ($) balance ($)

1 12,000 1,445 -5,000 8,445


2 8,445 1,017 -5,000 4,463
3 4,463 537 -5,000 0

Finance cost of $1,445 goes to the P/L in year 1.

Current vs Non-current Liability

The lease liability must be split between the amount that is to be paid within a year and
the remainder which is payable in more than one year.

At the end of year 1,

Non-Current Liability
Lease liability $4,463

Current Liability
Lease liability (8,445 – 4,463) $3,982

Illustration 2: Payment in Advance

• Lease term 4 years


• Rental of $8,000 payable in advance
• PV of lease payments is $28,000
• Interest rate implicit in the lease is 9.8%

Year Opening balance (Payment) ($) Balance Finance Closing


($) cost ($) balance ($)

1 28,000 -8,000 20,000 1,960 21,960


2 21,960 -8,000 13,960 1,368 15,328
3 15,328 -8,000 7,328 718 8,046
4 8,046 -8,000 0

! 148 acowtancy.com
The difference of $46 is due to rounding of interest rate.

Finance cost of $1,960 goes to P/L in year 1.

In SFP, at the end of year 1,

At the end of year 1,

Non-Current Liability
Lease liability $13,960

Current Liability
Accrued interest $1,960
Lease liability (21,960 – 13,960) – 1,960 $6,040

Right-of-Use Asset

The right-of-use asset is measured using the cost model (cost less accumulated
depreciation less accumulated impairment losses) unless: -

• the asset is an investment property and the lessee applies fair value model to its
investment property; or
• the asset relates to a class of PPE for which the lessee applies the revaluation
model.

Depreciation on the right-of-use asset

• If ownership of the asset is transferred to the lessee at the end of the lease term
– charge depreciation over the asset’s remaining useful economic life.

• If ownership of the asset is not transferred to the lessee at the end of the lease
term – charge depreciation on the shorter of the useful life and the lease term.

Lecture example 1

On 1 January 2017, AB Co entered into a two year lease for a machine. The contract
contains the option to extend the lease term for a further year. At the commencement
date, AB Co is reasonably certain to exercise this option. The machine has a useful
economic life of eight years.

! 149 acowtancy.com
Lease payments due at the end of each year are:

Year 1 and 2: $10,000 each

Year 3: $15,000

To obtain the lease, AB Co incurred initial direct costs of $5,000. However, the lessor
reimbursed $3,000 of these costs.

The interest rate implicit in the lease is not readily determinable. AB Co’s incremental
borrowing rate is 5 % p.a.

Question 1

Calculate the initial carrying amount of the right-of-use asset and the lease liability.

Question 2

Prepare extracts from AB Co’s financial statements for the year ended 31 December
2017.

Question 3

How is depreciation calculated if ownership of the asset is transferred to the lessee at


the end of the lease term?

Lecture example 2

On 1 October 2015 Nina Co entered into an agreement to lease a machine that had an
estimated life of four years. The lease period is also four years with annual rentals of
$10,000 payable in advance from 1 October 2015. The machine is expected to have a
nil residual value at the end of its life. The present value of the lease payments is
$35,000. The interest rate implicit in the lease is 10%.

How should the lease be accounted for in the financial statements of Nina for the year
ended 31 March 2017?

! 150 acowtancy.com
16.2 ACCA SYLLABUS GUIDE OUTCOME 2:-
Explain the exemption from the recognition criteria for leases in the records of the
lessee.

A lessee may choose not to apply the requirements mentioned above for: -

1. Short-term leases
2. Leases for which the underlying asset is of low value

When the exemption is applied, lease payments are recognised as an expense over
the lease term on a straight-line basis or on another systematic basis, if that basis
represents better the pattern of the lessee’s benefits.

What is a short-term lease?

A short-term lease is defined as “a lease that, at the commencement date, has a lease
term of 12 months or less”.

A lease that contains a purchase option cannot be classified as a short-term lease.

What is a low-value asset?

IFRS 16 does not give an explicit definition of a low-value asset. However, it states that
low-value assets are assets with a value, when new, of $5,000 or less 57.

E.g. of low-value assets are tablet and personal computers, small items of office
furniture and telephones. However, a motor vehicle would not qualify as a low-value
asset even if the leased vehicle is old at the beginning of the lease. Why? The vehicle’s
value, when new, is expected to be more than $5,000.

Illustration 3

• Lease term 5 years


• Total value of office equipment when new is $4,000
• Entity elects to apply the low-value asset exemption
• Lease payments are $1,200 for the first 2 years, $1,000 for the last 3 years
• The lessee’s benefit under the lease arises on a straight-line basis over the full
lease term

Total payments = ($1,200 x 2) + ($1,000 x 3) = $5,400


Length of lease = 5 years
Lease expense recognised each year = $5,400/5 = $1,080

57 IFRS 16, Basis for conclusion

! 151 acowtancy.com
In the first year, there is a prepayment of $120 ($1,200 - $1,080) which is included in the
SFP.

Lecture example 3

A Co leases telephones for 5 years. The total value of the telephones when new is
$4,500. A Co elects to apply the low-value asset exemption.

Lease payments are payable as follows:

Year 1 Nil : rent-free period


Years 2 & 3 $1,500 per year
Years 4 & 5 $1,300 per year

The lessor provides a lease incentive with a value of $500.

The lessee’s benefit under the lease arises on a straight-line basis over the full lease
term.

Calculate the lease expense to be recognised each year. For year 1, calculate any
accruals or prepayments relating to the lease.

16.3 ACCA SYLLABUS GUIDE OUTCOME 3:-


Account for sale and leaseback agreements.

Let’s start with an example to illustrate a sale and leaseback agreement.

P Co sells a machine to Y Co on 1 January 2017. On the same date, P Co enters into a


contract with Y Co for the right to use the asset for the next 5 years.

In this example, P is the seller of the machine but then he leases it back – he becomes
a lessee.

C is the buyer but then becomes the lessor.

The important question here is this: Has a performance obligation been satisfied?

We discussed performance obligations in IFRS 15, Revenue from Contracts with


Customers.

! 152 acowtancy.com
When is a performance obligation satisfied?
When the buyer obtains control of the asset, i.e. it has the ability to obtain
substantially all of the remaining benefits.

If the transfer is a sale: -

• The seller-lessee must measure the right-of-use asset at the proportion of the
previous carrying amount that relates to the rights retained.
• The profit or loss on disposal is based on the rights transferred to the buyer-
lessor.

If the transfer is not a sale: -

• The seller-lessee will continue to recognise the asset in its SFP


• It will recognise a financial liability (a loan) for the amount of the proceeds
obtained from the transfer.

Illustration 4

On 1 January 2017, A Co sells an item of machinery to a finance company and leases it


back for a period of four years. The remaining useful life of the asset is four years. Is
this a sale?

No, A Co is retaining the asset for the remainder of its useful life. Hence, a performance
obligation has not been satisfied. The “sales proceeds” effectively represents a loan: - a
financial liability should be recognised in the books of A Co and a finance cost
recognised each year in the P/L.

Illustration 5

A seller-lessee sells an office building for its fair value of $1.8m. Its carrying amount at
that time was $1m.

On the same date, the seller-lessee leases back the building for 18 years, paying
$120,000 p.a. in arrears.

The interest rate implicit in the lease is 4.5%.

The present value of the annual payments, discounted at this rate of 4.5%, is
$1,459,000.

The sale of the machine has been assessed to meet the satisfaction of a performance
obligation under IFRS 15.

! 153 acowtancy.com
Hence, the transfer is a sale.

1. Recognise the right-of-use asset at the proportion of the previous carrying


amount that relates to the rights retained.

Rights retained = (1,459,000/1,800,000) x 1,000,000 = $810,556

2. Calculate the lease liability (= PV of the lease payments)


These are given as $1,459,000

Dr Cash $1,800,000
Dr Right-of-use asset $810,556
Cr Asset $1,000,000
Cr Lease liability $1,459,000
Cr Gain on sale (P/L) $151,556 (balancing figure)

The gain on sale of asset is the proportion of the total gain on disposal of
$800,000($1.8m - $1m) that relates to the rights transferred to the customer: -

(1,800,000 – 1,459,000) = 341,000/1,800,000 x 800,000 = $151,556.

The right-of-use asset and the lease liability will then be accounted for similar to the
rules described above.

Lecture example 4

On 1 January 2017, P Co sells an item of plant to Q Co for its fair value of $300,000.
Prior to the sale, the plant had a carrying value in P’s books of $120,000.

On the same date, P Co enters into a contract with Q Co to lease back the asset for the
next five years, paying $50,000 at the end of each year.

The interest rate implicit in the lease is 10%.

The sale of the asset has been assessed to meet the satisfaction of a performance
obligation under IFRS 15.

How should P Co account for this transaction on 1 January 2017?

! 154 acowtancy.com
CHAPTER 17:

IFRS 15: Revenue from Contracts with


Customers
17.1 ACCA SYLLABUS GUIDE OUTCOME 1
Explain and apply the principles of recognition of revenue:
i. Identification of contracts
ii. Identification of performance obligations
iii. Determination of transaction price
iv. Allocation of the price to performance obligations
v. Recognition of revenue when/as performance obligations are satisfied

Introduction

IFRS 15, Revenue from contracts with customers, was issued in May 2014. It replaces
both IAS 18, Revenue, and IAS 11, Construction Contracts. It applies to most contracts
with customers.

Please note the definition of revenue given in this standard and also in the Framework:

‘Revenue’ is income arising in the course of an entity’s ordinary activities.

17.2 Recognition and Measurement – The Revenue Model

17.2.1 Core Principle

An entity should recognise revenue: -

1. to record the transfer of promised goods or services to the customer


2. in an amount that reflects the consideration to which the entity expects to be
entitled in exchange for those goods or services.

This core principle is set in a five-step model framework.

! 155 acowtancy.com
17.2.2 The five-step model framework

Step 1 Identify the contract(s) with the customer

A contract with a customer will fall within the scope of IFRS 15 when all the following
criteria are met:

• The parties to the contract have approved the contract;


• Each party’s rights can be identified;
• The payment terms and conditions can be identified;
• The contract has commercial substance; and
• The collection of an amount of consideration to which the entity is entitled to in
exchange for the goods or services is probable.

A contract can be oral, written or implied.

Step 2 Identify the performance obligations in the contract

A contract includes promises to transfer the goods or services to the customer. These
promises are called performance obligations. An entity would account for a performance
obligation separately only if the promised good or service is distinct.

A good or service is distinct if the following criteria are met:

1. The customer can benefit from the good or service on its own or together with
other readily available resources;
2. The entity’s promise to transfer the good or service to the customer is separately
identifiable from other promises in the contract.

Lecture Example 158

A software developer entered into a contract with a customer to supply the following:

1. A software licence,
2. An installation service
3. Software updates for 2 years
4. Technical support, both online and by telephone, for 2 years

58 Adapted from, Example 11, Illustrative examples, IFRS 15

! 156 acowtancy.com
The developer sells the above separately. The installation service is routinely performed
by other entities and does not significantly modify the software. The software remains
functional without the updates and the technical support.

Are the goods or services promised to the customer distinct in accordance with
IFRS 15 ‘Revenue from contracts with customers’?

Step 3 Determine the transaction price

The transaction price would be the amount of consideration that an entity expects to be
entitled to in exchange for transferring promised goods or services to a customer.
If a contract contains a variable amount, the entity will estimate the amount to which it
will be entitled under the contract. The consideration can also vary if an entity’s right to
consideration is contingent on the occurrence of a future event.

These variable contingent amounts should only be included where it is highly probable
that there will not be a reversal of revenue when any uncertainty associated with the
variable consideration is resolved.

Examples of where a variable consideration can arise: - discounts, rebates, refunds,


credits, price concessions, incentives, performance bonuses, penalties.

Lecture Example 259

PC Ltd enters in a contract with a customer to sell computers for $200 per computer on
1 January 20x5. If the customer purchases more than 1000 computers in a calendar
year, the contract states that the price per unit is retrospectively reduced to $190 per
computer. As a result of this, the consideration in the contract is variable. PC’s year end
is 31 December.

As at 31 March 20x5, PC Ltd sells 80 computers to the customer. Therefore it estimates


that the customer’s purchase will not exceed the 1000 unit threshold required for the
volume discount in the calendar year.

However, at the beginning of May 20x5, the customer acquired another company and at
the end of the second quarter, 30 June 20x5, PC Ltd sells an additional 500 computers
to the customer. Now, PC Ltd estimates that the customer’s purchases will exceed the
1000 unit threshold for the calendar year and therefore it would have to retrospectively
reduce the price per unit.

59 Adapted from, Example 24, Illustrative examples, IFRS 15

! 157 acowtancy.com
Calculate the revenue PC Ltd would recognise in: -

a) The first quarter ending 31 March 20x5

b) The second quarter ending 30 June 20x5

Step 4 Allocate the transaction price to the performance obligations

Where a contract has many performance obligations, an entity shall allocate the
transaction price to the performance obligations in the contract by reference to their
relative stand-alone selling prices. If a standalone selling price is not directly
observable, an entity will need to estimate it. How?

E.g. A telephone company gives a free telephone set to customers when they sign a
two-year contract. The contract is for $50 per month and the telephone set has a stand-
alone price of $200.

Under IAS 18 Revenue, the company would have recognised a total of $600 p.a. ($50 x
12 months) but it would not have recongised any revenue in relation to the handset.

However, under IFRS 15, revenue will be allocated both to the telephone set and the
contract. Why? Because the telephone set constitutes a performance obligation.

$ %
Contract (24 months x $50) 1200 85.71
Telephone set 200 14.29
Total value 1400 100

Revenue is included as follows: -

Year 1
Telephone set (14.29% x 1200) $171.48
Contract (1200 – 171.48 = 1028.52/2 years) $514.26

Year 2
Contract $514.26

Sometimes the transaction price may include a discount. Any overall discount should be
allocated between the performance obligations on a relative stand-alone selling price

! 158 acowtancy.com
basis. In some circumstances, it may be appropriate to allocate the discount to some
but not all of the performance obligations.

Lecture Example 360

White Goods Ltd regularly sells washing machines, refrigerators and ovens individually,
thereby establishing the following stand-alone selling prices:

Washing Machine $400


Refrigerator $550
Oven $450
$1400

In addition, it regularly sells refrigerators and ovens together for $600.

White Goods Ltd enters into a contract with a customer to sell all three products in
exchange for $1000. It will satisfy the performance obligations for each of the products
at different points in time.

How should this transaction be accounted for?

Step 5 Recognise revenue when (or as) a performance obligation is


satisfied

An entity shall recognise revenue when (or as) it satisfies a performance obligation, i.e.
when control of a promised good or service is passed to the customer, either over time
or at a point in time.

An entity recognises revenue over time if one of the following criteria is met:

a) The customer simultaneously receives and consumes the benefit provided by the
entity as the entity performs;
b) The entity’s performance creates or enhances an asset that the customer
controls as the asset is created or enhanced; or
c) The entity’s performance does not create an asset with an alternative use to the
entity and the entity has an enforceable right to payment for the performance
completed to date.

60 Adapted from, Example 34, Illustrative examples, IFRS 15

! 159 acowtancy.com
An entity must be able to reasonably measure the outcome of a performance obligation
before the related revenue can be recognised.

17.3 ACCA SYLLABUS GUIDE OUTCOME 2 / 3


Explain and apply the criteria for recognising revenue generated from contracts
where performance obligations are satisfied over time or at a point in time.

Describe the acceptable methods for measuring progress towards complete


satisfaction of a performance obligation.

Performance obligations satisfied over time

In this type of contract, an entity often has a right to payment for performance
completed to date, i.e. an amount that approximates the selling price of the goods or
services transferred to date.

How can we measure the amount of performance completed to date?

1. Output methods: these recognise revenue on the basis of the value of goods or
services transferred to the customer, e.g. surveys of performance completed,
appraisal of units produced etc.

2. Input methods: these recognise revenue on the basis of the entity’s inputs, e.g.
costs incurred, labour hours.

Performance obligations satisfied at a point in time

If a performance obligation is not satisfied over time, it will be satisfied at a point in time,
i.e. at the point in time when the customer obtains control of the asset and the entity
satisfies a performance obligation.

Factors which may indicate that control is passed at a point in time include, but are not
limited to:

1. The entity has a present right to payment for the asset;


2. The customer has legal title to the asset;
3. The entity has transferred physical possession of the asset;
4. The customer has significant risks and rewards related to the ownership of the
asset; and
5. The customer has accepted the asset.

! 160 acowtancy.com
17.4 ACCA SYLLABUS GUIDE OUTCOME 4
Explain and apply the criteria for recognition of contract costs.

Incremental costs of obtaining a contract

If the entity expects to recover incremental costs of obtaining a contract with a


customer, the entity shall recognise those costs as an asset.

What are these incremental costs?


Those costs that an entity would not have incurred if the contract had not been
successfully obtained, for example, a sales commission.

If the amortisation period would be one year or less, then the entity is allowed to
expense these incremental costs.

Costs, that would have been incurred regardless of whether the contract was obtained
or not, are recognised as an expense when incurred.

Illustration 1 61

A consulting services entity, wins a competition bid to provide consulting services to a


new customer. The following costs were incurred by the entity to obtain the contract:

External legal fees for due diligence $15,000


Travel costs to deliver the proposal $25,000
Commissions paid to sales employees $10,000

Total costs incurred $50,000

The entity must recognise an asset for $10,000 commission. Why? It is an incremental
cost of obtaining the contract and the entity expects to recover these costs through
future fees for consulting services.

The legal fees and travel costs are recognised as expenses when incurred. Why?
Because they would have been incurred whether the bid was won or not.

61 Adapted from, Example 36, Illustrative examples, IFRS 15

! 161 acowtancy.com
Costs to fulfil a contract

Costs incurred to fulfil a contract with a customer are recognised as an asset only if all
of the following criteria are met:

a) The costs relate directly to an identifiable contract


b) The costs generate or enhance resources of the entity that will be used in
satisfying performance obligations in the future; and
c) The costs are expected to be recovered.

The asset shall be amortised on a systematic basis that is consistent with the pattern
of transfer of the goods or services to which the asset relates.

Illustration 2 62

An entity enters into a service contract to manage a customer’s information technology


data centre for five years. The contract is renewable for subsequent one-year periods.
The average customer term is seven years. The entity pays an employee a $10,000
sales commission upon customer signing the contract.

Before providing the services, the entity designs and builds a technology platform for
the entity’s internal use that interfaces with the customer’s systems. That platform is not
transferred to the customer, but will be used to deliver services to the customer.
The entity recognises an asset for the $10,000 incremental costs of obtaining the
contract for the sales commission because the entity expects to recover those costs
through future fees for the services to be provided.

The entity amortises the asset over seven years: - the contract term of five years +
anticipation that the contract will be renewed for two subsequent one-year periods.

Illustration 3 63

The initial costs incurred to set up a technology platform are as follows:

Design services 40,000


Hardware 120,000
Software 90,000
Migration and testing of data centre 100,000
Total costs 350,000

62 Adapted from, Example 37, Illustrative examples, IFRS 15


63 Adapted from, Example 37, Illustrative examples, IFRS 15

! 162 acowtancy.com
The initial setup costs relate primarily to activities to fulfil the contract but do not transfer
goods or services to the customer. The entity accounts for the initial setup costs as
follows:

a) Hardware costs – accounted for in accordance with IAS16 Property, plant and
equipment.
b) Software costs – accounted for in accordance with IAS38 intangible assets.
c) Costs of the design, migration and testing of the data centre – we need to
determine whether an asset can be recognised for the costs to fulfil the contract.
Any resulting asset would be amortised on a systematic basis over the seven-
year period.

In addition to the initial costs to set up the technology platform, the entity also assigns
two employees who are primarily responsible for providing the service to the customer.

Although the costs for these two employees are incurred as part of providing the service
to the customer, the entity concludes that the costs do not generate or enhance
resources of the entity. Therefore, the costs cannot be recognised as an asset. They are
recognised as a payroll expense when incurred.

17.5 ACCA SYLLABUS GUIDE OUTCOME 5


Apply the principles of recognition of revenue, and specifically account for the
following types of transaction:
i) principal versus agent
ii) repurchase agreements
iii) bill and hold arrangements
iv) consignments

17.5.1 Warranties

IFRS 15 distinguishes between two types of warranties: -


1. Assurance type (apply IAS 37):
a. An assurance to the customer that the good or service will function as
specified
b. The customer cannot purchase this warranty separately from the entity.

2. Service type (accounted for separately in accordance with IFRS 15):

! 163 acowtancy.com
a. A service is provided in addition to an assurance to the customer that the
good or service will function as specified
b. This applies regardless of whether the customer is able to purchase this
warranty separately from the entity.

In determining the classification of a warranty, an entity considers:


• Legal requirements: (warranties required by law are usually assurance type)
• Length: (the longer the length of coverage, more likely additional services are
being provided)
• Nature of tasks: (do they provide a service or are they related to assurance (e.g.
return shipping for defective goods)).

64
Illustration 4

A manufacturer provides its customer with a warranty with the purchase of a product.
The warranty provides assurance that the product complies with agreed-upon
specifications and will operate as promised for one year from the date of purchase. The
contract also provides the customer with the right to receive up to 20 hours of training
services on how to operate the product at no additional cost.

1. Are the goods and services in the contract distinct?


Yes, both the product and the training services are distinct and give rise to two separate
performance obligations. The customer can benefit from the product on its own without
the training services. The manufacturer regularly sells the product separately without
the training services.

2. How should the warranty be accounted for?

The warranty provides the customer with the assurance that the product will function as
intended for one year. The warranty does not provide the customer with a good or
service in addition to that assurance. Hence, the manufacturer accounts for the
assurance-type warranty in accordance with IAS 37.

Hence, the entity allocates the transaction price to the two performance obligations, the
product and the training services, and recognises revenue when or as those
performance obligations are satisfied.

17.5.2 Principal versus agent

Is the entity acting as principal or agent?

64 Adapted from, Example 44, Illustrative examples, IFRS 15

! 164 acowtancy.com
A principal controls the promised good or service before it is transferred to the customer.
When the performance obligation is satisfied, the entity recognises revenue: the gross
amount of the consideration.

An agent arranges for the provision of goods or services by another party. Revenue will
be in the form of fees or commissions.
The following points indicate that an entity is an agent rather than a principal: -

1. Another party is responsible for fulfilling the contract


2. The entity does not have inventory risk
3. The entity does not have discretion in establishing prices for the other party’s
goods or services
4. The entity’s consideration is in the form of a commission
5. The entity is not exposed to credit risk for the amount receivable from the
customer.

Lecture Example 4 65

PQ Ltd operates a website that enables customers to purchase goods from a range of
suppliers who deliver the goods directly to the customers. When a good is purchased
via the website, PQ is entitled to a commission that is equal to 10% of the sales price.
PQ’s website facilitates payment between the supplier and the customer at prices that
are set by the supplier.

PQ requires payment from customers before orders are processed and all orders are
non-refundable. PQ has no further obligations to the customer after arranging for the
products to be provided to the customer.

Is PQ a principal or an agent?

17.5.3 Repurchase Agreements

A repurchase agreement arises when an entity sells an asset to a customer and


promises, or has the option, to repurchase the asset.

There are 3 forms of repurchase agreements: -

1. Forward contract – an entity has an obligation to repurchase the asset


2. Call option – an entity has the right to repurchase the asset

65 Adapted from, Example 45, Illustrative examples, IFRS 15

! 165 acowtancy.com
3. Put option – an entity must repurchase the asset if requested to do so by the
customer

17.5.4 Forward contract or call option

Here, the customer does not obtain control of the asset, even if it has physical
possession. How should the entity account for this contract?

1. Repurchase price is below the original selling price – record as a lease as per
IAS 17
2. Repurchase price is equal or greater than the original selling price – the entity will
recognise this as a financing arrangement

Illustration 5 – Call Option66

An entity enters into a contract with a customer for the sale of a tangible asset on 1
January 2015 for $1 million

The contract includes a call option that gives the entity the right to repurchase the asset
for $1.1 million on or before 31 December 2015.

Control of the asset does not transfer to the customer on 31 December 2015 because
the entity has a right to repurchase the asset. The customer is limited in its ability to use
the asset. Consequently, the entity accounts for the transaction as a financing
arrangement, because the exercise price ($1.1m) is more than the original selling price
($1m).

The entity does not derecognise the asset and instead recognises the cash received as
a financial liability. The entity also recognises interest expense for the difference
between the exercise price ($1.1 million) and the cash received ($1 million), which
increases the liability

Dr Cash
Cr Loan (SFP)

Every year:
Record interest as finance cost. Increase value of loan

Dr Finance costs (P/L)


Cr Loan (SFP)

66 Adapted from, Example 62 Case A, Illustrative examples, IFRS 15

! 166 acowtancy.com
On 31 December 2015, the option lapses unexercised; therefore, the entity
derecognises the liability and recognises revenue of $1.1 million

17.5.5 Put Option

If the entity must repurchase the asset if requested to do so by the customer, it must
consider whether or not the customer is likely to exercise that option.

1. Repurchase price is lower than the original selling price –


Does the customer have significant economic incentive to exercise the option?

Yes: entity should account for the agreement as a lease as per IAS 17
No: contract accounted for as an outright sale, with a right of return.

2. If the repurchase price is greater than or equal to the original selling price and above
the expected market value of the option – treat as financing arrangement

Illustration 6 – Put Option67

From previous illustration...

Instead of having a call option, the contract includes a put option that obliges the entity
to repurchase the asset at the customer’s request for $900,000 on or before 31
December 2015. The market value is expected to be $750,000 on 31 December 2015.

At the inception of the contract, the entity assesses whether the customer has a
significant economic incentive to exercise the put option. The entity concludes that the
customer has a significant economic incentive to exercise the put option because the
repurchase price significantly exceeds the expected market value of the asset at the
date of repurchase.

The entity concludes that control of the asset does not transfer to the customer,
because the customer is limited in its ability to direct the use of, and obtain substantially
all of the remaining benefits from, the asset.

The entity accounts for the transaction as a lease in accordance with IAS 17 Leases.

17.5.6 Consignment Arrangements

67 Adapted from, Example 62 Case B, Illustrative examples, IFRS 15

! 167 acowtancy.com
An entity may deliver a product to another party, e.g. dealer or retailer. The main
question here is:

Has this party obtained control of the product?

1. No

If this party has not obtained control, the product may be held in a consignment
arrangement.

• No revenue is recognised upon delivery


• Inventory not included in the other party’s SFP
• Any deposit paid included under ‘other receivables’

2. Yes

If control has been transferred to the other party,

• In the other party’s SFP, recognise inventory as an asset, together with a


corresponding liability to the manufacturer
• Deduct any deposit paid from the liability. Any excess is classified as a trade
payable.

The following indicate the existence of a consignment arrangement: -

a) The product is controlled by the entity until a specified event occurs (e.g. sale of
the product to a customer of the dealer or retailer, or until a specified period
expires)
b) The entity can require the return of the product, or transfer it to another party
c) The dealer or retailer does not have an unconditional obligation to pay for the
product

Lecture Example 5 68

Angelino is a motor car dealer selling vehicles to the public. Most of its new vehicles are
supplied on consignment by two manufacturers, Monza and Capri, who trade on
different terms.

68 ACCA Paper 2.5 December 2006 Q3

! 168 acowtancy.com
Monza supplies cars on terms that allow Angelino to display the vehicles for a period of
three months from the date of delivery or when Angelino sells the cars on to a retail
customer if this is less than three months. Within this period Angelino can return the
cars to Monza or can be asked by Monza to transfer the cars to another dealership
(both at no cost to Angelino). Angelino pays the manufacturer’s list price at the end of
the three month period (or at the date of sale if sooner). In recent years Angelino has
returned several cars to Monza that were not selling very well and has also been
required to transfer cars to other dealerships at Monza’s request.

Capri’s terms of supply are that Angelino pays 10% of the manufacturer’s price at the
date of delivery and 1% of the outstanding balance per month as a display charge. After
six months (or sooner if Angelino chooses), Angelino must pay the balance of the
purchase price or return the cars to Capri. If the cars are returned to the manufacturer,
Angelino has to pay for the transportation costs and forfeits the 10% deposit. Because
of this Angelino has only returned vehicles to Capri once in the last three years.

Required:

Describe how the above transactions and events should be treated in the financial
statements of Angelino for the year ended 30 September 2006.

17.5.7 Bill-and-Hold Arrangements

Bill-and-hold arrangements involve the seller invoicing a customer for a product but,
instead of delivering it to the customer, the seller retains physical possession with the
product being delivered to the customer at a later date, e.g. the customer does not have
enough storage space.

When has the customer obtained control of the product?

Is it when the goods have been delivered to the customer? In some circumstances, a
customer may obtain control even though the goods remain in the entity’s possession.

For a customer to have obtained control of a product in a bill-and-hold arrangement, all


these criteria must be met: -

a) The reason for the bill-and-hold arrangement must be substantive (e.g.


requested by the customer due to lack of physical space to store the goods)
b) The product must be identified separately as belonging to the customer

c) The product must currently be ready for physical transfer to the customer

d) The entity cannot have the ability to use the product or to transfer it to another
customer

! 169 acowtancy.com
Illustration 7 69

An entity enters into a contract with a customer on 1 January 2014 for the sale of a
machine and spare parts. The delivery time for the machine and spare parts is two
years.

Upon completion of manufacturing, the entity demonstrates that the machine and spare
parts meet the agreed-upon specifications in the contract. The promises to transfer the
machine and spare parts are distinct and result in two performance obligations that
each will be satisfied at a point in time.

On 31 December 2015, the customer pays for the machine and spare parts, but only
takes physical possession of the machine. Although the customer inspects and accepts
the spare parts, the customer requests that the spare parts be stored at the entity’s
warehouse because of its close proximity to the customer’s factory.

The customer has legal title to the spare parts and the parts can be identified as
belonging to the customer. Furthermore, the entity stores the spare parts in a separate
section of its warehouse and the parts are ready for immediate shipment at the
customer’s request. The entity expects to hold the spare parts for two to four years and
the entity does not have the ability to use the spare parts or direct them to another
customer.

The entity identifies the promise to provide custodial services as a performance


obligation because it is a service provided to the customer and it is distinct from the
machine and spare parts.

1. The entity accounts for three performance obligations in the contract: the
promises to provide: -
• the machine
• the spare parts
• the custodial services

2. The transaction price is allocated to the three performance obligations

3. Revenue is recognised when (or as) control transfers to the customer

• Control of the machine is transferred to the customer on 31 December 2015


when the customer takes physical possession.

69 Adapted from, Example 63, Illustrative examples, IFRS 15

! 170 acowtancy.com
• It recognises revenue for the spare parts on 31 December 2015 when control
transfers to the customer.
• The performance obligation to provide custodial services is satisfied over time as
the services are provided. The entity considers whether the payment terms
include a significant financing component: - interest expense.

17.6 ACCA SYLLABUS GUIDE OUTCOME 6


Prepare financial statement extracts for contracts where performance obligations
are satisfied over time.

17.6.1 Performance obligations satisfied over time

Where performance obligations are satisfied over time, an entity must determine what
amounts to include as revenue and costs in each accounting period. These should be
recorded in profit or loss as the contract activity progresses.

Lecture Example 6

$
Contract Price 1,000
Estimated total costs 800
Costs incurred to date 600
Value of performance obligations satisfied 700
Amount invoiced and paid up 600

The company calculates satisfaction of performance obligations based on work certified


as a percentage of contract price.

Required:-

Calculate the effect of the above contract on the financial statements.

! 171 acowtancy.com
Construction contracts affect the following parts of the statement of profit or loss and
SFP:

Statement of Profit or Loss


Sales x
COS (x)
Profit x

SFP

Contract Asset (presented separately under current assets)

Costs to date x
Profits/(Losses) to date x/(x)
Less amount invoiced (x)
Contract asset (amount due from customer) x

Contract Liability

If the net amount above is negative, that shows a net amount due to the customer.
Hence it is a contract liability, which is included separately under current liabilities.
Receivables (under current assets)

This represents amounts invoiced but not yet paid up.

Important points: -

Remember that

1. IFRS 15 states that an entity’s right to payment for performance completed to date
should approximate the selling price of the service completed to date. The selling price
would be a recovery of costs incurred plus a reasonable profit margin.

2. Where no profit can be estimated, revenue is limited to recoverable costs.

3. Where a loss is anticipated, a proportion of the entity’s costs will not be recovered,
and this needs to be recognised.

! 172 acowtancy.com
Lecture Example 7

$
Contract Price 1,000
Estimated total costs 800
Costs incurred to date 600
Value of performance obligations satisfied 700
Amount invoiced and paid up 600

The company calculates satisfaction of performance obligations based on the proportion


of costs incurred.

Required:-

Calculate the effect of the above contract on the financial statements.

Lecture Example 8

$
Contract Price 1,000
Costs incurred to date 500
Estimated costs to complete the contract 300
Amounts invoiced and paid up 540
Estimated % of obligations satisfied 60%

Required:-
Calculate the effect of the above contract on the financial statements.

! 173 acowtancy.com
Lecture Example 9 - Loss-Making Contract

The first thing you should actually do is check to see if the contract still looks like it is
going to be profitable. If it isn’t you must bring the FULL loss in immediately.

$
Contract Price 1,000
Costs incurred to date 550
Estimated costs to complete the contract 550
Amounts invoiced 475
Amounts paid up 400
Estimated % of obligations satisfied 60%

Required:-

Calculate the effect of the above contract on the financial statements.

Where a contract is already part way through

Where a contract is already part way through, i.e. in its second year, some revenue and
costs have previously been recognised. Therefore, it is important to take this into
account in the calculations to make sure they show the current year revenue and costs.

1) Do contract for year 2 as normal.


2) Take away the Sales and COS figures that were in Yr 1 on the statement of profit
or loss.

! 174 acowtancy.com
FURTHER EXAMPLES70

EXAMPLE 1

Profit-making contract

Lily is a construction company that prepares its financial statements to 31 December


each year.

During the year ended 31 December 2008, the company commenced a contract that is
expected to take more than one year to complete. The contract summary at 31
December 2008 is as follows:

$000
Progress payments 1,400
Contract price 2,736
Work certified complete 1,824
Contract costs incurred to
31 December 2008 2,160
Estimated total cost at
31 December 2008* 2,520

* The examiner sometimes presents information in this manner – ‘estimated total cost’
means costs incurred plus costs to complete.

The company calculates satisfaction of performance obligations based on the work


certified as complete compared to the contract price.

Required:-

Calculate the effect of the above contract on the financial statements at 31


December 2008.

70 Examples 1 and 2 are extracted from the article “Construction Contracts” by B. Retallack, Student Accountant,
November 2008. These have been adapted to IFRS 15.
http://www.accaglobal.com/content/dam/acca/global/PDF-students/2012/sa_novdec08_retallack.pdf

! 175 acowtancy.com
Statement of Profit or Loss extract – 31 December 2008

$000
Revenue (work certified) 1,824
COS (ß) 1,680
Gross profit (W2) 144

Statement of financial position extract – 31 December 2008

Current assets
Asset on a construction contract (W3) 904

WORKINGS:-

(W1) Expected outcome

$000
Contract price 2,736
Total costs 2,520
Expected profit 216

(W2) Percentage of completion

Accounting policy = Work certified complete

Work certified to date


Contract price

1,824 = 66.67%
2,736

(As a round percentage was not found, use the fraction to complete workings instead)

Profit to be recognised = $216 (W1) x $1,824 /$2,736 = $144

(W3) Asset on construction contract

Costs incurred to date 2160


Profit recognised to date 144

! 176 acowtancy.com
Less: Progress payments (1400)
904

EXAMPLE 2

Loss-making contract

Gladioli is a construction company that prepares its financial statements to 31 March


each year. During the year ended 31 March 2008, the company commenced a contract
that is expected to take more than one year to complete.

The contract summary at 31 March 2008 is as follows:

$000
Amounts invoiced and paid up 3,780
Contract price 4,500
Contract costs incurred to 31 March 2008 3,600
Estimated cost to complete at
31 March 2008 1,200

The performance obligation satisfied is calculated using the proportion of costs incurred
method.

Required:-

Calculate the effect of the above contract in the financial statements at 31 March
2008.

Solution 1

Statement of Profit or Loss extract – 31 March 2008

$000
Revenue (0.75 x 4500) 3,375
COS 3,675
Gross loss (W1) (300)

Statement of financial position extract – 31 March 2008

! 177 acowtancy.com
Current liabilities

Liability on a construction contract 480

WORKINGS:-

(W1) Expected outcome

$000
Contract price 4,500
Total cost (3,600 + 1,200) (4,800)
Expected loss (300)

(W2) Liability on construction contract

Costs incurred to date 3,600


Loss recognised to date (300)
Less: Progress payments (3,780)
Liability on construction (480)

(W3) Percentage of completion


Costs to date
Total estimated costs
=

3600 = 75%
4800

! 178 acowtancy.com
CHAPTER 18:

IAS 37 –
PROVISIONS AND CONTINGENCIES

18.1 ACCA SYLLABUS GUIDE OUTCOME 1:-


Explain why an accounting standard on provisions is necessary.

The objective of IAS 37 is to ensure that:

1. appropriate recognition criteria and measurement bases are applied to


provisions, contingent liabilities and contingent assets and
2. sufficient information is disclosed in the notes to the financial statements to
enable users to understand their nature, timing and amount.

18.2 ACCA SYLLABUS GUIDE OUTCOME 2:-


State when provisions may and may not be made and demonstrate how they
should be accounted for.

A provision is a liability of uncertain timing or amount.

IAS 37 requires a provision be recognised when all of the following apply:

1. an entity has a present obligation (legal or constructive) as a result of a past


event
2. it is probable that an outflow of resources embodying economic benefits will be
required to settle the obligation
3. a reliable estimate can be made of the amount of the obligation

Double entry

Dr Expense
Cr Provision (Liability SFP)

If it is part of a cost of an asset (e.g. Decommissioning costs71 )

Dr Asset

71To remove something such as a ship, nuclear power station, machinery, or weapons from service

! 179 acowtancy.com
Cr Provision (Liability SFP)
18.3 ACCA SYLLABUS GUIDE OUTCOME 3:-
Distinguish between legal and constructive obligations.

A legal obligation is an obligation that derives from:


• a contract
• legislation
• other operation of law.

A constructive obligation arises if past practice creates a valid expectation on the part of
a third party, for example, a retail store that has a long-standing policy of allowing
customers to return merchandise within, say, a 30-day period.

18.4 ACCA SYLLABUS GUIDE OUTCOME 4:-


Explain how provisions should be measured72.

The amount recognised as a provision should be the best estimate of the expenditure
required to settle the present obligation at the end of the reporting period.
Provisions for one-off events (restructuring, environmental clean-up, settlement of a
lawsuit) are measured at the most likely amount.
Provisions for large populations of events (product warranties, customer refunds) are
measured at a probability-weighted expected value.
Provisions should be discounted.

Example

A future liability of $1,000 in 2 years time (discount rate 10%)

1,000 x 1/1.10 x 1/1.10 = $826

Dr Expense $826
Cr Provision $826

Then the discount is unwound

72 Examined June 2014 Qs 5ii

! 180 acowtancy.com
Year 1

$826 x 10% = $83

Dr Interest $83
Cr Provision $83

Year 2

(826 + 83) x 10% = $91

Dr Interest $91
Cr Provision $91

! 181 acowtancy.com
18.5 ACCA SYLLABUS GUIDE OUTCOME 5:-
Identify and account for:
i) warranties/guarantees
ii) onerous contracts
iii) environmental and similar provisions
iv) provisions for future repairs or refurbishments

Circumstance Accrue a Provision?


Accrue a provision (past event was the sale of
Warranties/guarantees
defective goods)
Accrue if the established policy is to give
Customer refunds
refunds
Onerous (loss-making) contract Accrue a provision
Accrue a provision if the company's policy is
Land contamination to clean up even if there is no legal
requirement to do so
Future operating losses No provision (no present obligation)
No provision (there is no obligation to provide
Firm offers staff training
the training)
Major overhaul or repairs No provision (no obligation)
Restructuring by sale of an Accrue a provision only after a binding sale
operation/line of business agreement
Accrue a provision only after a detailed formal
Restructuring by closure of
plan is adopted and announced publicly. A
business locations or reorganisation
Board decision is not enough

In the case of a restructuring, it is important to provide only for costs necessarily


entailed by the restructuring.

! 182 acowtancy.com
18.6 ACCA SYLLABUS GUIDE OUTCOME 6:-
Define contingent assets and liabilities and describe their accounting treatment.

Contingent liabilities are:

(a) possible obligations that arise from past events and whose existence will be
confirmed only by the occurrence or nonoccurrence of one or more uncertain
future events not wholly within the control of the entity

(b) present obligations that arise from past events but are not recognised because:
i. they are not probable that an outflow of resources embodying economic
benefits will be required to settle the obligation; or
ii. the amount of the obligation cannot be measured with sufficient reliability

Contingent liabilities should not be recognized in financial statements but they should be
disclosed, unless the possibility of any outflow is remote.

The required disclosures are:


• A brief description of the nature of the contingent liability
• An estimate of its financial effect

! 183 acowtancy.com
Contingent assets are possible assets that arise from past events and whose existence
will be confirmed only by the occurrence or nonoccurrence of one or more uncertain
future events not wholly within the control of the entity.

A contingent asset must not be recognized. Only when the realization of the related
economic benefits is virtually certain should recognition take place. At that point, the
asset is no longer a contingent asset!

Contingent assets must only be disclosed in the notes if they are probable. A brief
description of the contingent asset must be provided together with an estimate of its
financial effect and details of any uncertainties.

Virtually Probable Possible Remote


certain
Liability Create a Contingent -
provision liability
(disclose)
Asset Create an Contingent - -
asset asset
(disclose)

! 184 acowtancy.com
Lecture Example 1

In the post balance sheet period, prior to authorising for issue the financial statements
of Tentacle for the year ended 31 March 2007, the following material information has
arisen.

Tentacle is being sued by an employee who lost a limb in an accident while at work on
15 March 2007. The company is contesting the claim as the employee was not following
the safety procedures that he had been instructed to use. Accordingly the financial
statements include a note of a contingent liability of $500,000 for personal injury
damages. In a recently decided case where a similar injury was sustained, a settlement
figure of $750,000 was awarded by the court. Although the injury was similar, the
circumstances of the accident in the decided case are different from those of Tentacle’s
case.

Required:-

State and quantify how the item above should be treated when finalising the
financial statements of Tentacle for the year ended 31 March 2007.

ACCA Paper 2.5 June 2007 Qs 5b (iii)

Further questions

Question 1

Which of the following is not essential for recognition as a liability :

A. There should be a present obligation to transfer economic benefit


B. The obligating event triggering the obligation should have already happened
C. It should be possible to make a reliable estimate of the amount
D. The obligation to transfer economic benefit should be certain

! 185 acowtancy.com
Question 2

Which of the following statements is correct :

A. When an asset is acquired a provision will need to be made for all repairs needed
during its use
B. There is no need to account for a liability if the amount is not known for certain
C. The amount of audit fees remaining unpaid may be referred to as a Provision
D. A liability with uncertainty regarding eighter its timing or amount may be called a
Provision
Question 3

Which of the following statements is incorrect :

A. A liability is a present obligation, arising from past transaction, which probably has
to be paid
B. If the occurrence of the obligation is in doubt there is no need to account for the
liability
C. Damages awarded by court against the business may be ignored because i twill be
appealed
D. A liability has to be accounted for at the best reliable estimate even if the amount is
not certain

Question 4

Which of the following requirements in IAS 37 are calculated to ensure that Provisions
are not created and used to eighter understate or overstate the performance and net
assets of a business:

i) A provision cannot be made for anticipated future losses


ii) A provision is defined as a liability with uncertainity in timing or amount
iii) A provision cannot be used for any purpose other than what it was intended for
iv) The need for & amount of provision shall be reviewed on every reporting date

A. All four
B. ii, iii and iv
C. i, ii and iii
D. i and ii only

! 186 acowtancy.com
Question 5

A manufacturer received a claim for $200,000 as damages because some products


were not delivered in time. The manufacturer admits the delay but disputes the amount
of the claim pointing out that alternative products cost no more than $28,000. How
should the manufacturer treat the claim when finalising the financial statements for the
year?

A. Account for a liability of $28,000


B. Account for a liability of $200,000
C. Account for the best estimate as liability
D. Ignore the claim

Question 6

On 1 June 2014 AB Printers signed a contract with B plc for $24 million. In terms of this
contract B plc is required to supply 80,000 boxes of paper per year for three years from
1 July 2014. By 31 December, B plc has supplied 32,000 boxes and has sent an invoice
for $8 million. AB Printers has made no payment to B plc. How should AB Printers show
the amount owed to B plc as at 31 December 2014 ?

A. $3.2 million as a liability


B. $2.4 million as a liability
C. $3.2 million as a provision
D. $8 million as a liability

Question 7

P plc is facing a claim for $500,000 from a customer, in respect of a fire due to defective
electric circuit in a kitchen appliance sold by P. The legal opinion is that the claim will
succeed ; but P will be able to re-claim 80% of the amount from F plc who supplied the
malfunctioned circuit-breakers. How should P plc report this on its SFP ?

A. $100,000 as a provision
B. $500,000 as provision and $400,000 as asset
C. $500,000 as a liability and $400,000 as asset
D. $100,000 as a liability

! 187 acowtancy.com
Question 8

A contractor finds that, due to the negligence of his employees, he has to replace the
tiles in a shop. The replacement is expected to cost $5,000; but it is anticipated that the
damaged tiles may fetch around $1,000. At what amount should the contractor account
for his obligation to replace the tiles on his year-end SFP ?

A. $5,000 as a liability and $1,000 as an asset


B. $4,000 as a liability
C. $4,000 as a provision
D. $5,000 as a liability

Question 9

Which of the following statements is correct

A. A contingent liability should always be disclosed in a note


B. A contingent liability may be accounted for if its occurrence is certain
C. A contingent liability should be disclosed as a note, unless its occurrence is remote
D. A contingent liability may always be ignored

Question 10

Which of the following should be disclosed as a note to financial statements as a


contingent liability ?

A. A customer’s claim which, according to legal opinion, has a 75% chance of success
B. A customer’s claim that is unlikely to succeed
C. Damages awarded against a company by court, for wrongful dismissal of an
employee
D. A customer’s claim for injuries suffered on company premises with 5% chance of
success

! 188 acowtancy.com
CHAPTER 19:

IAS 12 –
TAXATION
19.1 ACCA SYLLABUS GUIDE OUTCOME 1:-
Account for current taxation in accordance with relevant accounting standards.

From an accounting point of view, tax is an informed guess made at the year end. By
the time it is actually paid the real figure may be more or less than what we put in our
accounts.

A debit balance on the trial balance for tax means that there has been an under
provision for tax last year and thus needs adding to this year’s current tax.

A credit balance on the trial balance for tax means that there has been an over
provision for tax last year and thus needs reducing from this year’s current tax.

Therefore, each year, the statement of profit or loss will show either an over or under
provision from the previous year as well as the current tax guess for that year.

Lecture Example 1

Flora, a limited liability company, shows an over provision of $3,400 on its tax liability
account at the end of the year ended 31 December 20X8 before accounting for that
year’s tax charge.

It estimates tax on profits for the year to be $67,900.

What amounts should be shown in the financial statements for the year ended 31
December 20X8 in respect of tax?

! 189 acowtancy.com
Lecture Example 2

Sasha has estimated its income tax liability for the year ended 31 December 2009 at
$180,000.

Extract from the trial balance as at 31 December 2009


Dr Cr
$ $
Income tax 3,000

Show the income tax expense for the year ended 31 December 2009 and the
liability for income taxes in the SFP at that date.

19.2 ACCA SYLLABUS GUIDE OUTCOME 2:-


Explain the effect of taxable temporary differences on accounting and taxable
profits.

Deferred Tax Liability

A deferred tax liability is the amount of income tax payable in future periods in respect of
taxable temporary differences. In simple terms, deferred tax is tax that is payable in the
future. This is caused by differences between IFRS rules and Tax rules (tax base), also
known as taxable temporary differences.

Temporary Differences

Temporary differences are defined as being differences between the carrying amount of
an asset (or liability) within the SFP and its tax base, i.e. the amount at which the asset
(or liability) is valued for tax purposes by the relevant tax authorities.

A typical example behind deferred tax in Paper F7 is depreciable non-current assets.

Within financial statements, non-current assets with a limited economic life are subject
to depreciation. However, within tax computations, non-current assets are subject to
capital allowances (also known as tax depreciation) at rates set within the relevant tax
legislation. Where at the year end, the cumulative depreciation charged and the
cumulative capital allowances claimed are different, the carrying value of the asset (cost
less accumulated depreciation) will then be different to its tax base (cost less
accumulated capital allowances) and hence a taxable temporary difference arises.

! 190 acowtancy.com
19.3 ACCA SYLLABUS GUIDE OUTCOME 3:-
Compute and record deferred tax amounts in the financial statements.

Tax Base

Let’s presume in one country’s tax law royalties receivable are only taxed when they are
received.

IFRS

IFRS, on the other hand, recognises them when they are receivable.

Now let’s say in year 1, there are $1,000 royalties receivable but not received until year
2.

The statement of profit or loss would show:

Royalties Receivable $1000


Tax 0 (They are taxed when received in yr 2)

This does not give a faithful representation as we have shown the income but not the
related tax expense.

Therefore, IFRS actually states that matching should occur so this tax needs to be
brought into year 1.

Dr Tax (I/S)
Cr Deferred Tax (SFP provision)

So, basically deferred tax is caused simply by timing differences between IFRS rules
and tax rules.

Therefore IFRS demands that matching should occur i.e.:

Difference Tax effect Deferred Tax Double entry


More Income in I/S More tax needed Liability Dr Tax (I/S)
Cr Def Tax
Liability (SFP)

! 191 acowtancy.com
Hopefully you can see then that the opposite also applies:

Difference Tax effect Deferred Tax Double entry


More expense in I/S Less tax needed Asset Dr Def tax asset
Cr Tax (I/S)

In fact, the following table all applies:

Difference Tax effect Deferred Tax


1 More Income More tax Liability
2 Less income Less tax Asset
3 More expense Less tax Asset
4 Less expense More tax Liability
Remember this “more income etc” is from the point of view of IFRS, i.e.: The accounts
are showing more income, as the taxman does not tax it until next year.

We will now look at each of these 4 cases in more detail.

Case 1

Difference Tax effect Deferred Tax


1 More Income More tax Liability
2 Less income Less tax Asset
3 More expense Less tax Asset
4 Less expense More tax Liability

Issue

IFRS shows more income than the taxman has taken into account.

Example

Royalties receivable above.

! 192 acowtancy.com
Double entry required

Dr Tax (I/S)
Cr Deferred tax Liability (SFP)

Case 2

Difference Tax effect Deferred Tax


1 More Income More tax Liability
2 Less income Less tax Asset
3 More expense Less tax Asset
4 Less expense More tax Liability

Issue

IFRS shows less income than the taxman has taken into account.

Example

Taxman taxes some income which IFRS states that this should be deferred, such as
upfront receipts on a long term contract.

Double entry required

Dr Deferred Tax Asset (SFP)


Cr Tax (I/S)

This will have the effect of eliminating the tax charge for now, so matching the fact that
IFRS is not showing the income yet either.

Once the income is shown, then the tax will also be shown by:

Dr Tax (I/S)
Cr Deferred tax asset (SFP)

! 193 acowtancy.com
Case 3

Difference Tax effect Deferred Tax


1 More Income More tax Liability
2 Less income Less tax Asset
3 More expense Less tax Asset
4 Less expense More tax Liability

Issue

IFRS shows more expense than the taxman has taken into account.

Example

IFRS depreciation is more than Tax depreciation (WDA or CA)

Double entry required

Dr Deferred Tax Asset (SFP) 100


Cr Tax (I/S) 100

Illustration

IFRS TAX

Asset Cost 1,000 1,000


Depn (400) (300)
NBV 600 700

Simply compare 700 – 600 = 100

! 194 acowtancy.com
Case 4

Difference Tax effect Deferred Tax


1 More Income More tax Liability
2 Less income Less tax Asset
3 More expense Less tax Asset
4 Less expense More tax Liability

Issue

IFRS shows less expense than the taxman has taken into account.

Example

IFRS depreciation is less than Tax depreciation (WDA or CA)

Double entry required

Dr Tax I/S 100


Cr Deferred Tax Liability 100

Illustration

IFRS TAX

Asset Cost 1,000 1,000


Depn (300) (400)
NBV 700 600

Simply compare 700 – 600 = 100


NOTE

In actual fact, the standard refers to assets and liabilities rather than more income and
more expense etc.

! 195 acowtancy.com
Simply use the above tables and substitute the word asset for income and expense for
liability.

Difference Tax effect Deferred Tax


1 More Income More tax Liability
2 Less income Less tax Asset
3 More expense Less tax Asset
4 Less expense More tax Liability

Possible Examination examples of Case 1 & 4

• Accelerated capital allowances (accelerated tax depreciation) See above

• Interest revenue
Some interest revenue may be included in profit or loss on an accruals basis, but
taxed when received.

• Revaluations to fair value


Revaluations of NCA are a further example of a taxable temporary difference.
When a NCA is revalued to its current value within the financial statements, the
revaluation surplus is recorded in equity (in a revaluation reserve) and reported
as other comprehensive income. While the carrying value of the asset has
increased, the tax base of the asset remained the same and so a temporary
difference arises.

Tax will become payable on the surplus when the asset is sold and so the
temporary difference is taxable. Since the revaluation surplus has been
recognized within equity, to comply with matching, the tax charge on the surplus
is also charged to equity.

NOTE: Double entry here is

Dr Revaluation Reserve with the tax (as this is where the “income” went)
Cr Deferred tax liability (SFP)

! 196 acowtancy.com
Difference Tax effect Deferred Tax
1 More Income More tax Liability
2 Less income Less tax Asset
3 More expense Less tax Asset
4 Less expense More tax Liability

Possible Examination examples of Case 2 & 3

• Provisions
Provisions may not be deductible for tax purposes until the expenditure is
incurred.

• Losses
Current losses can be carried forward to be offset against future taxable profits.
This will result in a deferred tax asset.

Lecture Example 373

A non-current asset costing $2,000 was acquired at the start of year 1. It is being
depreciated straight line over four years.
The capital allowances granted on this asset are:

$
Year 1 800
Year 2 600
Year 3 360
Year 4 240

Calculate the tax charged to the statement of profit or loss in each of the four
years and the tax liability at the end of each year. Assume the tax rate is 25%.

73 Article,
“Deferred Tax”, S. Baker and T.Clendon, Student Accountant, August 2009
http://www.accaglobal.com/content/dam/acca/global/pdf/sa_aug09_baker_clendon.pdf

! 197 acowtancy.com
Lecture Example 4

Trial Balance (extract) as at 31 March 2012

Dr Cr
$ $
Income tax 120
Deferred tax 9,000

• The estimated income tax on the profits for the year to 31 March 2012 is $11,500
• During the year, $10,300 was paid in full and final settlement of income tax on
the profits for the year ended 31 March 2011. The statement of financial position
at 31 March 2011 had included $10,180 in respect of this liability.
• At 31 March 2012, the carrying amounts of the net assets of Delta exceeded their
tax base by $38,000.
• The rate of income tax in this jurisdiction is 25%.

Calculate the tax charged to the statement of profit or loss and the tax liability at
31 March 2012.

Lecture Example 5

Tadeon trial balance as at 30 September 2006:

Deferred tax balance 1 October 2005 $12,000

The directors have estimated the provision for income tax for the year ended 30
September 2006 at $38 million.

At 30 September 2006 there were $74 million of taxable temporary differences, of which
$20 million related to the revaluation of the leasehold property. The income tax rate is
20%.

Calculate the tax charged to the statement of profit or loss and the tax liability at
30 September 2006.

(Paper 2.5 Dec 2006 Qs 2 Part)

! 198 acowtancy.com
CHAPTER 20:

IAS 10 –
EVENTS AFTER THE REPORTING PERIOD

20.1 ACCA SYLLABUS GUIDE OUTCOME 1:-


Distinguish between and account for adjusting and non-adjusting events after the
reporting date.
Identify items requiring separate disclosure, including their accounting treatment
and required disclosures

“Events after the reporting period” are those events, both favourable and unfavourable,
that occur between the end of the reporting period and the date when the financial
statements are authorised for issue”.
Two types of events can be identified:
a. those that provide evidence of conditions that existed at the end of the reporting
period (adjusting events); and
b. those that are indicative of conditions that arose after the end of the reporting
period (non-adjusting events).

20.1.1 Examples of adjusting events given in IAS 10 are:


a. the resolution of a court case, as the result of which a provision has to be
recognised instead of the disclosure by note of a contingent liability;
b. evidence of impairment of assets e.g. property;
c. bankruptcy of a major customer;
d. sale of inventories at prices suggesting the need to reduce the figure in the SFP
to the net value actually realized (inventories sold at a loss);
e. discovery of fraud or errors that show the financial statements were incorrect.

! 199 acowtancy.com
20.1.2 Examples of non-adjusting events given in IAS 10 are:
a. decline in market value of investments;
b. announcement of a plan to discontinue part of the enterprise;
c. major purchases and sales of assets;
d. destruction of a major asset by fire etc;
e. sale of a major subsidiary;
f. major dealings in the company's ordinary shares;

20.1.3 Further provisions covered by IAS 10:


a. Authorisation for issue of financial statements
An enterprise should disclose the date when the financial statements were authorised
for issue and who gave that authorisation. If the owners or others have the power to
amend the financial statements after issue, that fact should be disclosed.
b. Going concern
If the management decides after the end of the reporting period that it is necessary to
liquidate the enterprise, the financial statements should not be prepared on a going
concern basis.
c. Dividends
If an entity declares dividends after the reporting period, the entity shall not recognise
those dividends as a liability at the end of the reporting period. That is a non-adjusting
event.

! 200 acowtancy.com
Lecture Example 1

In the post balance sheet period, prior to authorising for issue the financial statements
of Tentacle for the year ended 31 March 2007, the following material information has
arisen.

The notification of the bankruptcy of a customer. The balance of the trade receivable
due from the customer at 31 March 2007 was $23,000 and at the date of the notification
it was $25,000. No payment is expected from the bankruptcy proceedings.

Required:-

State and quantify how the item above should be treated when finalising the
financial statements of Tentacle for the year ended 31 March 2007.

ACCA Paper 2.5 June 2007 Qs 5b (i)

Lecture Example 2

Triangle, a public listed company, is in the process of preparing its draft financial
statements for the year to 31 March 2005. The following matters have been brought to
your attention:
On 15 May 2005 the company’s auditors discovered a fraud in the material requisitions
department. A senior member of staff who took up employment with Triangle in August
2004 had been authorising payments for goods that had never been received. The
payments were made to a fictitious company that cannot be traced. The member of staff
was immediately dismissed. Calculations show that the total amount of the fraud to the
date of its discovery was $240,000 of which $210,000 related to the year to 31 March
2005. (Assume the fraud is material).

Required:-

Explain how this item above should be treated in Triangle’s financial statements
for the year to 31 March 2005 in accordance with current international accounting
standards. Your answer should quantify the amounts where possible.

ACCA Paper 2.5 June 2005 Qs 5 (ii)

! 201 acowtancy.com
Lecture Example 3

At 30 September 2003 Bowtock had included in its draft balance sheet inventory of
$250,000 valued at cost. Up to 5 November 2003, Bowtock had sold $100,000 of this
inventory for $150,000. On this date new government legislation (enacted after the year
end) came into force which meant that the unsold inventory could no longer be
marketed and was worthless.

Bowtock is part way through the construction of a housing development. It has prepared
its financial statements to 30 September 2003 in accordance with IAS 11 ‘Construction
Contracts’ and included a proportionate amount of the total estimated profit on this
contract. The same legislation referred to above (in force from 5 November 2003) now
requires modifications to the way the houses within this development have to be built.
The cost of these modifications will be $500,000 and will reduce the estimated total
profit on the contract by that amount, although the contract is still expected to be
profitable.

Required:-

Assuming the amounts are material, state how the information above should be
reflected in the financial statements of Bowtock for the year ended 30 September
2003.

ACCA Paper 2.5 December 2003 Qs 5c (ii)

! 202 acowtancy.com
Further Questions74

Question 1

Which TWO of the following events which occur after the reporting date of a company
but before the financial statements are authorised for issue are classified as
ADJUSTING events in accordance with IAS 10 Events after the Reporting Period?

(i) A change in tax rate announced after the reporting date, but affecting the
current tax liability
(ii) The discovery of a fraud which had occurred during the year
(iii) The determination of the sale proceeds of an item of plant sold before the
year end
(iv) The destruction of a factory by fire

A. (i) and (ii)


B. (i) and (iii)
C. (ii) and (iii)
D. (iii) and (iv)

Question 2

Isaac is a company which buys agricultural produce from wholesale suppliers for retail
to the general public. It is preparing its financial statements for the year ending 30
September 2014 and is considering its closing inventory.

In addition to IAS 2 Inventories, which of the following IFRSs may be relevant to


determining the figure to be included in its financial statements for closing inventories?

A. IAS 10 Events After the Reporting Period


B. IFRS 5 Revenue from contracts with customers
C. IAS 16 Property, Plant and Equipment
D. IAS 41 Agriculture

74 Specimen Exam Applicable from December 2014

! 203 acowtancy.com
Question 3

The events below took place between the reporting date (30.06.14) and the date the
financial statements were authorised for issue(31.08.14). Are the following events
adjusting or non-adjusting events?
i) The rate of exchange of the country of the main supplier has become very
unfavourable

A. Adjusting event
B. Non-adjusting event

ii) Payment was received from a customer whose debt has already been written off

A. Adjusting event
B. Non-adjusting event

iii) In a re-organisation the company has abandoned one of its main production lines

A. Adjusting event
B. Non-adjusting event

iv) Received information of an accident that occurred in May 2014 and destroyed
most of the inventory

A. Adjusting event
B. Non-adjusting event

v) Received claim for compensation from an employee who was wrongly dismissed
in April 2014

A. Adjusting event
B. Non-adjusting event

vi) Company made a right issue

A. Adjusting event
B. Non-adjusting event

! 204 acowtancy.com
vii) Inventory, reported at cost at reporting date, is found to be defective and cannot
be sold

A. Adjusting event
B. Non-adjusting event
viii) The court awarded substantial damages to a customer whose claim was not
expected to succeed.

A. Adjusting event
B. Non-adjusting event

! 205 acowtancy.com
CHAPTER 21:

IFRS 5 –
NON-CURRENT ASSETS HELD FOR SALE AND
DISCONTINUED OPERATIONS

21.1 ACCA SYLLABUS GUIDE OUTCOME 1:-


Discuss the importance of identifying and reporting the results of discontinued
operations.

An item of PPE becomes subject to the provisions of IFRS 5 (rather than IAS 16) if it is
classified as held for sale. This classification can either be made for a single asset
(where the planned disposal of an individual and fairly substantial asset takes place) or
for a group of assets (where the disposal of a business component takes place).

An analysis between continuing and discontinuing operations tends to improve the


usefulness of the financial statements. It is important to know which parts of it are
continuing their operations and those which have ceased or been sold or are about to
be in the near future. Only the results of continuing operations should be used in
forecasting future results; profits or losses from discontinuing operations will not be
repeated.

For example, if a group made a large profit from one of its subsidiaries that it has
recently sold (or will soon sell), this will have a material effect on any forecast of the
group’s future profit. This is because the profits from the subsidiary disposed of will no
longer contribute to future group profit. Also, the converse would be true where the
disposal or closure of a loss-making subsidiary could improve future profitability.75

Information on discontinued operations can also help to assess management’s strategy.


One would expect loss-making activities to be sold or closed down, but selling a
profitable activity may indicate that a company has liquidity or debt problems.

75 June 2013 Qs 4a

! 206 acowtancy.com
21.2 ACCA SYLLABUS GUIDE OUTCOME 2:-
Define and account for non-current assets held for sale.

21.2.1 When is an asset held for sale?

An asset is classified as held for sale if its carrying amount will be recovered principally
through a sale transaction rather than through continued use.

1. management is committed to a plan to sell


2. the asset is available for immediate sale in its present condition
3. an active programme to locate a buyer is initiated
4. the sale is highly probable, within 12 months of classification as held for sale
5. the asset is being actively marketed for sale at a sales price reasonable in
relation to its fair value

21.2.2 Measurement

Immediately before the initial classification

The carrying amount of the asset will be measured in accordance with applicable
IFRSs. Generally, bring depreciation up to date (if cost model followed) or revalue (if
revaluation policy followed).

After classification as held for sale

When non-current assets or disposal groups are classified as held-for-sale, they are
measured at the lower of the carrying amount and fair value less costs to sell.

If fair value less costs to sell is below the current carrying value, then the asset is written
down to fair value less costs to sell and an impairment loss recognized. Any impairment
loss that arises by using the measurement principles in IFRS 5 must be recognised in
profit or loss, even for assets previously carried at revalued amounts.

Revalued assets will need to deduct costs to sell from their fair value and this will result
in an immediate charge to profit or loss.

! 207 acowtancy.com
Subsequent increases in fair value

A gain is recognised in the statement of profit or loss up to the amount of all previous
impairment losses.
No depreciation

Non-current assets or disposal groups that are classified as held for sale shall not be
depreciated as its carrying value will be recovered principally through sale rather than
continuing use.

Presentation on the Statement of Financial Position

When an asset is classified as held for sale, IFRS 5 requires that it be moved from its
existing presentation on the SFP (non-current assets) to a new category (under current
assets) of the SFP – ‘non-current assets held for sale’.

If the criteria for classifying a non-current asset as held-for-sale occur after the end of
the reporting period, then the non-current asset should not be shown as held-for-sale
but disclosure of the fact in the notes should be made.

When a held for sale asset is sold

When the asset is sold, any difference between the new carrying value and the net
selling price is shown as a profit or loss on sale.

Change of plans

If criteria for an asset to be classified as held-for-sale are no longer met, then the asset
or disposal group ceases to be held-for-sale.

In this case, it should be valued at the lower of the carrying amount before the
asset or disposal group was classified as held-for-sale (as adjusted for any
subsequent depreciation, amortisation or re-valuation), and its recoverable amount at
the date of the decision not to sell. Any adjustment to the value should be shown in
income from continuing operations for the period.

! 208 acowtancy.com
Lecture Example 176

An asset has a carrying value of $600,000. It is classified as held for sale on 30


September 20X6. At that date its fair value less costs to sell is estimated at $550,000.
The asset was sold for $555,000 on 30 November 20X6. The year end of the entity is
31 December 20X6.
1. How would the classification as held for sale, and subsequent disposal, be
treated in the 20X6 financial statements?
2. How would the answer differ if the carrying value of the asset at 30 September
20X6 was $500,000, with all other figures remaining the same?

Lecture Example 277

An asset being classified as held for sale is currently carried under the revaluation
model at $600,000. Its latest fair value is $700,000 and the estimated costs of selling
the asset are $10,000.

Show how this transaction would be recorded in the financial statements.

21.2.3 Subsidiaries Held for Disposal

IFRS 5 applies to accounting for an investment in a subsidiary held only with a view to
its subsequent disposal in the near future.

Subsidiaries already consolidated now held for sale

The parent must continue to consolidate such a subsidiary until it is actually disposed
of. It is not excluded from consolidation and reported as an asset held for sale
under IFRS 5.

76 Article,
“Property, Plant and Equipment and Tangible Assets”, by P. Robins, Student Accountant, August 2007
http://www.accaglobal.com/content/dam/acca/global/pdf/sa_aug07_robins.pdf
77 Article,
“Property, Plant and Equipment and Tangible Assets”, by P. Robins, Student Accountant, August 2007
http://www.accaglobal.com/content/dam/acca/global/pdf/sa_aug07_robins.pdf

! 209 acowtancy.com
21.3 ACCA SYLLABUS GUIDE OUTCOME 3:-
Define and account for discontinued operations.

Classification

A discontinued operation is a component of an entity that either has been disposed of or


is classified as held for sale, and:

1. represents a separate major line of business or geographical area of operations,


2. is part of a single co-ordinated plan to dispose of a separate major line of
business or geographical area of operations, or
3. is a subsidiary acquired exclusively with a view to resale and the disposal
involves loss of control.

Statement of Profit or Loss and other Comprehensive Income Presentation

The profit after tax of the discontinued operation and the post-tax gain or loss
recognised on the measurement to fair value less cost to sell or fair value adjustments
on the disposal of the assets (or disposal group) should be presented as a single
amount on the face of the statement of profit and loss and other comprehensive
income.

Detailed disclosure of revenue, expenses, pre-tax profit or loss, and related income
taxes is required either in the notes or on the face of the statement of profit or loss in a
section distinct from continuing operations.

Statement of Cash Flow Presentation

The net cash flows attributable to the operating, investing, and financing activities of a
discontinued operation shall be separately presented on the face of the statement of
cash flow or disclosed in the notes.

No Retroactive Classification

IFRS 5 prohibits the retroactive classification as a discontinued operation, when the


discontinued criteria are met after the end of the reporting period.

! 210 acowtancy.com
Illustration

An entity plans to dispose of a group of net assets which form a disposal group. The net
assets at 31 December 2006 are set out below.

Carrying value at 31
Dec 2006 $m
Goodwill 16
Property, plant and
28
equipment
Inventory 20
Financial assets (profit of
$4m recognised in 17
equity)
Financial liabilities (14)
67

The property, plant and equipment and inventory were stated at deemed cost on moving
to IFRS. Under IFRS, property, plant and equipment would be stated at $26m, and
inventory stated at $18m. The fair value less costs to sell of the disposal group is $47m.
Assume that the disposal group qualifies as held-for-sale.

Show how the disposal group would be accounted for in the financial statements for the
year ended 31 December 2006.

Answer

Fair value less


Carrying value Re- measured Impairment
costs to sell
$m $m $m
$m
Goodwill 16 16 (16) –
Property, plant
28 26 26
and equipment
Inventory 20 18 18
Financial
17 17 17
assets

211
! acowtancy.com
Financial
(14) (14) (14)
liabilities
67 63 (16) 47

IFRS 5 requires that immediately before the initial classification of the disposal group as
held-for-sale, the carrying amounts of the disposal group be measured in accordance
with applicable IFRS, and any profit or loss dealt with under that IFRS. The reduction in
the carrying amount of property, plant and equipment will be dealt with in accordance
with IAS 16, and that of the inventory in accordance with IAS 2.

After the re-measurement, the entity will recognise an impairment loss of $16m on re-
measurement to the lower of carrying amount and fair value less cost to sell. This loss is
allocated to goodwill in accordance with IAS 3678. Thus, goodwill will be reduced to
zero. The loss will be charged against profit or loss.

In the SFP, the major classes of assets and liabilities classified as held-for-sale should
be separately disclosed on the face of the SFP or in the notes. Thus, in this case, there
would be separate disclosure of the disposal group as follows.

$m
Assets
Non-current assets
Current assets
Non-current assets and current assets

61
classified as held-for-sale (note)

Equity and liabilities


Equity attributable to parent
Amounts recognised directly in equity relating to non-current
assets held-for-sale

78 The impairment loss is allocated in the following order:

• first, reduce the carrying amount of any goodwill allocated to the cash-generating unit (group of units); and
• then, reduce the carrying amounts of the other assets of the unit (group of units) pro rata on the basis.

! 212 acowtancy.com
Minority interest
Total equity

Non-current liabilities
Current liabilities
Liabilities directory associated with non-

14
current assets classified as held-for-sale
Total liabilities

Total equity and liabilities

! 213 acowtancy.com
Lecture Example 3
Partway is in the process of preparing its financial statements for the year ended 31
October 2006. The company’s main activity is in the travel industry mainly selling
package holidays (flights and accommodation) to the general public through the Internet
and retail travel agencies.

During the current year the number of holidays sold by travel agencies declined
dramatically and the directors decided at a board meeting on 15 October 2006 to cease
marketing holidays through its chain of travel agents and sell off the related high-street
premises. Immediately after the meeting the travel agencies’ staff and suppliers were
notified of the situation and an announcement was made in the press.

The directors wish to show the travel agencies’ results as a discontinued operation in
the financial statements to 31 October 2006. Due to the declining business of the travel
agents, on 1 August 2006 (three months before the year end) Partway expanded its
internet operations to offer car hire facilities to purchasers of its internet holidays.

The following are Partway’s summarised Statement of Profit or Loss results – years
ended:
31 Oct 2006 31 Oct 2005
Internet Travel Agencies Car Hire Total Total
$’000 $’000 $’000 $’000 $’000

Revenue 23,000 14,000 2,000 39,000 40,000


Cost of sales (18,000) (16,500) (1,500) (36,000) (32,000)
Gross profit/(loss) 5,000 (2,500) 500 3,000 8,000
Operating expenses (1,000) (1,500) (100) (2,600) (2,000)
Profit/(loss) before tax 4,000_ (4,000) 400 400_ 6,000_

The results for the travel agencies for the year ended 31 October 2005 were: revenue
$18 million, cost of sales $15 million and operating expenses of $1.5 million.

Required:-
1. Discuss whether the directors’ wish to show the travel agencies’ results as
a discontinued operation is justifiable.

2. Assuming the closure of the travel agencies is a discontinued operation,


prepare the (summarised) statement of profit or loss of Partway for the year
ended 31 October 2006 together with its comparatives.

Note: Partway discloses the analysis of its discontinued operations on the face of its
statement of profit or loss.

(ACCA Paper 2.5 December 2006 Qs 5b)

! 214 acowtancy.com
CHAPTER 22:

PREPARING THE ACCOUNTS OF A SINGLE


ENTITY: - HOW TO ANSWER THIS QUESTION
This question will ask you to prepare a set of accounts (normally a Statement of Profit or
Loss and Other Comprehensive Income, Statement of Financial Position (SFP) and
statement of changes in equity (SOCIE)).

The examiner normally gives you a trial balance and some adjustments to make, which
you then do and prepare the accounts.

Otherwise, he will give you a set of accounts already prepared and just give you a list of
adjustments to make to these.

The most common is the trial balance.

Step 1

Take the trial balance figures and put them immediately into a statement of profit or loss
and other comprehensive income and SFP.

Leave lots of space for adjustments.

Leave the SOCIE until later.

Step 2

Do the following common adjustments:

• Taxation
• Interest
• Dividends
• Simple depreciation

! 215 acowtancy.com
22.1 Current Tax

As discussed in the chapter “Taxation”, tax, from an accounting point of view, is an


informed guess made at the year end. By the time it is actually paid the real figure may
be more or less than what we put in our accounts.
Therefore, each year, the statement of profit or loss will show either an over or under
provision from the previous year as well as the current tax guess for that year.

In the trial balance given to you if there is an amount for taxation given – this will be the
under or over provision from last year.

If it is a debit it is an under provision (interest expense last year was lower than it should
have been) and so needs adding to this year’s current tax.

If it is a credit it is an over provision (interest expense last year was higher than it should
have been) and so wants reducing from this year’s current tax.

Illustration

Trial Balance (extract) Dr Cr

Tax 10

Notes to the question:

1) Current tax for the year is 100

Answer:

Step 1 TB into accounts

I/S Tax (+10

Step 2 Do adjustment

I/S Tax (+10 + 100)


SFP Tax payable 100

If the figure in the trial balance was a credit – then that means it was an over provision
in the previous year and so we need to take this away from this year’s figure in the
statement of comprehensive income.

! 216 acowtancy.com
The answer would be:

I/S Tax (-10 + 100) = 90


SFP Tax Payable 100

22.2 Deferred Tax

The trial balance will show a deferred tax balance – this either goes to assets (if dr) or
liabilities (if cr) on the SFP.

All you need to do now is account for the movement between this opening balance
and the closing balance given to you as an adjustment.

Illustration

Trial Balance (extract) Dr Cr

Deferred tax 80

Notes to the question:

1) Deferred tax is to be 100

Answer:

Step 1 TB into accounts

SFP Deferred tax Liability 80

Step 2 Do adjustment

I/S Tax (+20)


SFP Deferred Tax Liability (80 + 20) 100

Deferred tax is calculated as:

Tax rate x Timing differences

! 217 acowtancy.com
Illustration

There are $1,000 timing differences caused by accelerated capital allowances. Tax rate
30%.

Solution

Deferred Tax is to be 300

NOTE

Any deferred tax caused by a timing difference on a revaluation – the deferred tax goes
to the SFP as normal but the other side is NOT to the statement of profit or loss. It
reduces the revaluation surplus instead.

Illustration

Trial Balance (extract) Dr Cr

Deferred tax 80
Revaluation reserve 300

Notes to the question:

1) The revaluation caused a timing difference of 300 (Tax 30%).

Answer:

Step 1 TB into accounts

SFP Deferred tax Liability 80


Revaluation Reserve 300

Step 2 Do adjustment

SFP Deferred Tax Liability (80 + 10) 90


Revaluation reserve (300 – 10) 290

! 218 acowtancy.com
N.B.: Deferred tax = 300 x 30% = 90 (b/f 80) so movement is 10.

Comprehensive example

Trial Balance (extract) Dr Cr

Tax 10
Deferred tax 70
Revaluation reserve 300

Notes to the question:

1) The revaluation caused a timing difference of 300 (Tax 30%).


2) Tax for year is estimated to be 200.

Answer:

Step 1 TB into accounts

I/S Tax (+10


SFP Deferred tax Liability 70
Revaluation Reserve 300

Step 2 Do adjustment

I/S Tax (+10 + 200) 210


SFP Tax Payable 200
Deferred Tax Liability (70 + 20) 90
Revaluation reserve (300 – 20) 280

22.3 Interest Adjustment

Issue

Simply ensure the correct amount payable is in the accounts regardless of what has
been paid.

An 8% 1,000 Loan – should show $80 interest in I/S


An 8% 1,000 Loan for 9 months = $80 x 9/12 in I/S

! 219 acowtancy.com
Illustration

Trial Balance (extract) Dr Cr

Interest 120
8% Loan 2,000

Answer:

Step 1 TB into accounts

I/S Interest (120


SFP 8% Loan 2,000

Step 2 Do adjustment

I/S Interest (120 + 40) = 160


SFP Interest Accrual +40
8% Loan 2,000

Note Interest = 2,000 x 8% = 160

NOTE – Effective Interest Rates

If you are given a loan with an effective interest rate and a paid (coupon) interest rate –
always use the effective rate to calculate the interest expense.

The interest due this time, though, does not go to accruals but gets added onto the
loan.

Illustration

Trial Balance (extract) Dr Cr

Interest 120
2% Loan 2,000

Note to Question

! 220 acowtancy.com
The loan has an effective interest rate of 10%.

Answer:

Step 1 TB into accounts

I/S Interest (120


SFP 2% Loan 2,000

Step 2 Do adjustment

I/S Interest (120 + 80) = 200 (10% of $2000)


SFP 2% Loan 2,000 + 80 = 2,080

22.4 Dividend Adjustment

Double Entry

Dr Retained Earnings Cr Dividend Payable

Presentation

In the Statement of Changes in Equity

Share Capital Share Premium Retained Earnings


B/f X X X
Issue of shares X X
Profit after tax X
Dividends (X)
C/f X x X

Sometimes in the question, the double entry is not needed as the examiner will tell you
that is already accounted for correctly. Therefore, you simply have to show it in the
SOCIE.

Calculation

Number of shares x Dividend per share

! 221 acowtancy.com
Illustration

Trial Balance (extract) Dr Cr

Share Capital (@ $1) 2,000


Share Premium 500

Note to Question

A dividend of 0.10 per share. This has been accounted for correctly.

Answer

Number of shares x Dividend per share


? x 0.10

The number of shares is calculated by taking the share capital figure and dividing it by
the nominal value – given in brackets after the share capital on the TB.

2,000 / 1 x 0.1 = 200

This is shown in the SOCIE

Share Capital Share Premium Retained Earnings


B/f X X X
Issue of shares X X
Profit after tax X
Dividends -200
C/f X X X

! 222 acowtancy.com
Illustration

Trial Balance (extract) Dr Cr

Share Capital (@ $0.5) 2,000


Share Premium 500

Note to Question

A dividend of 0.10 per share. This has been accounted for correctly.

Answer

2,000 / 0.5 x 0.1 = 400

! 223 acowtancy.com
22.5 The Financial Statements

One of the statements introduced by IAS 1 (revised) is the statement of profit or loss
and other comprehensive income. This statement presents all items of income and
expense recognized in profit or loss together with all other items recognized in income
and expense. Entities may present all items together in a single statement or present
two linked statements – one displaying the items of income and expense recognised in
the statement of profit or loss and the other statement beginning with profit or loss and
displaying all the items included in ‘other comprehensive income’.

Therefore, whereas the statement of profit or loss includes all realised gains and losses
(e.g. net profit for the year), the statement of profit or loss and other comprehensive
income would include both the realised and unrealised gains and losses (e.g.
revaluation surplus).

Proforma 1: One single statement

Statement of Profit or Loss and Other Comprehensive Income for the year ended
31 March 20X8

20X8 20X7
$’000 $’000
Revenue X X
Cost of sales (X) (X)
Gross profit X X
Other income X X
Distribution costs (X) (X)
Administrative expenses (X) (X)
Finance costs (X) (X)
Investment income X X
Profit before tax X X
Income tax expense (X) (X)
Profit for the year X X
Other comprehensive income:
Gains on property revaluation X X
Total comprehensive income for the year X X

! 224 acowtancy.com
Proforma 2: Two separate statements

Statement of Profit or Loss for the year ended 31 March 20X8

20X8 20X7
$’000 $’000
Revenue X X
Cost of sales (X) (X)
Gross profit X X
Other income X X
Distribution costs (X) (X)
Administrative expenses (X) (X)
Finance costs (X) (X)
Investment income X X
Profit before tax X X
Income tax expense (X) (X)
Profit for the year X X

Statement of comprehensive income for the year ended 31 March 20X8

20X8 20X7
$’000 $’000
Profit for the year X X
Other comprehensive income:
Gains on property revaluation X X
Income tax relating to components of other
comprehensive income X X
Total comprehensive income for the year X X

! 225 acowtancy.com
Statement of financial position as at 31 March 20X8

$'000
ASSETS
Non-current assets
Property, plant and equipment X
Other intangible assets X
X
Current assets
Inventories X
Trade receivables X
Other current assets X
Cash and cash equivalents X
X
Total assets X
EQUITY AND LIABILITIES
Equity
Share capital X
Share premium account X
Revaluation reserve X
Retained earnings X
X
Non-current liabilities
Long term borrowings X
Long term provisions X
Current liabilities
Trade payables X
Short term borrowings X
Current tax payable X
Short term provisions X
Total equity and liabilities X

! 226 acowtancy.com
Statement of changes in equity – Proforma

Share Share Revaluation Retained Total


Capital Premium Reserve Earnings
’000 ’000 ’000 ’000 ’000
Balance at 31 March 20X7 X X X X X
Changes in accounting X X
policy
Restated balance X X X X X
Issue of share capital X X X
Dividends (X) (X)
Total comprehensive X X X
income
Balance at 31 March 20X8 X X X X X

! 227 acowtancy.com
22.6 ACCA SYLLABUS GUIDE OUTCOME 1:-
Indicate the circumstances where separate disclosure of material items of
income and expense is required

The term ‘exceptional items’ refers to material items of income and expense of such
size, nature or incidence that disclosure is necessary in order to explain the
performance of the entity.

The accounting treatment is to:


a. Include the item in the standard line in the statement of profit or loss
b. Disclose the nature and amount in notes.

In some cases it may be more appropriate to show the item separately on the face of
the statement of profit or loss.

Examples include:
• include the item in the standard line in the statement of profit or loss
• disclose the nature and amount in notes.
• write down of inventories to net realisable value (NRV)
• write down of property, plant and equipment to recoverable amount
• restructuring
• gains/losses on disposal of non-current assets
• discontinued operations
• litigation settlements
• reversals of provisions.

! 228 acowtancy.com
CHAPTER 23:

IAS 33 – EARNINGS PER SHARE

The objective of IAS 33 is to prescribe principles for determining and presenting


earnings per share (EPS) amounts to improve performance comparisons between
different entities in the same reporting period and between different reporting periods for
the same entity.

23.1 ACCA SYLLABUS GUIDE OUTCOME 1:-


Calculate the EPS in accordance with relevant accounting standards (dealing
with bonus issues, full market value issues and rights issues).

Basic earnings per share


Calculation

Profits After Tax - Preference dividends


Weighted average number of ordinary shares

The earnings numerator should be after deducting all expenses including taxes and
preference dividends.
The denominator (number of shares) is calculated by adjusting the shares in issue at
the beginning of the period by the number of shares bought back or issued during the
period, multiplied by a time-weighting factor.

! 229 acowtancy.com
Example - Weighted average number of shares calculation

1/1/2009 100 shares


1/4/2009 Full market price issue of 200 shares
1/7/2009 1 for 3 bonus issue 


Date Total Time Bonus Weighted


Shares Apportionment Factor Average
1/1 100 3/12 4/3 33
1/4 300 3/12 4/3 100
1/7 400 6/12 200
333

23.1.1 Bonus Issue

In a bonus issue of shares, ordinary shares are issued to existing shareholders for no
additional consideration. In these circumstances, the number of ordinary shares
outstanding before the issue is adjusted for the proportionate change in the number of
shares outstanding as if the event had occurred at the beginning of the earliest period
reported.

Bonus Factor Calculation

1 for 2 – now got 3 used to have 2 = 3/2


2 for 5 – now got 7 used to have 5 = 7/5
3 for 4 – now got 7 used to have 4 = 7/4

To compare to last year

(Remember this can only be used for comparison year on year NOT between
companies)

To compare, we must include the bonus factor into last year’s EPS:

Last year’s EPS x Inverse of bonus factor

! 230 acowtancy.com
Lecture Example 1

The issued share capital of Savoir, a publicly listed company, at 31 March 2003 was $10
million. Its shares are denominated at 25 cents each. Savior’s earnings attributable to its
ordinary shareholders for the year ended 31 March 2003 were also $10 million, giving
earnings per share of 25 cents.
Year ended 31 March 2004
On 1 July 2003 Savoir issued eight million ordinary shares at full market value. On 1
January 2004 a bonus issue of one new ordinary share for every four ordinary shares
held was made. Earnings attributable to ordinary shareholders for the year ended 31
March 2004 were $13,800,000.

Required:-

Calculate Savoir’s earnings per share for the year ended 31 March 2004 including
comparative figures.

23.1.2 Rights Issue

A rights issue often gives rise to a bonus element since the exercise price is often less
than the fair value of the shares. The number of ordinary shares to be used in
calculating basic EPS for all relevant periods is the number of ordinary shares
outstanding before the issue, multiplied by the factor:

Fair value (MV) of share immediately before the exercise of rights 



Theoretical ex-rights fair value of share

Bonus Factor Calculation

Example:-

Rights issue 2 for 3 @ $4 (Market Value $5)

Being offered 2 @ $4 = $ 8
For every 3 @ $5 = $15
So now have 5 @ $23

Theoretical ex-rights price=

$23 / 5 = $4.6

MV is $5
Bonus factor is 5/4.6

! 231 acowtancy.com
To compare to last year

To compare, we must include the bonus factor into last year’s EPS (same as we have
done in the case of the bonus issue):

Last year’s EPS x Inverse of bonus factor


Lecture Example 2

Following from lecture example 1

Year ended 31 March 2005


On 1 October 2004 Savoir made a rights issue of shares of two new ordinary shares at
a price of $1.00 each for every five ordinary shares held. The offer was fully subscribed.
The market price of Savoir’s ordinary shares immediately prior to the offer was $2.40
each. Earnings attributable to ordinary shareholders for the year ended 31 March 2005
were $19,500,000.

Required:-

Calculate Savoir’s earnings per share for the years ended 31 March 2005
including comparative figures.

23.2 ACCA SYLLABUS GUIDE OUTCOME 2:-


Explain the relevance of the diluted EPS and calculate the diluted EPS involving
convertible debt and share options (warrants).

From time to time an entity may enter into commitments to issue shares in the future
that would result in a change in the basic EPS. These are referred to as potential
ordinary shares. Such commitments may arise from, for example:

❖ the issue of debt instruments (e.g. loan stock) that are convertible into ordinary
shares;
❖ share options and share warrants;

Where potential ordinary shares are actually issued the basic EPS will be affected due
to two factors:

❖ the number of shares in issue will increase;


❖ the profits available to the shareholders may increase — for example, due to the
saving of interest on convertible debt instruments.

The diluted EPS tries to show the possible effect of this on the current EPS.

! 232 acowtancy.com
Earnings Shares

As reported
In EPS e.g. 100 e.g. 50

Convertible
Loan + interest saved + convertible shares

Options NIL + ‘bonus’ shares

Illustration:-

Earnings $100
Weighted average no. of shares 50

5% convertible loan $800 – each $100 can be converted into 20 ordinary shares (tax
30%)

100 share options @ $2 (their MV is $5)

Step 1: How to calculate Interest Saved

5% x $800 = $40 x 70% (tax adjusted) = $28 is the amount of interest saved when the
loan is converted into ordinary shares

Step 2: How to calculate the extra convertible shares

$800 x 20 = 160 shares


$100

Step 3: How to calculate the free shares in share options

Cash in from option $200 (100 x $2)


This would mean the company issuing (200/5) 40 shares at the MV of $5 instead of the
100 shares.

Therefore, there have effectively been 60 shares issued for ‘free’. We use this figure in
the diluted EPS calculation.

! 233 acowtancy.com
An alternative calculation is:

100 x (5-2) / 5 = 60

Earnings Shares Effect per share

As reported
In EPS 100 50 $2
Options ---- +60
100 110 $0.91 - dilutive
Convertible Loan +28 + 160
128 270 $0.48 - dilutive

Therefore, DEPS are $0.48

If the effect per share is anti-dilutive (i.e. an increase in earnings per share or a
reduction in loss per share resulting from the assumption that convertible instruments
are converted or that options or warrants are exercised), this should be ignored in
calculated the diluted EPS.

Lecture Example 3

Following from lecture examples 1 and 2

On 1 April 2005 Savoir issued $20 million 8% convertible loan stock at par. The terms of
conversion (on 1 April 2008) are that for every $100 of loan stock, 50 ordinary shares
will be issued at the option of loan stockholders. Alternatively the loan stock will be
redeemed at par for cash. Also on 1 April 2005 the directors of Savoir were awarded
share options on 12 million ordinary shares exercisable from 1 April 2008 at $1.50 per
share. The average market value of Savoir’s ordinary shares for the year ended 31
March 2006 was $2.50 each. The income tax rate is 25%. Earnings attributable to
ordinary shareholders for the year ended 31 March 2006 were $25,200,000. The share
options have been correctly recorded in the statement of profit or loss.

Required:-

Calculate Savoir’s basic and diluted earnings per share for the year ended 31
March 2006 (comparative figures are not required).

You may assume that both the convertible loan stock and the directors’ options are
dilutive.

(ACCA Paper 2.5 June 2006 Question 5b)

! 234 acowtancy.com
23.3 ACCA SYLLABUS GUIDE OUTCOME 3:-
Explain why the trend of EPS may be a more accurate indicator of performance
than a company’s profit trend and the importance of EPS as a stock market
indicator .

Whilst profit after tax is a useful measure, it may not give a fair representation of the
true underlying earnings performance. Users could interpret the large annual increase
in profit after tax as being indicative of an underlying improvement in profitability, but it’s
really an increase in absolute profit.

It is possible, even probable, that some of the profit growth is achieved through the
acquisitions of other companies (acquisitive growth). Where companies are acquired
from the proceeds of a new issue of shares, or where they have been acquired through
share exchanges, this will result in a greater number of equity shares of the acquiring
company being in issue.

This explains why the EPS and the trend of EPS is considered a more reliable indicator
of performance because the additional profits which could be expected from the greater
resources (proceeds from the shares issued) is matched with the increase in the
number of shares. Simply looking at the growth in a company’s profit after tax does not
take into account any increases in the resources used to earn them.

The diluted EPS is useful as it alerts existing shareholders to the fact that future EPS
may be reduced as a result of share capital changes. Where the finance cost per
potential new share is less than the basic EPS, there will be a dilution79 (as shown in the
example above).

Further Questions

Question 180

On 1 October 2013, Hoy had $2·5 million of equity shares of 50 cents each in issue.

No new shares were issued during the year ended 30 September 2014, but on that date
there were outstanding share options to purchase 2 million equity shares at $1·20 each.
The average market value of Hoy’s equity shares during the year ended 30 September
2014 was $3 per share.

79 ACCA F7 Exam December 2009 Qs 5a


80 Specimen Exam Applicable from December 2014

! 235 acowtancy.com
Hoy’s profit after tax for the year ended 30 September 2014 was $1,550,000.

In accordance with IAS 33 Earnings per Share, what is Hoy’s diluted earnings per share
for the year ended 30 September 2014?

A. 25·0 cents
B. 22·1 cents
C. 31·0 cents
D. 41·9 cents

Question 2

On 1 January 2010, the issued share capital of Dan Ltd was 1 million ordinary shares of
$1 each. The income tax rate is 30%. The earnings for the year ended 31 December
were $595,000.

Calculate the EPS separately in respect of the year ended 31 December 2010 for
each of the following circumstances: -

a. There was no change in the issued share capital of the company during the year
ended 31 December
b. The company made a bonus issue on 1 October 2010 of one ordinary share for
every four shares in issue at 30 September 2010
c. The company issued 1 share for every 10 on 1 August 2010 at full market value
of $4
d. The company made a rights issue of $1 ordinary shares on 1 October 2010 in the
proportion of 1 of every 3 shares held, at a price of $3. The middle market price
for the shares on the last day of quotation cum rights was $4 per share.
e. The company made no new issue of shares during the year ended 31 December
2010, but on that date it had in issue $260,000 10% convertible bonds. Every
$100 of these bonds will be convertible into 90 ordinary shares of $1 each.
f. The company made no issue of shares during the year ended 31 December
2010, but on that date there were outstanding options to purchase 7,400 ordinary
$1 shares at $2.50 per share. Share price during the year was $4.

! 236 acowtancy.com
CHAPTER 24:

Analysing and Interpreting Financial Statements

24.1 ACCA SYLLABUS GUIDE OUTCOME 1:-


Indicate the problems of using historic information to predict future performance
and trends.

Ratios are a tool which assists in the analysis of financial information. They summarise
information and assist in identifying trends. However, ratios are not predictive if they are
based on historical information.

1) Financial data is time sensitive and past trends may not continue
2) The company will probably have changed strategy and so the market will have
different expectations
3) There is also no dealing with the effects of inflation

24.2 ACCA SYLLABUS GUIDE OUTCOME 2:-


Discuss how financial statements may be manipulated to produce a desired
effect (creative accounting, window dressing).

Creative accounting describes the practice of applying inappropriate accounting policies


or entering into complex or ‘special purpose’ transactions with the objective of making a
company’s financial statements appear to disclose a more favourable position,
particularly in relation to the calculation of certain ‘key’ ratios, than would otherwise be
the case81 .

The most criticised area of creative accounting relates to off balance sheet financing.
This occurs where a company has financial obligations that are not recorded on its
balance sheet. Examples:-

• Finance leases treated as operating leases


• Convertible loan stock being classified as equity
• Offsetting liabilities against assets ( e.g. accounts receivable factoring)

81 Paper 2.5 December 2002 Qs 3a(i)

! 237 acowtancy.com
The other main area of creative accounting is that of increasing or smoothing profits.
Examples:-

• The use of inappropriate provisions to reduce profits in good years and increase
them in poor years.
• Not providing for liabilities as they arise

24.3 ACCA SYLLABUS GUIDE OUTCOME 3:-


Explain why figures in a statement of financial position may not be representative
of average values throughout the period for example, due to:
i) seasonal trading
ii) major asset acquisitions near the end of the accounting period.

Seasonal Trading

Companies with a seasonal business often position their year end after the busy season
for practical reasons, but this may also coincide with when the financial position is at its
most solvent.

Window Dressing

Year-end figures are often not representative, because they include year end
accounting adjustments and may be subject to ‘window dressing’.

Major asset acquisitions just before the year end may have the following effects:

1. No/little sales revenue


2. A low relative depreciation charge

! 238 acowtancy.com
24.4 ACCA SYLLABUS GUIDE OUTCOME 4:-
Define and compute relevant financial ratios.
(a) Explain what aspects of performance specific ratios are intended to assess.
(b) Analyse and interpret ratios to give an assessment of an entity’s performance
and financial position in comparison with:
i) an entity’s previous period’s financial statements
ii) another similar entity for the same reporting period
iii) industry average ratios.
(c) Interpret an entity’s financial statements to give advice from the perspectives
of different stakeholders.
(d) Discuss how the interpretation of current value based financial statements
would differ from those using historical cost based accounts.

24.4.1 Profitability Ratios

24.4.1.1 Return on Capital Employed (ROCE)

A business buys assets such as trucks, computers, etc to help makes its operations
more efficient, cut down on costs and make bigger profits.

ROCE shows how well a business has generated profit from its long-term financing.

It is expressed in the form of a percentage, and the higher the percentage, the better.

ROCE is calculated either:

Profit Before Interest and Tax


Total Assets – Current Liabilities

OR

Profit before Interest and Tax


Shareholder’s Equity + Debt

How can firms increase the ROCE ratio?

Movements in return on capital employed are best interpreted by examining profit


margins and asset turnover (in more detail below) as ROCE is made up of these
component parts.

! 239 acowtancy.com
Firms can increase their ROCE ratio by:
(a) Cutting costs so as to increase the profit margin ratio
(b) Increasing the revenue made from their assets, i.e. more efficient use of assets

Limitations of Using ROCE ratio

Be careful when using the ROCE ratio because it does not always yield the correct
percentage.

For instance, a company may simply run down its old assets. This means the
denominator “Total Assets – Current Liabilities” (value of assets is lower) will be lower
and so give a higher ROCE percentage.

In this case, there has been no improvement in operations of the company, in fact the
firm is cutting down on potentially profitable capital investments.

Note

Always compare a company’s ROCE to the interest rate it is charged. The ROCE needs
to be higher.

Similarly if a company pays off a 5% loan, while its current ROCE is 10%, then this is
illogical. It should use the money to get 10% not pay off a loan which only costs 5%.

24.4.1.2 Asset Turnover

Asset turnover shows how efficiently management have utilised assets to generate
revenue.

It is calculated as: -

Revenue
Total assets - current liabilities

When looking at the components of the ratio, a change will be linked to either a
movement in revenue, a movement in net assets, or both.

An increase in asset turnover can result from: -


(a) a significant increase in sales revenue

! 240 acowtancy.com
(b) the business entering into a sale and operating lease agreement, then the asset
base would become smaller, thus improving the result.

24.4.1.3 Return on Equity (ROE)

The ROE ratio reveals how much profit has been made in comparison to shareholder
equity.

A business that has a high return on equity is more likely to be one that is capable of
generating cash internally.

Profit after tax – preference dividends


Average Shareholder Equity

24.4.1.4 Gross Profit Margin

The gross profit margin looks at the performance of the business at the direct trading
level.

Gross profit
Revenue

Variations in the Gross Profit Margin are as a result of:

(i) changes in the selling price/sales volume


(ii) changes in cost of sales.

For example, cost of sales may include inventory write downs that may have occurred
during the period due to damage or obsolescence, exchange rate fluctuations or import
duties.

24.4.1.5 Net Profit Margin

The net profit margin is generally calculated by comparing the profit before interest and
tax of a business to revenue.

Net profit
Revenue

However, the examiner may specifically request the calculation to include profit before
tax.

! 241 acowtancy.com
Analysing the net profit margin enables you to determine how well the business has
managed to control its indirect costs during the period. In the exam, when interpreting
operating profit margin, it is advisable to link the result back to the gross profit margin.

For example, if gross profit margin deteriorated in the year then it would be expected
that the net profit margin would also fall. However, if this is not the case, or the fall is not
so severe, it may be due to good indirect cost control or perhaps there could be a one-
off profit on disposal distorting the operating profit figure.

Lecture Example 1

Comparator assembles computer equipment from bought-in components and distributes


them to various wholesalers and retailers. It has recently subscribed to an inter-firm
comparison service. Members submit accounting ratios as specified by the operator of
the service, and in return, members receive the average figures for each of the specified
ratios taken from all of the companies in the same sector that subscribe to the service.

The specified ratios and the average figures for Comparator’s sector are shown below.

Ratios of companies reporting a full year’s results for periods ending between 1
July 2003 and 30 September 2003:

Return on capital employed 22.1%


Net assets turnover 1.8 times
Gross profit margin 30%
Net profit (before tax) margin 12.5%
Current ratio 1.6:1
Quick ratio 0.9:1
Inventory holding period 46 days
Accounts receivable collection period 45 days
Accounts payable payment period 55 days
Debt to equity 40%
Interest cover 7 times
Dividend yield 6%
Dividend cover 3 times

! 242 acowtancy.com
Comparator’s financial statements for the year to 30 September 2003 are set out below:

Statement of Profit or Loss $000


Sales revenue 2,305
Cost of sales (1,870)
______
Gross profit 435
Other operating expenses (215)
______
Operating profit 220
Interest payable (34)
______
Profit before taxation 186
Income tax (90)
______
Profit after taxation 96
______
Extracts of changes in equity:
Retained earnings – 1 October 2002 179
Net profit for the period 96
Dividends paid (interim $60,000; final $30,000) (90)
______
Retained earnings – 30 September 2003 185
______

! 243 acowtancy.com
Statement of Financial Position $000 $000
Non-current assets (note (i)) 540

Current Assets
Inventory 275
Accounts receivable 320
Bank nil 595
_____ _______
1,135
_______
Share Capital and Reserves
Ordinary shares (25 cents each) 150
Retained Earnings 185
_______
335

Non-current liabilities
8% loan notes 300

Current liabilities
Bank overdraft 65
Trade accounts payable 350
Taxation 85 500
_____ ______
1,135
______

Notes

1) The details of the non-current assets are:

Cost Accum depn Carrying value


$000 $000 $000
At 30 September 2003 3,600 3,060 540

2) The market price of Comparator’s shares throughout the year averaged $6.00
each.

! 244 acowtancy.com
Required:-

a. Calculate the profitability ratios for Comparator equivalent to those


provided by the inter-firm comparison service.

b. Assess the performance of Comparator based on the ratios calculated in


(1) above.

(December 2003 ACCA Paper 2.5 adjusted)

24.4.2 Liquidity Ratios

24.4.2.1 Current Ratio

___Current Assets___
Current Liabilities

The current ratio considers how well a business can cover the current liabilities with its
current assets. It is a common belief that the ideal for this ratio is between 1.5 and 2 : 1
so that a business may comfortably cover its current liabilities should they fall due.

However this ideal should be considered in the context of the company : the nature of
the assets in question, the company’s ability to borrow further to meet liabilities and the
stability of its cash flows.

For example, a business in the service industry would have little or no inventory and
therefore could have a current ratio of less than 1. This does not necessarily mean that
it has liquidity problems so it is better to compare the result to previous years or industry
averages.

24.4.2.2 Quick Ratio

Current Assets – Inventories


Current Liabilities

One of the problems with the current assets ratio is that the assets counted include
inventories which may or may not be quickly sellable (or which may only be sellable
quickly at a lower price).

! 245 acowtancy.com
The ideal ratio is thought to be 1:1, but as with the current ratio, this will vary depending
on the industry in which the business operates.

The quick ratio is also known as the acid test ratio. This name is used because it is the
most demanding of the commonly used tests of short term financial stability.

When assessing both the current and the quick ratios, remember that both of these
ratios can be too high. This would mean too much cash is being tied up in current
assets as opposed to new more profitable investments.

It is important to look at the information provided within the question to consider whether
or not the company has an overdraft at year-end. The overdraft is an additional factor
indicating potential liquidity problems and this form of finance is both expensive (higher
rates of interest) and risky (repayable on demand)

Lecture Example 2

Following from Lecture Example 1:-

a. Calculate the liquidity ratios for Comparator equivalent to those provided


by the inter-firm comparison service.

b. Assess the liquidity position of Comparator based on the ratios calculated


in (1) above.

24.4.3 Efficiency Ratios

24.4.3.1 Receivables Collection Period (in days)

Trade Receivables x 365


Credit Sales

This ratio calculates how long credit customers take to pay.

A short credit period for receivables will aid a business’ cash flow. However, some
businesses base their strategy on long credit periods to achieve higher sales in highly
competitive markets.

! 246 acowtancy.com
If the receivables days are shorter compared to the prior period, it could indicate better
credit control or potential settlement discounts being offered to collect cash more quickly
whereas an increase in credit periods could indicate a deterioration in credit control or
potential bad debts.

24.4.3.2 Payables Payment Period (in days)

__Trade Payables__ x 365


Credit Purchases82

This ratio calculates how long the company takes to pay its suppliers.

An increase in payables days could indicate that a business is having cash flow
difficulties and is therefore delaying payments. It is important that a business pays
within the agreed credit period to avoid conflict with suppliers.

If the payables days are reducing, this indicates suppliers are being paid more quickly.
This could be due to credit terms being tightened or taking advantage of early
settlement discounts being offered.

24.4.3.3 Inventory Days

__Closing (or average) Inventory_ x 365


COS

This ratio calculates how long goods to be sold stay in stock.

Generally, the lower the number of days that inventory is held the better as holding
inventory for long periods of time constrains cash flow and increases the risk associated
with holding the inventory. The longer inventory is held the greater the risk that it could
be subject to theft, damage or obsolescence. However, a business should always
ensure that there is sufficient inventory to meet the demand of its customers.

82 Take cost of sales if credit purchases are not given

! 247 acowtancy.com
24.4.3.4 Working Capital Cycle

A company only gets cash once an item has been in stock and then the debtor pays
(Inventory days + receivables days).

This total should then be reduced by the payable days (the company doesn’t need the
cash until the end of this).

So, the working capital cycle (in days) is:

Inventory (in days) + Receivables (in days) – Payables (in days)

This needs to be kept as small as possible for liquidity purposes.

Lecture Example 3

Following from Lecture Example 1:-

a. Calculate the efficiency ratios for Comparator equivalent to those provided


by the inter-firm comparison service.

b. Assess the performance of Comparator based on the ratios calculated in


(1) above.

! 248 acowtancy.com
24.4.4 Gearing Ratios

24.4.4.1 Gearing

A company can raise money by loans (Debt) or issuing shares (Equity).

Gearing can be calculated either:

_____Debt83______
Debt + Equity 84

OR
___Debt___
Equity

The gearing ratio is of particular importance to a business as it indicates how risky a


business is perceived to be based on its level of borrowing.

High gearing means high debt (in relation to equity). As borrowing increases so does
the risk as the business is now liable to not only repay the debt but meet any interest
commitments under it. If interest rates increase, then the company could be in trouble
unless they have high enough profits to cover this. In addition, to raise further debt
finance could potentially be more difficult and more expensive.

24.4.4.2 Interest Cover

If a company has a high level of gearing it does not necessarily mean that it will face
difficulties as a result of this.

For example, if the business has a high level of security in the form of tangible non-
current assets and can comfortably cover its interest payments, a high level of gearing
should not give an investor cause for concern.

83 Debt = Loans + Preference Shares


84 Equity = Ordinary share capital + Reserves + Non-controlling interest

! 249 acowtancy.com
The interest cover is calculated:

Profit before Interest and Tax (PBIT)


Interest payable

A ratio of at least 3 is deemed to be satisfactory.

The interest coverage ratio is a measurement of the number of times a company could
make its interest payments with its earnings.

It is the equivalent of a person taking the combined interest expense from their
mortgage, credit cards etc, and calculating the number of times they can pay it with their
annual income.

PBIT has its short fallings; companies do pay taxes, therefore it is misleading to act as if
they didn’t. A wise and conservative investor would simply take the company’s earnings
before interest and divide it by the interest expense. This would provide a more
accurate picture of safety.

Lecture Example 4

Following from Lecture Example 1:-

a. Calculate the gearing ratios for Comparator equivalent to those provided


by the inter-firm comparison service.

b. Assess the performance of Comparator based on the ratios calculated in


(1) above.

24.4.5 Investor Ratios

24.4.5.1 Dividend per share

Ordinary dividend
Number of ordinary shares in issue ranking for dividend

The ratio tells equity holders the amount of dividend distributed per share. It will be
influenced by the profitability of the company, its capital structure and dividend policy.

! 250 acowtancy.com
Some companies at some stages of their development choose to distribute a large
percentage of post tax profits. Others – especially those experiencing fast growth – will
want to plough as much as possible back into their business.

24.4.5.2 Dividend cover

It is expressed as:

Earnings after tax and preference dividends = Number of times


Ordinary dividend

This shows how many times over the profits could have paid the dividend. For example,
if the dividend cover is 3, this means that the firm’s profit attributable to shareholders
was three times the amount of dividend paid out.

Dividend cover is a measure of the ability of a company to maintain the level of dividend
paid out. The higher the cover, the better the ability to maintain dividends if profits drop.
This needs to be looked at in the context of how stable a company’s earnings are: a low
level of dividend cover might be acceptable in a company with very stable profits, but
the same level of cover at a company with volatile profits would indicate that dividends
are at risk.

Because buyers of high yield shares tend to want a stable income, dividend cover is an
important ratio for income investors. Dividend cover is the inverse of the dividend
payout ratio.

24.4.5.3 Dividend yield

This is the percentage rate of return by investing in shares at current market price. It is
expressed as:

Dividend per share x 100


Market price per share

! 251 acowtancy.com
This shows how much a company pays out in dividends each year relative to its share
price. It is the return on investment for a share.

It is also a way to measure how much cash flow you are getting for each dollar invested
in an equity position – in other words, how much “bang for your buck” you are getting.

Investors who require a minimum stream of cash flow from their investment portfolio can
secure this cash flow by investing in stocks paying relatively high, stable dividend yields.

If two companies both pay annual dividends of $1 per share, but ABC company’s stock
is trading at $20 while XYZ company’s stock is trading at $40, then ABC has a dividend
yield of 5% while XYZ is only yielding 2.5%. Thus, assuming all other factors are
equivalent, an investor looking to supplement his or her income would likely prefer
ABC’s stock over that of XYZ.

Lecture Example 5

Following from Lecture Example 1:-

a. Calculate the investor ratios for Comparator equivalent to those provided


by the inter-firm comparison service.

b. Assess the performance of Comparator based on the ratios calculated in


(1) above.

24.4.6 How to answer interpretation questions

Step 1 – Look at the movement in cash

If up – How has this happened?

(a) Increased sales? Excellent news. Is this maintainable?


(b) Increased margins? Excellent again but is this sustainable long term?
(c) Better Liquidity? Good receivable, stock and payable management is good news,
be careful we are not missing out on suppliers early settlement discounts though.
(d) Got a loan? How much interest is being charged? Can this interest be paid? Is it
below the ROCE? What is the effect on gearing? What has the proceeds from
the loan been spent on? Assets or covering losses?
(e) Issued shares? What is the effect on dividend cover and yield? Again what has
the money been spent on?

! 252 acowtancy.com
Also what is the ROCE? Should the extra cash be better invested rather than remaining
in the bank account.

The opposite applies if cash is down. Note though that if a loan has been repaid ensure
that the loan is charging interest higher than the current ROCE. If not the money is
better spent investing in new projects if possible.

NOTE: See how in all of these ratios will help!

Step 2

How profitable is the company?

(a) Look at ROCE compared to previous year. What has caused the increase/
decrease?
(b) Look at all the margin ratios – does this help us understand where we are
performing well or badly?
(c) Has an increase in profitability resulted in an increase in cash? If not – then the
company may be overtrading. Overtrading arises when a company expands its
sales revenue fairly rapidly without securing additional long-term capital
adequate for its needs. This means that it is focussing too heavily on profits and
not enough on liquidity. This is dangerous and the symptoms are: -
1. Inventory increasing, possibly more than proportionately to revenue
2. Receivables increasing, possibly more than proportionately to revenue
3. Cash and liquid assets declining at a fast rate
4. Trade payables increasing rapidly

Step 3

How is gearing?

1) If high – is there a problem with interest cover?


2) If low – encourage the company to take out a loan if there are profitable projects.

NOTE: Do all of this but additionally ALWAYS, always, always use the scenario to help
explain the answers – this is where the marks are. Therefore, do not simply list all the
possibilities of why a ratio may have changed; link the reason to the scenario that you
have been provided with.

! 253 acowtancy.com
24.6 ACCA SYLLABUS GUIDE OUTCOME 6:-
Discuss the effect that changes in accounting policies or the use of different
accounting policies between entities can have on the ability to interpret
performance.

It can be useful to compare ratios for an individual company with those of other firms in
the same industry. However, comparing the financial statements of similar businesses
can be misleading because:

• the businesses may use different accounting policies


• ratios may not be calculated according to the same formula (for example, there
are several possible definitions of gearing and ROCE)
• large organisations can achieve economies of scale (e.g. by negotiating
extended credit periods, or discounts for bulk buying with suppliers) while these
measures may not be available to smaller businesses
• companies within the same industry can serve completely different markets and
there may be differences in sales mix and product range. These can affect
profitability and activity ratios such as profit margin and expenses to sales.

24.7 ACCA SYLLABUS GUIDE OUTCOME 7:-


Indicate other information, including non-financial information that may be of
relevance to the assessment of an entity’s performance.

Additional financial information may be required in analysing the performance of a


company such as:

• Budgeted figures
• Other management information
• Industry averages
• Figures for a similar business
• Figures for the business over a period of time.

You may also require other non-financial information such as:

• Market share
• Key employee information
• Sales mix information
• Product range information
• The size of the order book
• The long-term plans of management

! 254 acowtancy.com
24.8 ACCA SYLLABUS GUIDE OUTCOME 8:-
Discuss the different approaches that may be required when assessing the
performance of specialised, not-for-profit and public sector organisations

The main goal of specialised, not-for-profit and public sector organisations is to achieve
value for money. This is in contrast with the aim of a profit-making organisation whose
aim is to achieve a profit or return on capital.

Value for money is achieved by a combination of the three E’s: -

1. Effectiveness – success in achieving its objectives/providing its service


2. Efficiency – how well its resources are used
3. Economy – keep cost of inputs low

As profit and return are not so meaningful, many ratios will have little importance in
these organisations, for e.g. ROCE, gearing, investor ratios in general.

However, such organisations must also keep control of income and costs, therefore
other ratios will still be important such as working capital ratios.

As the main aim of these organisations is to achieve value for money, other non-
financial ratios take on added significance: -

1. Measures of effectiveness such as the time taken to treat out-patients are treated
in a hospital
2. Measures of efficiency such as the pupil-to-teacher ratio in a school
3. Measures of economy such as the teaching time of cheaper classroom
assistants in a school as opposed to more expensive qualified teachers

Further Questions

Question 185

Quartile is in the jewellery retail business which can be assumed to be highly seasonal.
For the year ended 30 September 2014, Quartile assessed its operating performance
by comparing selected accounting ratios with those of its business sector average as
provided by an agency. You may assume that the business sector used by the agency is
an accurate representation of Quartile’s business.

85 Specimen Exam Applicable from December 2014

! 255 acowtancy.com
Which of the following circumstances may invalidate the comparison of Quartile’s ratios
with those of the sector average?
(i) In the current year, Quartile has experienced significant rising costs for its
purchases
(ii) The sector average figures are complied from companies whose year end is
between 1 July 2014 and 30 September 2014
(iii) Quartile does not revalue its properties, but is aware that other entities in this
sector do
(iv) During the year, Quartile discovered an error relating to the inventory count at
30 September 2013. This error was correctly accounted for in the financial
statements for the current year ended 30 September 2014

A. All four
B. (i), (ii) and (iii)
C. (ii) and (iii) only
D. (ii), (iii) and (iv)

Question 286

The following information has been taken or calculated from Fowler’s financial
statements for the year ended 30 September 2014.

Fowler’s cash cycle at 30 September 2014 is 70 days.


Its inventory turnover is six times.
Year-end trade payables are $230,000.
Purchases on credit for the year were $2 million.
Cost of sales for the year was $1·8 million.

What is Fowler’s trade receivables collection period as at 30 September 2014?

All calculations should be made to the nearest full day. The trading year is 365 days.

A. 106 days
B. 89 days
C. 56 days
D. 51 days

86 Specimen Exam Applicable from December 2014

! 256 acowtancy.com
Question 387

Trent uses the formula:

(trade receivables at its year end/revenue for the year) x 365

to calculate how long on average (in days) its customers take to pay.

Which of the following would NOT affect the correctness of the above calculation of the
average number of days a customer takes to pay?

A. Trent experiences considerable seasonal trading


B. Trent makes a number of cash sales through retail outlets
C. Reported revenue does not include a 15% sales tax whereas the receivables
do include the tax
D. Trent factors with recourse the receivable of its largest customer

Question 4

Extracts from the financial statements of Miller Co are shown below:


31 May 2012
$000
Revenue 475
Cost of sales (342)
Gross profit 133
Expenses (59)
Finance cost (26)
Profit before tax 48

What is the interest cover ratio for the year ended 31 May 2012?

A. 2.85
B. 1.85
C. 5.12
D. 0.35

87 Specimen Exam Applicable from December 2014

! 257 acowtancy.com
CHAPTER 25:

STATEMENTS OF CASH FLOWS

25.1 ACCA SYLLABUS GUIDE OUTCOME 1:-


Prepare a statement of cash flows for a single entity (not a group) in accordance
with relevant accounting standards using the direct and the indirect method.

IAS 7, Statements of Cash Flows, splits cash flows into the following headings:
1. Cash flows from operating activities
2. Cash flows from investing activities
3. Cash flows from financing activities

Cash flows

outflows inflows

Cash Cash equivalents

Cash on hand Short term highly liquid


investments e.g. current
investments

! 258 acowtancy.com
25.1.1 Cash flows from operating activities

These represent cash flows derived from operating or trading activities. There are two
methods which can be used to find the net cash from operating activities:- direct and
indirect method.

The indirect method begins with profit before tax from the statement of profit or loss.
This is adjusted for the interest expense and investment income and also for non-cash
items, e.g. depreciation, amortisation, profit/loss on disposal and impairments. It is also
adjusted for increases and decreases in working capital.

Finance Costs – Illustration

Statement of profit or loss $80

SFP

X2 X1
$ $
Interest Payable 100 140

Opening Payable 140


Charge in statement of profit or loss 80

Cash paid (120)


Closing Payable 100

Finance costs go to the operating activities section of the statement of cash flow.

! 259 acowtancy.com
Taxation – Illustration

Statement of Profit or Loss $80

SFP

X2 X1
$ $
Tax Payable 100 140
Deferred Tax 50 80

Opening Payable 140 + 80 220


Charge in Statement of profit or loss 80
300
Cash paid (150)
Closing Payable 100 + 50 150
Tax goes to the operating activities section of the statement of cash flow.

25.1.2 Cash flows from investing activities

These are related to the acquisition or disposal of any non-current assets or


investments together with returns received in cash from investments, i.e. dividends and
interest.

Investment Property – Illustration

Statement of Profit or Loss $80

SFP

X2 X1

Investment Property 200 140

(N.B: There were no purchases of investment property during the year)

Opening 140
Statement of profit or loss 80
200
Cash received (20)
Closing Payable 200

! 260 acowtancy.com
Investment property income goes to the investing activities section of the statement of
cash flows.

Property, plant and equipment

We deal with this slightly differently:

Step 1: Write down the PPE figures per the accounts


Step 2: Work out the cash element of each item (if any)

Illustration

SFP
X2 X1

PPE 200 140

Notes:

Depreciation in year = $50

Revaluation = $100

Disposal = asset sold for $100 making $20 profit

The key here is to try and find the balancing figure which will be additions in the year.

Note: we are dealing with NBVs

Step 1: Write down the PPE figures per the accounts

Opening 140
Depreciation (50)
Revaluation 100
Disposal (80) NBV = 100-20)
Additions 90
Closing 200

The balancing figure is $90 and this is additions.

! 261 acowtancy.com
Step 2: Work out the cash element of each item (if any)

Cash

Opening 140 -
Depreciation (50) -
Revaluation 100 -
Disposal (80) 100
Additions 90 (90)
Closing 200 -
Both the additions and the cash proceeds go the investing activities section of the
statement of cash flows.

25.1.3 Cash flows from financing activities

Financing cash flows comprise receipts from or repayments to external providers of


finance in respect of principal amounts of finance. For e.g.:

(i) Cash proceeds from issuing shares


(ii) Cash proceeds from issuing debentures, loans, notes, bonds,
mortgages and other short or long term borrowings
(iii) Cash repayments of amounts borrowed
(iv) Dividends paid to shareholders

Shares – Illustration

SFP
X2 X1

Share Capital 200 140


Share Premium 150 80

Opening 140 + 80 220


Cash received 130
Closing Payable 200 + 150 350

Share income goes to the financing activities section of the statement of cash flows.

N.B.: If there’s been a bonus issue:

1) Out of share premium – ignore it


2) Out of Retained earnings – reduce the cash by the amount

! 262 acowtancy.com
Loans – Illustration

SFP X2 X1

10% Loan 100 140


Opening 140
Cash paid (40)
Closing Payable 100

Loan repayments go to the financing activities section of the statement of cash flows.

! 263 acowtancy.com
25.1.4 Statement of Cash Flows – Indirect Method

Statement of cash flows for the year ended 31 December 20X7


(INDIRECT METHOD)

$000 $000
Cash flows from operating activities
Profit before taxation 3,390
Adjustment for:
Depreciation 450
Investment income (500)
Interest expense 400
3,740
Increase in trade and other receivables (500)
Decrease in inventories 1,050
Decrease in trade payables (1,740)
Cash generated from operations 2,550
Interest paid (270)
Income taxes paid (900)
Net cash from operating activities 1,380

Cash flows from investing activities


Purchase of property, plant and equipment (900)
Proceeds from sale of equipment 20
Interest received 200
Dividends received 200
Net cash used in investing activities (480)

Cash flows from financing activities


Proceeds from issue of share capital 250
Proceeds from long-term borrowings 250
Dividends paid* (1,290)
Net cash used in financing activities (790)
Net increase in cash and cash equivalents 110
Cash and cash equivalents at beginning of period 120
Cash and cash equivalents at end of period 230
* This could also be shown as an operating cash flow.

! 264 acowtancy.com
Lecture Example 1

The following draft financial statements relate to Tabba, a private company.

Statements of Financial Positions as at

30 September 2005 30 September 2004


$’000 $’000 $’000 $’000
Tangible non-current assets (note (ii)) 10,600 15,800
Current assets
Inventories 2,550 1,850
Trade receivables 3,100 2,600
Insurance claim (note (iii)) 1,500 1,200
Cash and bank 850 8,000 nil 5,560
Total Assets 18,600 21,450

Equity and liabilities


Share capital ($1 each) 6,000 6,000
Reserves:
Revaluation (note (ii)) nil 1,600
Retained earnings 2,550 2,550 850 2,450
8,550 8,450

Non-current liabilities
Finance lease obligations (note (ii)) 2,000 1,700
6% loan notes 800 nil
10% loan notes nil 4,000
Deferred tax 200 500
Government grants (note (ii)) 1,400 4,400 900 7,100

Current liabilities
Bank overdraft nil 550
Trade payables 4,050 2,950
Government grants (note (ii)) 600 400
Finance lease obligations 900 800
Current tax payable 100 5,650 1,200 5,900
Total equity and liabilities 18,600 21,450

The following information is relevant:

(i) Statement of Profit or Loss extract for the year ended 30 September 2005:

! 265 acowtancy.com
$’000
Operating profit before interest and tax 270
Interest expense (260)
Interest receivable 40
Profit before tax 50
Net income tax credit 50
Profit for the period 100

Note: the interest expense includes finance lease interest.

(ii) The details of the tangible non-current assets are:

Cost Accumulated Depreciation Carrying value


$’000 $’000 $’000
At 30 Sept 2004 20,200 4,400 15,800
At 30 Sept 2005 16,000 5,400 10,600

During the year Tabba sold its factory for its fair value $12 million and agreed
to rent it back, under an operating lease for a period of five years at $1 million
per annum, at the date of sale it had a carrying value of $7.4 million based on
a previous revaluation of $8.6 million less depreciation of $1.2 million since
the revaluation. The profit on the sale of the factory has been included in
operating profit. The surplus on the revaluation reserve related entirely to the
factory. No other disposals of non-current assets were made during the year.

Plant acquired under finance leases during the year was $1.5 million. Other
purchases of plant during the year qualified for government grants of
$950,000.

Amortisation of government grants has been credited to cost of sales.

(iii) The insurance claim relates to flood damage to the company’s inventories
which occurred in September 2004. The original estimate has been revised
during the year after negotiations with the insurance company. The claim is
expected to be settled in the near future.

Required:
Prepare a statement of cash flow using the indirect method for Tabba in
accordance with IAS 7 Statements of Cash Flow for the year ended 30
September 2005.

(ACCA Paper 2.5 December 2005 Qs 4)

! 266 acowtancy.com
25.1.5 Statement of Cash Flows – Direct Method

In the direct method, the cash records of the business are analysed for the period,
picking out all payments and receipts relating to operating activities. These are
summarised to give the net figure for the cash flow statement. Not many businesses
adopt this approach as it can be quite time consuming. However, this is the preferred
method under IAS 7.

$000 $000
Cash flows from operating activities
Cash receipts from customers 30,150
Cash payments to suppliers and employees (27,600)
Cash generated from operations 2,550
Interest paid (270)
Income taxes paid (900)
Net cash from operating activities 1,380

Lecture Example 2

The following information is available about the transactions of Mermot, a limited liability
company, for the year ended 31 December 20X1.
$000
Depreciation 880
Cash paid for expenses 2,270
Increase in inventories 370
Cash paid to employees 2,820
Decrease in receivables 280
Cash paid to suppliers 4,940
Decrease in payables 390
Cash received from customers 12,800
Net profit before taxation 2,370
Mermot has no interest payable or investment income.

Required: Compute Mermot’s net cash flow from operating activities for the company’s
cash flow statement for the year ended 31 December 2001 using:
a. Direct method
b. Indirect method

! 267 acowtancy.com
25.2 ACCA SYLLABUS GUIDE OUTCOME 2:-
Compare the usefulness of cash flow information with that of a statement of profit
or loss or a statement of profit or loss and other comprehensive income.

A business may appear profitable on its statement of profit or loss, however if its cash
outflow exceeds its cash inflow over a prolonged period then it will not survive.

Readers of a company's financial statements might also be misled by a reported profit


figure.

1. Shareholders might believe that if a company makes a profit after tax, then this is
the amount which it could afford to pay as a dividend.
2. Employees might believe that if a company makes profits, it can afford to pay
higher wages next year.
3. Survival of a business entity depends not so much on profits as on its ability to
pay its debts when they fall due.

Indeed, a business must generate sufficient cash from its operations to reward the
various stakeholders e.g., shareholders and lenders. An expanding company might
have negative operating cash flow as it builds up the level of its inventories and
receivables in line with the increased turnover. However, an increase in working capital
without an increase in turnover might indicate operational inefficiencies and will lead to
liquidity problems.

Benefits and Drawbacks of the Statement of Cash Flows

One of the most useful financial statements produced by a business is the statement of
cash flow because it provides a clear and understandable picture of cash movements
over the financial year. A statement of cash flow provides useful additional information
that is not provided by the statement of profit or loss. For example, it identifies whether
cash has increased or decreased from one year to the next and also where the cash
has come from.

Statements of cash flow are a useful addition to the financial statements of a company
because accounting profit is not the only indicator of performance. They concentrate on
the sources and uses of cash and are a useful indicator of a company's liquidity and
solvency. Also, users of accounts can readily understand cash flows, as opposed to
statements of profit or loss and statements of financial position which are subject to
manipulation by the use of different accounting policies.

! 268 acowtancy.com
However, the main weakness of a statement of cash flow is that it is a historic
statement. Therefore, it does not indicate whether the business will be able to meet its
debts in the future. A more helpful statement would be a forecast statement of cash flow.

25.3 ACCA SYLLABUS GUIDE OUTCOME 3:-


Interpret a statement of cash flows (together with other financial information) to
assess the performance and financial position of an entity.
Statements of cash flow should help an external user to evaluate the effect of financial
management decisions taken during the year. However, the user will then need to
decide whether these cash movements are normal in relation to past movements and
comparable industry information, or abnormal and require further analysis.
Over time a business needs to generate positive cash flows from its operating activities,
otherwise it is unlikely to survive. Indeed, a business must generate sufficient cash from
its operations to reward the various stakeholders e.g., shareholders and lenders. An
expanding company might have negative operating cash flow as it builds up the level of
its stock and debtors in line with the increased turnover. However, an increase in
working capital without an increase in turnover might indicate operational inefficiencies
and will lead to liquidity problems.
The user of a statement of cash flow can also identify the impact of a business’s
performance in managing working capital. Changes in working capital on operating cash
flow can be analysed together with working capital ratios such as the inventory holding
period, the receivables collection period and the payables payment period. Another
useful ratio to monitor a business’s investment in working capital over time (and against
other businesses) is that of working capital/sales. This ratio shows how much capital is
required to finance operations in addition to the capital required to finance fixed assets.

Further Questions

! 269 acowtancy.com
Question 1

In the year ended 31 May 2012, Galleon Co purchased non-current assets with a cost
of $140,000, financing them partly with a new loan of $120,000. Galleon Co also
disposed of non-current assets with a carrying value of $50,000 making a loss of
$3,000. Cash of $18,000 was received from the disposal of investments during the year.

What should be Galleon Co’s net cash flow from investing activities according to IAS 7
Statement of cash flows?

A. $45,000
B. $75,000
C. $69,000
D. $48,000

Question 2

The following extract is from the financial statements of Pompeii, a limited liability
company at 31 October:
2010 2009
$000 $000
Equity and liabilities
Share capital 120 80
Share premium 60 40
Retained earnings 85 68
–––– ––––
265 188
Non current liabilities
Bank loan 100 150
–––– ––––
365 338
–––– ––––

What is the cash flow from financing activities to be disclosed in the statement of cash
flows for the year ended 31 October 2010?

A. $60,000 inflow
B. $10,000 inflow
C. $110,000 inflow
D. $27,000 inflow

! 270 acowtancy.com
Question 3

Carter, a limited liability company, has non-current assets with a carrying value of
$2,500,000 on 1 December 2007. During the year ended 30 November 2008, the
following occurred:
1. Depreciation of $75,000 was charged to the income statement
2. Land and buildings with a carrying value of $1,200,000 were revalued to
$1,700,000
3. An asset with a carrying value of $120,000 was disposed of for $150,000

The carrying value of non-current assets at 30 November 2008 was $4,200,000.

What amount should be shown for the purchase of non-current assets in the statement
of cash flows for the year ended 30 November 2008?

A. $1,395,000
B. $1,895,000
C. $1,425,000
D. $195,000

Question 4

Which of the following involves a movement of cash?


A. Depreciation of fixed assets
B. Creation of a provision for pensions
C. A rights issue
D. A bonus issue

Question 5

Which one of the following events will reduce the cash balances of a business?
A. Purchase of fixed assets on interest free credit
B. Dividend proposed pending shareholder approval
C. Purchase of stock on credit
D. Creditors paid amounts owed

! 271 acowtancy.com
Question 6

A company with healthy profits is facing a cash shortage. Which of the following events
could account for this?
A. Delaying payments to creditors
B. The recent acquisition of equipment
C. An increase in dividends proposed by the directors
D. The shortening of the credit period granted to debtors

Question 7

What is the immediate effect of making a rights share issue?


A. On profit – Decrease; On cash – Increase
B. On profit – Increase; On cash – Increase
C. On profit – None; On cash – Increase
D. On profit – Increase; On cash – Decrease

Question 8

In IAS 7 Statement of Cash Flows where would you find a bank current account debit
balance?
A. In investing activities
B. In operating activities
C. In financing activities
D. In cash and cash equivalents

! 272 acowtancy.com

You might also like